Final Revision Questions: Rocket

Download as docx, pdf, or txt
Download as docx, pdf, or txt
You are on page 1of 229

Final Revision Questions

2020
– Elite Batch

Rocket
dM
×u
a= d t
æ dM ö
çèM0 - ×t ÷
1. Acceleration of rocket dt ø
dM
F=u
2. Thrust on rocket dt
I = Fav ´ t = Dp
3. Impulse of force
mb v b
vg =
mg
4. Recoil velocity of the gun

OBJECTIVE

1. A reference frame attached to the earth is:


(A) an inertial frame sometimes (B) an inertial frame always
(C) a non-inertial frame (D) may be inertial or non-inertial.
Ans. (C)
Solution: As the earth itself is rotating on its axis the frame attached to it should be a non-
inertial one.

2. A vehicle is moving on a rough road in a straight line with uniform velocity.


(A) No force is acting on vehicle
(B) A force must act on the vehicle
(C) An acceleration is being produced in the vehicle
(D) An centriprital acceleration is being produced in the vehicle

Ans. (A)
Solution: As the vehicle is moving with uniform velocity, net force acting on it should be
zero.

3: Two identical particles A and B, each of mass m, are interconnected by a spring of


stiffness k. If the particle B experiences a force F & the elongation of the relative
spring is x, the relative acceleration between the particles is equal to:
F - kx
(A) F/2m (B) m
F - 2kx kx
(C) m (D) m
Ans. (C)
Join Elite Batch – Physics ( Batch -2 ) - Whatsapp -9264953797

Click Here - https://imjo.in/JKBjbX ( Batch Starting 01 March )


Final Revision Questions
2020
– Elite Batch

Solution : Equation of motion for A : m T T m


F
A
T = ma  B
T kx
=
a = m m
for B : F – T = ma
F−kx
 a = m
F−2 kx
 The relative acceleration ar = | a  a | = m .

4. A 60 kg man stands on a spring balance in a lift. At same instant he finds that the
reading on the scale has changed from 60kg to 50 kg for a while and then comes back
to the original mark. What is his conclusion?
(A) The lift was in constant motion upwards
(B) The lift was in constant motion downwards
(C) The lift while in motion downward suddenly stopped.
(D) The lift while in motion upward suddenly stopped.

Solution: When lift accelerates up W = W


(1+ ag )
When lift decelerates up W = W
(1− ag )
Here weight decreases i.e. lift decelerates up. Initially lift was moving with
constant velocity up and then it suddenly stopped i.e. decelerated,

5. A body is accelerated by applying a force of 30N. The change in the momentum of the
body after 2sec is:
(A) 7.5 kg-m/s (B) 30 kg-m/s
(C) 120 kg-m/s (D) 60 kg-m/s
Solution: P = Ft = 302 = 60 kg m/s
Hence D

6. Two blocks of masses 2kg and 5kg are connected by a light string passing over a
frictionless pulley. The tension in the cord connecting the masses will be
(A) 20N (B) 30N
(C) 28 N (D) 50 N
2 m1 m2
g
Solution: T= m1 +m 2
2×5×2
= × 9 .8
5+2 = 28N
Hence C

Join Elite Batch – Physics ( Batch -2 ) - Whatsapp -9264953797

Click Here - https://imjo.in/JKBjbX ( Batch Starting 01 March )


Final Revision Questions
2020
– Elite Batch

7. A vessel containing water moves with constant acceleration towards the right, along a
horizontal straight line. Which of the following figures: a, b and c represents the
surface of the liquid.

None of the above


(A) (B) (C) (D)
Solution: When the vessel accelerates forward, the pseudo force acts on water backward
and the resultant of the weight of water and the pseudo force, acts on the left
downward. The surface of water is perpendicular to this resultant.
Hence C

8: A bob hanging from the ceiling of the car acts as an accelerometer. Then the relation
expressing horizontal acceleration a of the car and the angle  made by bob with the
vertical is
(A) a = gtan (B) a = gsin
(C) a = gcot (D) None
Hence A
Ma
tan = Mg
a
Solution: Ma

 a = gtan  

Mg

C
9. A weight W is attached to two weightless strings B

AB and AC as shown in figure. The tension in 120


T T
each string will be:
W W A

(A) 4 (B) 2
3W
(C) 2 (D) W W

Join Elite Batch – Physics ( Batch -2 ) - Whatsapp -9264953797

Click Here - https://imjo.in/JKBjbX ( Batch Starting 01 March )


Final Revision Questions
2020
– Elite Batch

Solution: For vertical equilibrium


2T cos60 = W
W
T = 2 cos 60° = W
Hence D

10. A body of mass 2kg is acted upon by two forces each of magnitude 1N, making an
angle 60 with each other. The net acceleration of the body in m/s 2 is
(A) 0.1 (B) 1.0

(C) 3 2 (D) 23
1
Solution: Net force F =
F

√3 = √ 3 m/s 2
F 21 + F22 +2 F 1 F 2 cos60° = √ 12 + 12 +2×1× = √ 3 N
2
=
a= M 2 2
Hence C

11. A scooter of mass 120kg is moving with a uniform velocity of 108km/hr. The force
required to stop the scooter in 10s is
(A) 180N (B) 208N
(C) 360 N (D) 720 N
ΔP 0−mv mv
| |= | |=
Solution: F= Δt Δt Δt
5
(
( 120 kg ) 108 ×
18 )
= 10 = 360N Ans. (C)

12. Two blocks A and B of masse 4kg and 12 kg are placed B


A
on a smooth horizontal plane a force F of 16N is
applied on A, as shown, then the contact force between
F = 16N
the blocks is
(A) 4N (B) 8N
(C) 12N (D) 16N
Solution: Acceleration
F 16
=
a= M 1 +M 2 4 +12
= 1m/s2
P = 12a (for block B)
 F = 16 – M1a = 16 – 4  1 = 12N
Hence C

Join Elite Batch – Physics ( Batch -2 ) - Whatsapp -9264953797

Click Here - https://imjo.in/JKBjbX ( Batch Starting 01 March )


Final Revision Questions
2020
– Elite Batch

13. N bullets, each of mass m, are fired with a velocity v at the rate of n bullets/sec upon a
wall. The bullets are stopped by the wall. The reaction offered by the wall to the
bullets is
Nmv
(A) n (B) nNmv
Nv
n
(C) m (D) nmv
ΔP
Solution: F = Δt
Moment transferred to the wall,
1
P = mv, t = n
mv
= nmv
 F = 1/ A
Hence D

14. A knife edge of mass M is dropped from a height ‘h’ on a wooden floor with its tip
pointing of downward. If the blade penetrates a distance ‘s’ into the wood, the average
resistance offered by the wood to the blade is
 h
 1+ s 
(A) Mg (B) Mg  
2
 h  h
 1-   1+ 
(C) Mg  s  (D) Mg  s 
Solution: Velocity of knife edge when it hits the floor R

v= √ 2gh
v2 2 gh gh a
= =
Retardation, a = 2 s 2s s
 Equation R – Mg = Ma
R = Mg + Ma Mg

gh h
= Mg + M s
= Mg 1+( )
s
Hence B

15. A rocket, set for vertical launching, has a mass of 50 kg and contains 450 kg of fuel. It
can have a maximum exhaust speed of 2 km s –1. If g = 10 ms–2, what should be the
minimum rate of fuel consumption to just lift it off the launching pad?
(A) 2.5 kg s–1 (B) 5 kg s–1
–1
(C) 7.5 kg s (D) 10 kg s–1

Join Elite Batch – Physics ( Batch -2 ) - Whatsapp -9264953797

Click Here - https://imjo.in/JKBjbX ( Batch Starting 01 March )


Final Revision Questions
2020
– Elite Batch

Sol: (A) When the rocket is at the launching pad, M = M 0 = 50 + 450 = 500 kg. Also u = 2kms –
1
= 2000 ms–1.
dM
F=u
dt
But F = Mg. Therefore
dM
u = Mg
dt
dM 500 kg ´ 10 ms- 2
= -1
= 2.5 kg s- 1
or dt 2000 ms

16. In above Q. what should be the minimum rate of fuel consumption to give an initial
acceleration of 20 ms–2 to the rocket ?
(A) 2.5 kg s–1 (B) 5 kg s–1
–1
(C) 7.5 kg s (D) 10 kg s–1
dM
F=u - Mg
Sol: (C) dt
But F = Ma, where a is the initial acceleration. Thus
dM
Ma = u - Mg
dt
dM M(a + g) 500 kg ´ (20 + 10) ms - 2
= =
or dt u 2000 ms- 1
= 7.5 kg s–1

17. A block can slide on a smooth inclined plane of inclination  kept on the floor of a lift.
When the lift is descending with a retardation a. the acceleration of the block relative
to the incline is:
(A) (g + a) sin  (B) (g – a)
(C) g sin  (D) (g – a) sin 
Solution:
N


mg m(g  a)sin 
m(g  a)cos  ma
m(g  a)sin 
 (g  a)sin 
net accel. = m Ans. (A)

18. A force of 5 N acts on a body of weight of 10 N. What is the acceleration in m/s 2


(A) 50 (B) 5
(C) 0.5 (D) 2

Join Elite Batch – Physics ( Batch -2 ) - Whatsapp -9264953797

Click Here - https://imjo.in/JKBjbX ( Batch Starting 01 March )


Final Revision Questions
2020
– Elite Batch

Solution:
5N

10
1
mass = 10
5
 5m / s2
A = (10 /10)
Ans. (B)

19. A man of mass m stands on a crate of mass M. He pulls on a light rope passing over a
smooth light pulley. The other end of the rope is attached to the crate. For the
system to be is equilibrium, the force exerted by the man on the rope will be:
(A) mg (B) Mg
1
M (M  m)g
(C) 2 (D) (m + M)g
m Solution:

T T
M

2T = (M + m)g
(M  m)g
T= 2
Ans. (C)

20. A uniform rope of mass M and length L is pulled along a frictionless inclined plane of
angle  by applying a force F (> Mg) parallel to incline. Tension in the chain at a
distance x from the end at which force is applied is
x x  x
F  Mg sin  F  1- 
(A) L L (B)  L
 x x
F  1-   Mgsin 
(C)  L L (D) None of these
Solution:

Join Elite Batch – Physics ( Batch -2 ) - Whatsapp -9264953797

Click Here - https://imjo.in/JKBjbX ( Batch Starting 01 March )


Final Revision Questions
2020
– Elite Batch

N F

Mgsin 
Mgcos 
F – Mg sin = MA (1)
M  M 
 x  gsin   T   x  A
F– L  L 
m  M  F  Mgsin  
 x gsin    T  x  
F–  L  L  M 
 x
1  N
T = F L
Ans. (D)

21. In the system shown, the acceleration of the wedge of mass 5M is


M
(there is no friction anywhere)
5M
2M (A) zero (B) g/2
600 (C) g/3 (D) g/4
m g  (m  m1 )g
A 2
Solution: m  m2  m1
(6  0.4(m  4))g
A 0
m64
60
m4
0.4
44
m  11kg
 4
Ans. (C)

1 22. What is the maximum value of the force F such that the block
F 
2 3 shown in the arrangement, does not move?
600 m 3kg (A) 20 N (B) 10 N
(C) 12 N (D) 15 N
Solution:
60 N

F Fr1 Fcos 60

F sin60
mg

N = F sin60 + Mg (1)

Join Elite Batch – Physics ( Batch -2 ) - Whatsapp -9264953797

Click Here - https://imjo.in/JKBjbX ( Batch Starting 01 March )


Final Revision Questions
2020
– Elite Batch

Fr1
F cos60 = = N = (F sin60 + Mg)
1
 3 1
2 3 54
  20N
Mg 1 1 3 1 54
   20N
F = cos60   sin60 = 2 2 3 2 = 1
Ans. (A)

  
23. A force vector F = 6 i – 8 j + 10 k newton applied to a body accelerates it by 1 ms –2.
What is the mass of the body?

(A) 10 2 kg (B) 2 10 kg
(C) 10 kg (D) 20 kg

Solution: F=ma
|F| = 10 2
10 2 = m × A = m × 1  m = 10 2 kg. Ans. (A)

24. A flexible chain of weight W hangs between two fixed points A


A B
and B at the same level. The inclination of the chain with the
 
horizontal at the two points of support is . What is the tension
of the chain at the end point.
W W W
cos ec  s ec 
(A) 2 (B) 2
W
sin 
(C) W cos  (D) 3
Solution:
T
T
F 

w w
 cos ec
2T sin = w  T = 2 sin  2
Ans. (A)

25. Two blocks A and B of masses m1 and m2 are placed in contact with each other on a
horizontal platform. The coefficient of friction between the platform and the block A is
1 and that between block B is 2. The platform moves with a constant acceleration a
a as shown in fig. The force of interaction between the blocks
m1 m2
exists if

Join Elite Batch – Physics ( Batch -2 ) - Whatsapp -9264953797

Click Here - https://imjo.in/JKBjbX ( Batch Starting 01 March )


Final Revision Questions
2020
– Elite Batch

(A) 1 = 2 (B) 1  2
(C) 1 < 2 (D) none of these
Solution:
Fr1 Fr2

m1 m2 a

Form of indication will exhibit


Fr1  mg Fr2  mg

By m 1 m2
Fr1 Fr2 1m1g 1m2g
 
m1 m2  m1 m2
 1  2
Ans. (D)

26. For the system shown in the figure, the pulleys are light and frictionless. The tension
in the string will be

2 3
m mgsin  mgsin 
(A) 3 (B) 2
 m
1
mgsin 
(C) 2 (D) 2mgsin 
Solution: mg sin – T = mA (1)
T = mA (2)
(1) + (2)  mg sin = 2mA
gsin 
 A= 2
mgsin 
T= 2 Ans. (C)

27. Two masses M and m are connected by the arrangement shown in figure. What is the
downward acceleration of mass M.

 2M  m   2M  m 
 g   2g
(A)  3M  m  (B)  4M  m 
 Mm   M 
m  g  g
M
(C)  M  m  (D)  2M  m 
Solution: Mg – T = MA (1)
mA
2T – mg = 2 (2)
Join Elite Batch – Physics ( Batch -2 ) - Whatsapp -9264953797

Click Here - https://imjo.in/JKBjbX ( Batch Starting 01 March )


Final Revision Questions
2020
– Elite Batch

2T T
m M
A
2 × (1) + (2)
m
(2M – m) g = A(2M + 2 )
2(2M  m)g
A = 4M  m
Ans. (B)

28. In a rocket, the mass of the fuel is 90% of the total mass. The rocket is blasted from
the launching pad. If the exhaust gases are ejected at a speed of 2 km s –1, what is the
maximum speed attained by the rocket ? (Neglect the effects of gravity and air
resistance).
(A) 2 ln (10) kms–1 (B) 2 ln (30) kms–1
–1
(C) 2 ln (60) kms (D) 2 ln (90) kms–1
Sol: (A) The mass of the container = 10% of the total mass of the rocket, 90% being the
mass of the fuel. Hence M 0/Mc = 10. The rocket will attain the maximum speed v b when
all its fuel is burnt. This speed is given by
æM ö
v b = u ln ç 0 ÷ = 2 kms- 1 ln(10) = 2ln(10)kms - 1
è Mc ø .

29. Blocks A and C start from rest and move to the right
with acceleration aA = 12t m/s2 and aC = 3 m/s2. Here t
A C is in seconds. The time when block B again comes to
rest is
(A) 2s
(B) 1 s
(C) 3/2 s
(D) ½ s
B
Solution:
12 t 3

3
12 t

Join Elite Batch – Physics ( Batch -2 ) - Whatsapp -9264953797

Click Here - https://imjo.in/JKBjbX ( Batch Starting 01 March )


Final Revision Questions
2020
– Elite Batch

12 t  3
A= 2
3 1
And A = 0, t = 12 = 4 Ans. (E)

30. At some instant, the 10 kg mass has acceleration of 12 m/s2, what is the acceleration
F=200 N of 20 kg mass
20 kg
10 kg (A) 2.5 m/s2
  0.1 (B) 4.0 m/s2
(C) 3.6 m/s2
(D) 12 m/s2
Solution:
(1) (2)
10 20 200N

Fr1 max
= 0.1 × 10 ×10
= 10 N
Fr2 max
= 20 × 0.1 × 10 = 20 N
For 10 kg block Kx – 10 = 10A = 10 ×12
Kx = 120 + 10 = 130 N.
For 20 kg block = 200 – 130 = –20 = 20A
50
 2.5m / s
 A = 20
Ans. (A)

31. A block slides down an inclined plane of slope  with constant velocity. It is then
projected up the plane with an initial speed v 0. How far up the incline will it move
before coming to rest
v 02 v 02
(A) 4gsin (B) gsin
v 02 v 02
(C) 2gsin (D) 2g
Solution:

Join Elite Batch – Physics ( Batch -2 ) - Whatsapp -9264953797

Click Here - https://imjo.in/JKBjbX ( Batch Starting 01 March )


Final Revision Questions
2020
– Elite Batch

Fr

mgsin

For constant velocity the net force acting on the block will be O.
 Fr = mg sin
Vo

Fr  mgsin 

mA = mg sin + mg sin
= 2mg sin
 A = 2g sin
O = V02 – 2As
V01 Vo2

s = 2A 2gsin 

32. A force of 100N is applied on a block of mass 3kg as shown in the


12N figure. The coefficient of friction between the wall and block is 0.6. The
magnitude of the force exerted by the wall on the block is
30 (A) 15N downwards (B) 25N upwards
100N
(C) 20N downwards (D) 30N upwards
Solution:

F cos30 = N.  N = 6 3  10.39
F sin 30 = 6N < sin
So. Fr will act upward
And Fr max = N
= 0.6 × 10.39 = 6.23 N.

So force exerted by wall = (6.23)2  (10.4)2


= 15N down ward

Join Elite Batch – Physics ( Batch -2 ) - Whatsapp -9264953797

Click Here - https://imjo.in/JKBjbX ( Batch Starting 01 March )


Final Revision Questions
2020
– Elite Batch

Ans. (A)

33. A force acts for 8 seconds on a body of mass 10 kg initially at rest. The force then stops acting
and the body moves 80 m in next 5s. Calculate the force.
(A) 10 N (B) 20 N
(C) 30 N (D) 40 N
Solution : After the force stops acting, the body moves with a uniform velocity given by
distance travelled 80
v = time taken = 5 = 16 m/s
Initial velocity of the body, u = 0
From v = u + at
16 = 0 + at
or a = 2 m/s2
As F = ma
\ F = 10 ´ 2 = 20N
Ans. (B)

34. A block is gently placed on a conveyor belt moving horizontally with constant speed.
After t = 4s, the velocity of the block becomes equal to the velocity of the belt. If the
coefficient of friction between the block and the belt is  = 0.2, then the velocity of the
conveyor belt is
(A) 2ms–1 (B) 4ms–1
(C) 64ms–1 (D) 8ms–1
Solution:
V0

Fr = mg = 0.2 mg.


A = 0.2g = 2m/s
Let. velocity of belt = V0
So, vel. of block with respect belt will be V0
U = V0, A = 2, t = 4, v = 0
0 = V0 – 2 × 4
V0 = 8 m/s
Ans. (D)

35. A block of mass M is connected to a massless pulley and massless spring


of stiffness k. The pulley is frictionless. The spring connecting the block
and spring is massless. Initially the spring is untstretched when the block
is released. When the spring is maximum stretched, then tension in the
rope is K M
(A) zero (B) Mg
(C) 2Mg (D) Mg/2

Join Elite Batch – Physics ( Batch -2 ) - Whatsapp -9264953797

Click Here - https://imjo.in/JKBjbX ( Batch Starting 01 March )


Final Revision Questions
2020
– Elite Batch

Solution:

by consternation of energy
1
kx
mgx = 2
2mg
x= k
T = kx = 2mg Ans. (c)

36. A balloon has a mass of 10 gram in air. The air escapes from the balloon at a
uniform rate with a velocity of 5 cm/s and the balloon shrinks completely in
2.5s. Calculate the average force acting on the balloon.
(A) 10 Dyne (B) 20 Dyne

(C) 30 Dyne (D) 40 Dyne


Solution : Here, m = 10 gm, v = 5 cm/s
dm 10
=
dt 2.5 = 4 gm/s. F=?
dm
F=
dp
dt
d
= ( mv )
dt =
( )
dt
v

F = 4 ´ 5 = 20 dyne.
Ans. (B)

37. The weights W1 and W2 are suspended from the ends of a light string passing over a
smooth fixed pulley. If pulley is pulled up at an acceleration g, the tension in the string
will be
4W1W2 2W1W2
(A) W1  W2 (B) W1  W2
W1W2 W1W2
W1  W2 2  W1  W2 
(C) (D)
Solution:

Join Elite Batch – Physics ( Batch -2 ) - Whatsapp -9264953797

Click Here - https://imjo.in/JKBjbX ( Batch Starting 01 March )


Final Revision Questions
2020
– Elite Batch

w2

w1
consider w2 and w1 with respect to bully
 w2 
 A
2w – T =  g  (1)
2

w1
A
T – 2w1 = g (2)
(1) × w1 – (2) × w2  T(– w1 – w2) = – 2gw1w2
4w 1w 2
T = w1  w 2
Ans. (A)

38. A block of mass m is placed on a smooth wedge of inclination . The whole system is
accelerated horizontally so that the block does not slip on the wedge. The force
exerted by the wedge on the block has a magnitude
(A) mgtan (B) mgsin
(C) mgcos (D) mgsec
Solution:

m
A

consider F. B. D of m with respect to m
N
mA cos 

mA mgsin 
mA sin 
mg

mgcos 
mg sin = mA cos
 A = g tan
and N = mgcos + mAsin

Join Elite Batch – Physics ( Batch -2 ) - Whatsapp -9264953797

Click Here - https://imjo.in/JKBjbX ( Batch Starting 01 March )


Final Revision Questions
2020
– Elite Batch

gsin2 
= m(gcos + cos  )
= mg sin
Ans. (D)

39. A cube is resting on an inclined plane. What must be the value of coefficient of friction
between cube and plane so that cube topples before sliding
1

(A) 2 (B) <1
1

(C) >1 (D) 2
Solution:

The given condition on only take place when line of force of mg passes troughs
 = 45° …(i)
and mg sin  fr max =  mg cos
   tan
 1
(C)

F=200 N 40. A block is placed on a rough horizontal plane. A


20 kg
10 kg time dependent horizontal force F = K t acts on
  0.1 the block. Here K is a positive constant.
Acceleration-time graph of the block is
a a

(A) (B)
t

a a

(C) (D)

Join Elite Batch – Physics ( Batch -2 ) - Whatsapp -9264953797

Click Here - https://imjo.in/JKBjbX ( Batch Starting 01 March )


Final Revision Questions
2020
– Elite Batch

Solution:
a

F = kt
Fr1 max
Fnet = F – fr = 0 up to time Kt = mg =
mg
And for t > kt
Fnet = kt – mg

Ans. (C)
For Q. no. 41 to 44: A 20 kg box is dragged across a rough level floor ( mk = 0.3) by
means of a rope which is pulled upward at an angle of 30° to the horizontal. The
pulling force has a magnitude 80 N.
41. What is the normal force exerted by the floor on the block?
(A) 160 N (B) 140 N
(C) 180 N (D) 40 N

42. What is the frictional exerted by the floor on the block force?
(A) 48 N (B) 50 N
(C) 52 N (D) 60 N

43. What is the acceleration of the box?


(A) 1.06 m/s2 (B) 2.06 m/s2
2
(C) 2.08 m/s (D) 3.09 m/s2

44. If the force is reduced until the acceleration becomes zero, what is the tension
in the rope?
(A) 54.42 N (B) 55.42 N
(C) 58.42 N (D) 60.42 N

Solution for Q. 41 to 44: Let the normal F


contact force be N
(41) Þ F sin q + N = mg m 
Þ N = mg – F sin q
fk N
= 20 ´ 10 – 80 sin 300 = 160 N.
Ans (A) mg

(42) fk = mk N = (0.3 ) (160) = 48 N. Ans


(A)

Join Elite Batch – Physics ( Batch -2 ) - Whatsapp -9264953797

Click Here - https://imjo.in/JKBjbX ( Batch Starting 01 March )


Final Revision Questions
2020
– Elite Batch

40 √ 3−48
(43) a = (Fcos q - fk)/m = (20 ) = 1.06 m/s2. Ans (A)
(44) F' cos q = fk
Þ F' = fk sec q = 48 ´ (2 / Ö3) = (96/Ö3) = 55.42 N. Ans (B)

45. An elevator accelerates upward at a constat rate. A uniform spring of length L and
mass m supports a small block of mass M that hangs from the ceiling of the elevator.
The tension at distance 1 from the ceiling is T. The acceleration of the elevator is
T T
g g
(A) M  m  ml / L (B) 2M  m  ml /L
T T
g g
(C) M  ml (D) 2M  m  ml /L
Solution:

system

mass of system
m 
 L (L  )  M g
 
m  m 
T   (L  )  M g   (L  )  M A
L  L 
T
g
m
(L  )  M
A= L

46. A particle of mass 70 g, moving at 50 cm/s, is acted upon by a


10 variable force opposite to its direction of motion. The force F is
F(N) shown as a function of time t.
O -2
(A) Its speed will be –50 cm/s after the force stops
-3
4 x 10 -3
8 x 10 10
t(s) acting.
(B) Its direction of motion will reverse.
(C) Its average acceleration will be 1 m/s2 during the interval in which the force acts.
(D) Its average acceleration will be 10 m/s 2 during the interval in which the force
acts.

Join Elite Batch – Physics ( Batch -2 ) - Whatsapp -9264953797

Click Here - https://imjo.in/JKBjbX ( Batch Starting 01 March )


Final Revision Questions
2020
– Elite Batch

10
Solution: Total area under curve = (10 – 2 + 4  10–3) × 2
= (10–2 + 0.4 × 10–2) × 5
 1.4  10 2 
 5
 10 
– = 0.07(V – 0.5)
V = –0.5 m/s
= – 50 cm/s
Ans. (A)

47. Two masses M and m(M>m) are joined by a light string passing over a smooth light pulley
then what is the tension on block
 Mm 
  g.
(A) The acceleration of each block is  M  m 
2Mmg
.
m
(B) The tension in the string is M  m
(C) The centre of mass of the ‘M plus m’ system moves down with an
M 2
 Mm 
g  .
acceleration of  M  m 
(D) The tension in the string by which the pulley is attached to the roof is (M + m)g.
(M  m)
A g
Solution: Mm

m
And for m: Mg – T = MA
(M  m)
T = Mg – MA = Mg – Mg M  m
Mg(2m) 2mMg

= M  m (M  m)
Ans. (A, B)

48. In the figure, the blocks A, B and C of mass m each have acceleration a1, a2 and a3
respectively. F1 and F2 are external forces of magnitudes 2mg and mg respectively.
(A) a1 = a2 = a3
(B) a1 > a3 > a2
(C) a1 = a2, a2 > a3
(D) a1 > a2, a2 = a3

Join Elite Batch – Physics ( Batch -2 ) - Whatsapp -9264953797

Click Here - https://imjo.in/JKBjbX ( Batch Starting 01 March )


Final Revision Questions
2020
– Elite Batch

m m m
A B C
F1 = 2mg m
2m

F2 = mg

Solution:

2mg T T
m m m T
T
mg 2mg mg
2mg 2mg
2mg mg
I II III
2mg – T = 2mA1  2mg –2mA1 = T
Case II and Case III are identical to each other.
So a2 = a3 < a1
Ans. (D)

49. Block A is placed on block B, whose mass is greater than that


of A. There is friction between the blocks, while the ground is
P
A smooth. A horizontal force P, increasing linearly with time,
B begins to act on A. The accelerations a1 and a2 of A and B
respectively are plotted against time (t). Choose the correct
graph.
a1
a1 a1
a2
(A) a2
(B) a2

t t

a1 a1
a2 a2
a1 a2
(C) a2
(D) a1

t t
Solution: Ans. will be B because upto some moment both will move with each other and
after that B will move with constant accel. due to kinetic friction and aceel. Of A
will increase

Join Elite Batch – Physics ( Batch -2 ) - Whatsapp -9264953797

Click Here - https://imjo.in/JKBjbX ( Batch Starting 01 March )


Final Revision Questions
2020
– Elite Batch

50. In the figure, the ball A is released from rest when the spring is at its natural
(unstretched) length. For the block B of mass M to leave contact with the
ground at some stage, the minimum mass of A must be
A (A) 2M
(B) M
M
(C) 2
B M
(D) a function of M and the force constant of the spring
Solution:

m
x
o
M

1 2
kx
mg x = 2
2mg
 x= k (1)
and kx = Mg
Mg
x= k
Mg 2mg

k k  m = (M/2)
Ans. (c)

51. Neglecting friction and mass of pulley, what is the acceleration of mass B
(A) g/3 (B) 5g/2
(C) 2g/3 (D) 2g/5
m A
Solution: mg – T = mA (1)
mA
2T – mg = 2 (2)
m B

Join Elite Batch – Physics ( Batch -2 ) - Whatsapp -9264953797

Click Here - https://imjo.in/JKBjbX ( Batch Starting 01 March )


Final Revision Questions
2020
– Elite Batch

A 2T A
2 A 2

T
B
A
(1) × 2 + (2)
5mA
mg = 2
2g
A= 5 Ans. (D)

52. In the arrangement shown in figure, pulley is smooth and massless


and all the strings are light. Let F1 be the force exerted on the pulley
in case (i) and F2 the force in case (ii). Then
(A) F1>F2 (B) F1<F2
(C) F1 = F2 (D) F1 = 2F2
4m 4m m

m Solution: (c) F1 = F2 because cases are identical.


2m
(i) (ii)
53. A block of mass 2kg is pushed against a rough vertical wall with a force
of 40N, the coefficient of static friction being 0.5. Another horizontal force of 15N, is applied on
the block in a direction parallel to the wall. Will the block move? If yes, in which direction and
with what minimum acceleration ? If no, find the frictional force exerted by the wall on the block.
?
(A) 2 m/s2 (B) 2.5 m/s2
(C) 3 m/s2 (D) 4 m/s2

Solution : The force which may cause the tendency


of motion or motion in the body is its
own weight and the applied horizontal
force of 15N. The resultant of the forces
15N 40N
2 2
F= √ 20 +15 = 25 N
mg
15
In a direction tan-1
( )
20
= 370 with the
vertical.
The friction will, by its very virtue of
opposing the tendency will act in a

Join Elite Batch – Physics ( Batch -2 ) - Whatsapp -9264953797

Click Here - https://imjo.in/JKBjbX ( Batch Starting 01 March )


Final Revision Questions
2020
– Elite Batch

direction opposite to the resultant.


The acceleration is minimum when the resultant force is minimum = F –mN (as,
mN is the maximum frictional force)
= 25 – 0.5 x 40 = 5 N
5
= 2 .5 m/s 2
\ Minimum acceleration is 2 . Ans (B)

54. An inclined plane makes an angle 30 with the horizontal. A


cylinder
A groove OA = 5m cut in the plane makes an angle 30 with
OX. A short smooth cylinder is free to slide down the
influence of gravity. The time taken by the cylinder to reach
0
30

300
O X from A to O is (g = 10 m/s2)
(A) 4s (B) 2s

(C)
2 2s (D) 1s

mgsin2 30
 gsin2 30
Solution: Accel. of m
1
 4  t2
s=5= 2
10  4
t  2sec
 10

m 55. Two blocks of masses m1 = 4kg and m2 = 6 kg are


connected by a string of negligible mass passing over a
m1 frictionless pulley as shown in Fig. The coefficient of
friction between block m1 and the horizontal surface is 0.4.
When the system is released, the masses m1 and m2 start
accelerating. What additional mass m should be placed
over mass m1 so that the masses (m1 + m) slide with a
m2 uniform speed?
(A) 9 kg (B) 10 kg
(C) 11 kg (D) 12 kg
Solution: For m2 T = m2g.
For (m1 + m)
T = (m1 + m)g = 0.4(m1 + m) g = m2g
m2
 m1  m
 0.4
6  10
m  4  11kg
 0.4

Join Elite Batch – Physics ( Batch -2 ) - Whatsapp -9264953797

Click Here - https://imjo.in/JKBjbX ( Batch Starting 01 March )


Final Revision Questions
2020
– Elite Batch

Ans. (C)

56. A block of mass 4 kg is suspended through two light spring balances A and B
as shown in the figure. Then balance A and B will respectively read
A
(A) 4 kg and zero kg
(B) zero kg and 4 kg
(C) 4 kg and 4 kg
B
(D) 2 kg and 2 kg
Solution: (c) If we consider 4 kg block as our system then it will be the reading of B
and if we consider (spring B 4kg block) as our system then spring
4 kg A will give 4kg reading.

57. A block of mass m is placed on another block m


of mass M lying on smooth horizontal
surface, as shown in the figure. The co- M F
efficient of friction between the blocks is m.
What is the maximum horizontal force F that
can be applied to the block M so that the
blocks move together ?
(A) (M + m) g
(B) (M – m) g
(C) (M – mg) 
(D) None of these
Solution: If there is no relative motion between the blocks then acceleration of the blocks
F
is a = M +m
F.B.D. of m relative to M N
For vertical equilibrium N = mg
(1)
ma f
For horizontal equilibrium fl ³ ma
F mg
Þ mg ³ m ( M +m )
Þ F £ (M + m)g
Þ Fmax = (M + m) g. Ans (A)

F2 58. When a force F acts on a body of mass m, the acceleration


produced in the body is a. If three equal forces F1 = F2 = F3 = F
act on the same body as shown in Figure the acceleration
900
produced is
1350
m F1
(A)  
2 1 a
(B)  
2 1 a

Join Elite Batch – Physics ( Batch -2 ) - Whatsapp -9264953797


F3

Click Here - https://imjo.in/JKBjbX ( Batch Starting 01 March )


Final Revision Questions
2020
– Elite Batch

(C) 2a (D) None of these

Solution: Fnet = F  F (1  sin 45)  F cos 45


2 2 2 2 2

= F 1  1  1  2sin45
= F 3 2
F 3 2
a= m Ans. (D)

59. A body is moving down a long inclined plane of angle of inclination . The coefficient
of friction between the body and the plane varies as  = 0.5 x, where x is the distance
moved down the plane. The body will have the maximum velocity when it has travelled
a distance x given by
2
x=
(A) x = 2 tan  (B) tanθ
2
x=
(C)
x = 2 cot  (D) cot 
Solution:
N

Fr

mgsin 

mgcos 

N = mg cos
Velocity will be maximum when Fr max  N  mg sin
0.5x – mg cos = mg sin
x = 2tan
Ans. (A)

60. A man of mass 60 kg is standing on a horizontal conveyer


belt. When the belt is given an acceleration of 1 ms –2, the
a = 1 ms-2
man remains stationary with respect of the moving belt. If
g = 10 ms–2, the net force acting on the man is
(A) 0.6 N (B) 6N
(C) 60 N (D) 600 N
Fr1  60A  60  1
Solution:
F
Fnet = r1 = 60N
Ans. (C)

Join Elite Batch – Physics ( Batch -2 ) - Whatsapp -9264953797

Click Here - https://imjo.in/JKBjbX ( Batch Starting 01 March )


Final Revision Questions
2020
– Elite Batch

1. OBJECTIVE

1. The position of a particle moving on a straight line path is given by :


x = 12 + 18t + 9t 2 metre
Its acceleration at t = 5 5s is:
(A) 9 ms–2 (B) 12 ms–2
–2
(C) 18 ms (D) 45 ms–2
Sol: (C) Acceleration = d2x/dt2 = 18. It does not depend upon time.

d ( t )
 6  3 ( t )
2. The motion of a body is given by the equation dt Where  (t ) at time t is
in ms–1 and t is in seconds. If the body was at rest at t = 0, test the correctness of the
following results are. The terminal speed is
(A) 2 ms–1 (B) 2 ms–2
–1
(C) 3 ms (D) 2 ms
d ( t )
 6  3 ( t )
3. The motion of a body is given by the equation dt
Where  (t ) at time t is in ms–1 and t is in seconds. If the body was at rest at t = 0, test
the correctness of the following results are. The magnitude of the initial acceleration
is
(A) 5 ms–2 (B) 6 ms–2
(C) 8 ms–2 (D) 7 ms–2

d ( t )
 6  3 ( t )
4. The motion of a body is given by the equation dt
Where  (t ) at time t is in ms–1 and t is in seconds. If the body was at rest at t = 0, test
the correctness of the following results are. Then speed is?
  2  1  e 3 t    2  1  e 3 t 
(A) (B)
  2  1 e 3 t
  2  1  e 3 t 
(C) (D)

d ( t )
 6  3 ( t )
5. The motion of a body is given by the equation dt

Join Elite Batch – Physics ( Batch -2 ) - Whatsapp -9264953797

Click Here - https://imjo.in/JKBjbX ( Batch Starting 01 March )


Final Revision Questions
2020
– Elite Batch

Where  (t ) at time t is in ms–1 and t is in seconds. If the body was at rest at t = 0, test
the correctness of the following results are. The speed varies is, when the
acceleration is half the initial value.
(A)  = 2 ms –1 (B)  = 3 ms –1
(C)  = 4 ms –1
(D)  = 1 ms –1

Solution:(2-5) The acceleration of the body at time t is


d (t )
a(t )   6  3 (t ) (1)
dt
2. (A) The terminal speed is the speed when the acceleration is zero. Setting a(t) = 0 in
Eq.(i), we have 0 = 6 – 3 or terminal speed () = 6/3 = 2 ms–1
3. (B) Putting t = 0 in eq.(1), the initial acceleration is a(0) = 6 – 3(0).
Since the body is at t = 0; (0) = 0.
Hence a(0) = 6 – 0 = 6 ms–2
d (t )
 dt
4. (C) Eq.(1) can be written as 6  3
integrating, we have
d (t ) loge  6  3 
 6  3   dt or
3
 t C (2)

where C is the constant of integration. Now   0 at t = 0. Using this in eq.(2) we


loge (6)
C
have 3
Using this value of C in eq.(ii) we have
loge (6  3) loge (6)
t
3 3
or loge (6  3 )  3t  loge (6)
 6  3   
or loge    3t or loge 1    3t
 6   2

or  e 3t or   2  1  e 3t 
1
2
a(0) 6
a(t) =   3ms 2
5. (D) When 2 2 we have from eq. (1)
3 = 6 – 3 or  = 1 ms –1

6. A body thrown vertically upward with velocity 10 m/s then how many height traveled
by body.
(A) 5m (B) 10 m
(C) 15 m (D) 20 m
Solution:(A)

Join Elite Batch – Physics ( Batch -2 ) - Whatsapp -9264953797

Click Here - https://imjo.in/JKBjbX ( Batch Starting 01 March )


Final Revision Questions
2020
– Elite Batch

V 2 =u2 −2 gh
02 =102 −2×10 h
h=5 m
7. A driver applies brakes to the vehicle on seeing traffic signal 400 m ahead. At the time
of applying the brakes vehicle was moving with 15 ms -1 and then starts retarding with
0.3 ms-2. The distance of vehicle after 1 min from the traffic light:
(A) 25m (B) 375m
(C) 360m (D) 40m
Solution: The maximum distance covered by the vehicle before coming to rest
2 2
v (15)
   375 m
2a 2(0.3)
v 15
=t= = =50 s
The corresponding time a 0.3 . Therefore after 50 seconds, the distance
covered by the vehicle = 375 m from the instant of beginning of braking.
 The distance of the vehicle from the traffic after one minute
= (400 - 375) m = 25 m
 (A) is the correct answer.

8. Which of the following graph correctly represents velocity-time relationship for


a particle released from rest to fall freely under gravity?
v
v

t
t

v v

t t

Solution: Releasing of the particle from rest means that v 0 =0 at t = 0 and v =gt
at any time t.
 the slope of v/t graph is a constant.
 v/t graph is a straight line passing through the origin.
 (A)

Join Elite Batch – Physics ( Batch -2 ) - Whatsapp -9264953797

Click Here - https://imjo.in/JKBjbX ( Batch Starting 01 March )


Final Revision Questions
2020
– Elite Batch

9. A point moves rectilinearly with deceleration whose modulus depends on the velocity
v of the particle as a  k v where k is a constant, k  0 . At the initial moment the
velocity of particle is v0 What time will it take to cover that distance?
2 v0 v
t= t= 0
(A) k (B) k
v 3 v0
t= 0 t=
(C) 2k (D) k
dv dv dx
 k v   k v
Solution:(A) dt dx dt

0 x
dv
v  k v   vdv   k .dx
dx v0 0

2 2 3/2
  v 03 / 2  kx or Distance x  vo
3 3k and
0 t
dv
dv  k vdt      k dt
v0 v 0
0
or 2  v   kt
v0

or -2 v 0  kt
2 v0
or t =
k

10. A body starts from rest and acquires a velocity of 4 ms –1 during a displacement of magnitude
4m. Its acceleration is:
(A) 0.5 ms–2 (B) 1 ms–2
(C) 2 ms–2 (D) 4 ms–2

Sol: (C)
v 2 - v 0 2 = 2ax . Hence v = 4 ms–1, v = 0 and x = 4m. Thus a = 2 ms–2.
0

11. At height point velocity of body?


(A) Max (B) Min
(C) Zero (D) None of these
Solution: (C)

12. A body starts from rest and moves with a constant acceleration of 20 ms –2. After 10s the
velocity will be:
Join Elite Batch – Physics ( Batch -2 ) - Whatsapp -9264953797

Click Here - https://imjo.in/JKBjbX ( Batch Starting 01 March )


Final Revision Questions
2020
– Elite Batch

(A) 0.2 ms–1 (B) 2 ms–1


(C) 20 ms–1 (D) 200 ms–1

Sol: (D)
v = v 0 + at = (0 + 20 ´ 10)ms - 1 = 200 ms - 1 .

13. A stone falls from a balloon that is descending at a uniform rate of 12 m/s. The displacement
of the stone from the point of release after 10 sec is
(A) 490 m (B) 510 m
(C) 610 m (D) 725 m
1 2 1 
S  ut  at  (12  10)    9.8  100 
Solution: (C) 2 2 
 120  49  610

14. A particle moves along X-axis in such a way that its coordinate X varies with time t according
to the equation x  (2 – 5t + 6t2) m. The initial velocity of the particle is
(A) – 5 m/s (B) 6 m/s
(C) – 3 m/s (D) 3 m/s
Solution: (A) x  (2  5t  6t ) mt
2

dx
 5  12t
V  dt
V at t  0 is  5 m/s

15. An athlete completes one round of a circular track or radius R in 40 sec. What will be his
displacement at the end of 2 min. 20 sec.
(A) Zero (B) 2R
(C) 2R (D) 7R
Solution: (B) 2 min 20sec  120  20  140 sec
140 20  1
3   3   trip
40 40  2
So displacement  2R

16. A body is thrown vertically upwards. If air resistance is to be taken into account, then the time
during which the body rises is
(A) Equal to the time of fall (B) Less than the time of fall
(C) Greater than the time of fall (D) Twice the time of fall
Solution: (B) a down < a up
So t down > t up

17. A mass m slips along the wall of a semispherical surface of radius R.


The velocity at the bottom of the surface is
m
R

Join Elite Batch – Physics ( Batch -2 ) - Whatsapp -9264953797

Click Here - https://imjo.in/JKBjbX ( Batch Starting 01 March )


Final Revision Questions
2020
– Elite Batch

(A)
Rg (B)
2Rg

(C)
2 Rg (D)
Rg
Solution: (B) By conservation of energy
1
 mgR  mv 2  v  2gR
2

18. A particle moving with constant acceleration covers a distance of 30m in the 3rd second. It
covers a distance of 50 M is the 5th second. What is the acceleration of the particle ?
(A) 3 ms–2 (B) 5 ms–2
–2
(C) 8 ms (D) 10 ms–2
xn = v 0 + (a / 2)(2n - 1)
Sol: (D) Use

19. The initial velocity of a body moving along a straight line is 7 m/s. It has a uniform
acceleration of 4 m/s2. The distance covered by the body in the 5th second of its motion is
(A) 25 m (B) 45 m
(C) 50 m (D) 85 m
1 1
Solution: (A) S5  u  2 a (2t  1)  7  2  4(10  1)  7  18  25mt.

20. Which of the following four statements is false


(A) A body can have zero velocity and still be accelerated
(B) A body can have a constant velocity and still have a varying speed
(C) A body can have a constant speed and still have a varying velocity
(D) The direction of the velocity of a body can change
when its acceleration is constant

Solution: (B)

21. An aero plane is moving with horizontal velocity u at height h. The speed of a packet dropped
from it on the earth’s surface will be (g is acceleration due to gravity)

(A) u2 + 2gh (B)


2gh

(C) 2gh (D) u2 - 2gh


Solution: (A)
v y  2gh

v total  u2  2gh

22. The displacement of a particle is given by y  a + bt + ct2 –dt4. The initial velocity and
acceleration are respectively

Join Elite Batch – Physics ( Batch -2 ) - Whatsapp -9264953797

Click Here - https://imjo.in/JKBjbX ( Batch Starting 01 March )


Final Revision Questions
2020
– Elite Batch

(A) b, –4d (B) –b, –2c


(C) b, –2c (D) 2c, –4d
Solution: (C) y  a  bt  ct2  dt4
V  b  2ct  4dt3
A  2c  12dt2
at t  0, v  b, A  2C

23. A truck and a car are moving with equal velocity. On applying the brakes both will stop after
certain distance, then
(A) Truck will cover less distance before rest
(B) Car will cover less distance before rest
(C) Both will cover equal distance
(D) None
F
, mt  mc  A t  A c
Solution: (B) A  m
u2
s   S t  Sc
2a

24. The distance traveled by a particle is proportional to the squares of time, then the particle
travels with
(A) Uniform acceleration (B) Uniform velocity
(C) Increasing acceleration (D) Decreasing velocity
Solution: (A) S  kt  v  2kt  a  2k  constant .
2

25. The graph of displacement v/s time is

Its corresponding velocity-time graph will be

V
V

t
t

Join Elite Batch – Physics ( Batch -2 ) - Whatsapp -9264953797

Click Here - https://imjo.in/JKBjbX ( Batch Starting 01 March )


Final Revision Questions
2020
– Elite Batch

V V

t t

Solution: (A)
s
S vs. t is parabola
ds
0
So V vs. t must be line and at t = 0 dt
t

26. The initial velocity of a particle is u (at t = 0) and the acceleration a is given by at. Which of
the following relation is valid
t2
v u  a
(A) v = u + a t2 (B) 2
(C) v=u+at (D) v=u
2
at
v  u
Solution: (D) A  at  2

27. The initial velocity of the particle is 10 m/sec and its retardation si 2 m/sec 2. The distance
moved by the particle in 5th second of its motion is
(A) 1m (B) 19 m
(C) 50 m (D) 75 m
Solution: (C) u  10, a  2
v  0  10 2  t  t  5
1
t 5  10   2(10  1)  1mt.
2
28. Pick up the correct statements:
(A) Area under a-t graph gives velocity
(B) Area under a-t graph gives change in velocity
(C) Path of projectile as seen by another projectile is a parabola,
(D) A body, whatever be its motion, is always at rest in a frame of reference fixed to
the body itself.

(A) 
a dt   dv  v 2  v1
Solution:

Join Elite Batch – Physics ( Batch -2 ) - Whatsapp -9264953797

Click Here - https://imjo.in/JKBjbX ( Batch Starting 01 March )


Final Revision Questions
2020
– Elite Batch

29. A body is moving in a circle at a uniform speed  . What is the magnitude of the change in
velocity when the radius vector describes an angle 
 
2 cos  
(A)  cos (B) 2
 
2 sin  
(C)  sin (D) 2
 
v f  vi  v  v  2v cos 
2 2 2

Solution: (D) 0 0 0

 v 02 2(1  cos )
 v 02 2  2 sin2   / 2   2v 0 sin(  / 2)

30. A bicyclist encounter a series of hills uphill speed is always v 1 and down hill speed is always
v2. The total distance travelled is , with uphill and downhill portions of equal length. The
cyclist's average speed is
v1 + v2 v 21 + v 22
(A) 2 (B) v1 + v2
v1 v2 2 v1 v2
(C) v1 + v2 (D) v1 + v2
Solution: (D) For n down and n up hills
ns ns
T  (s  length of onehill)
v1 v 2
l  2ns
l 2ns 2v1v 2
v  
T  1 1  v1  v 2
ns   
 v1 v 2 

31. Choose the wrong statement


(A) Zero velocity of a particle does not necessarily mean that its acceleration is zero.
(B) Zero acceleration of a particle does not necessarily mean that its velocity is zero.
(C) If the speed of a particle is constant, its acceleration must be zero.
(D) None of these
Solution: (D)

32. A particle is moving eastward with a speed of 5 m/s. After 10 seconds, the direction changes
towards north, but speed remains same. The average acceleration in this time is
1
(A) zero (B) √2 m/s2 towards N-W

Join Elite Batch – Physics ( Batch -2 ) - Whatsapp -9264953797

Click Here - https://imjo.in/JKBjbX ( Batch Starting 01 March )


Final Revision Questions
2020
– Elite Batch

1 1
(C) √2 2 √2 m/s2 towards S-W
 m/s towards
 N-E (D)

Solution: (B) i
v  5i, v f  5j
 
v i  v f  5j  5i
 
v f  vi
a 
t (j - i) / 2 so direction is W  N
 1
a
2

33. A ballast bag is dropped from a balloon that is 300 m above the ground and rising at 13 m/s.
The time before the bag hits the ground is [take g = 10 m/s 2]
(A) 10 sec (B) 9.8 sec
(C) 9.5 sec. (D) 9.15 sec.
Solution: (D)  300  13t  ½  10  t2

5t2  13t  300  0


13  165  6000
 9.15 sec
t 10

34. A stone is thrown vertically upwards with a velocity 30 ms –1. If the acceleration due to gravity
is 10 ms–2 , what is the distance travelled by the particle during the first second of its motion ?
(A) 10 m (B) 25 m
(C) 30 m (D) None of the above.
1 2
x = v0t - gt
Sol: (B) 2
1
= 30 ´ 1 - ´ 10 ´ 1 = 25 m.
2
35. Displacement (x) of a particle is related to time (t) as x = at + bt 2 – ct3 where a, b and are
constants of motion. The velocity of the particle when its acceleration is zero is given by
b2 b2
a a
(A) c (B) 2c
2
b b2
a a
(C) 3c (D) 4c
Solution: (C) x  at  bt  ct , v  a  2bt  3ct
2 3 2

A  2b  6 ct2
b
A  0  t  3c

Join Elite Batch – Physics ( Batch -2 ) - Whatsapp -9264953797

Click Here - https://imjo.in/JKBjbX ( Batch Starting 01 March )


Final Revision Questions
2020
– Elite Batch

b
Velocity at t  3c
2b2 b2

a 3c 3c
b2
a
3c

36. A particle moves along a parabolic path y = 9x 2 in such a way that the x component of
1
ms 1
velocity remain constant and has a value 3 . The acceleration of the particle is
1
j ms 1 

(A) 3 (B) 3 j ms 2
2
j ms 2 

(C) 3 (D) 2 j ms 2

Solution: (D) y  9x 2

dy
 Vy  18x Vx
dx
A y  18VX2 and A x  0
1
A y  18x  2m / s2
9

37. The co-ordinate of the particle in x-y plane are given as


x  2 + 2t + 4t2 and y  4t + 8t2
The motion of the particle is
(A) along a straight line (B) uniformly accelerated
(C) along a parabolic path (D) nonuniformly accelerated
Solution: (A) x  2  2t  4t2
… (1)
y  4t  8t 2
… (2)
y
So path  x  2  2  line

38. A car start from rest then after 100 s is velocity become 100 cm/s then. What is acceleration?
(A) 2 (B) 5
(C) 1 (D) 4
Solution: (C)
V  u  at
100  0  100  a
a  1m / s2
Join Elite Batch – Physics ( Batch -2 ) - Whatsapp -9264953797

Click Here - https://imjo.in/JKBjbX ( Batch Starting 01 March )


Final Revision Questions
2020
– Elite Batch

39. The position of a particle moving along the x-axis is given by x = 3t - 4t 2 + t3, where x is
in meters and t in seconds. Find the position of the particle at t = 2s.
(A) 2 m (B) +2 m
(C) 3 m (D) 1 m

40. The position of a particle moving along the x-axis is given by x = 3t - 4t 2 + t3, where x is
in meters and t in seconds. Find the displacement of the particle in the time interval
from t = 0 to t = 4s.
(A) 13 m (B) 12 m
(C) 10 m (D) 8m

41. The position of a particle moving along the x-axis is given by x = 3t - 4t 2 + t3, where x is
in meters and t in seconds. Find the average velocity of the particle in the time interval
from t=2s to t = 4s.
(A) 8m / s (B) 2m / s
(C) 7m / s (D) 3m / s

42. The position of a particle moving along the x-axis is given by x = 3t - 4t 2 + t3, where x is
in meters and t in seconds. Find the velocity of the particle at t =2s.
(A) 2m / s (B) 5m / s
(C) 1m / s (D) 1m / s

43. The position of a particle moving along the x-axis is given by x = 3t - 4t 2 + t3, where x is
in meters and t in seconds. Find the acceleration of the particle at time t = 2 sec.

(A)
4 m / sec 2 (B)
3 m / sec 2
2 2
(C) 2 m / sec (D) 6 m / sec
44. The position of a particle moving along the x-axis is given by x = 3t - 4t 2 + t3, where x is
in meters and t in seconds. Find the average acceleration of the particle for the time
interval t = 2 sec to t = 4 sec.
2
(A) 8 m / sec (B) 10 m / sec 2
(C) 4 m / sec (D) 10 m / sec
Solution: (39-44)
39. (a) x(t) = 3t  4t2 + t3
 x(2) = 3  2  4  (2)2 + (2)3
= 6  4  4 + 8 = 2 m.
40. (b) x(0) = 0
x(4) = 3  4  4  (4)2 + (4)3 = 12m.

Join Elite Batch – Physics ( Batch -2 ) - Whatsapp -9264953797

Click Here - https://imjo.in/JKBjbX ( Batch Starting 01 March )


Final Revision Questions
2020
– Elite Batch

Displacement = x(4)  x(0) = 12 m.


x ( 4 )− x ( 2 ) 12−(−2 )
¿ v >= = m / s=7 m/ s
41. (c) ( 4−2 ) 2
dx
=3−8 t+3 t 2
42 (d) dt

 v(2) =
( dxdt )
( 2)

=3−8×2+3×( 2 )2 =−1 m/s


d2 x
a ( t )= =−8+6 t
43. (a) dt 2
2
⇒ a ( 2 )=4 m /sec
4

∫ a ( t ) dt
2
¿ a> = 4

∫ dt
44. (b) 2
4
[−8 t+ 3t 2 ]9
=
2
=10 m /sec2
45. A car starts moving with constant acceleration covers the distance between two
points 180 meters apart in 6 seconds. Its speed as it passes the second point is 45
m/s. Find its acceleration.
(A) 5m/s2 (B) 8m/s2
(C) 3m/s (D) 2m/s

46. A car starts moving with constant acceleration covers the distance between two
points 180 meters apart in 6 seconds. Its speed as it passes the second point is 45
m/s. Find its speed when it was at the first point.
(A) 15 m/s2 (B) 15 m/s
(C) 10 m/s (D) 5 m/s

47. A car starts moving with constant acceleration covers the distance between two
points 180 meters apart in 6 seconds. Its speed as it passes the second point is 45
m/s. Find the distance from the first point when it was at rest.
(A) 2m (B) 22 m2
(C) 22.5 m (D) 0.5 m

Solution :(45-47)

Join Elite Batch – Physics ( Batch -2 ) - Whatsapp -9264953797

Click Here - https://imjo.in/JKBjbX ( Batch Starting 01 March )


Final Revision Questions
2020
– Elite Batch

45. (a) Let the acceleration of the car be a and its speed at first point be u.
 v = u + at
 45 = u + at (1)
1 2
at
Also 180 = ut + 2 (2)
Multiplying equation (1) by t and subtracting from (2), we get
1 2
at
180  45t =  2
1
− a×36
 180  45  6 = 2
 a = 5m/s2.
46 (b) From (1)
u = 45  5  6 = 15 m/s.
47 (c) Let the distance be S
 v2 = u2 + 2aS
2
m 5m

( )15
s
=0+2× 2 ×S
s
 S = 22.5 m
48. A body is dropped from the top of the tower and falls freely. The distance covered by it after
n seconds is directly proportional to
(A) n2 (B) n
(C) 2n – 1 (D) 2n2 – 1

Sol: (A) Here


h = (1/ 2)gt 2 , hn = (1/ 2)g (2n - 1), v = gt.

49. In the above question, the distance covered in the nth second is proportional to
(A) n2 (B) n
(C) 2n – 1 (D) 2n2 – 1

Sol: (C) Here


h = (1/ 2)gt 2 , hn = (1/ 2)g (2n - 1), v = gt.

50. In the question 59, the velocity of the body after n seconds is proportional to
(A) n2 (B) n
(C) 2n – 1 (D) 2n2 – 1

Sol: (B) Here


h = (1/ 2)gt 2 , hn = (1/ 2)g (2n - 1), v = gt.

51. A body when projected vertically up covers a total distance D. During the time of its
flight is t. If there were no gravity, the distance covered by it during the same time is
equal to
(A) 0 (B) D
(C) 2D (D) 4D

Join Elite Batch – Physics ( Batch -2 ) - Whatsapp -9264953797

Click Here - https://imjo.in/JKBjbX ( Batch Starting 01 March )


Final Revision Questions
2020
– Elite Batch

Solution: The displacement of the body during the time t as it comes back to the point of
projection
 S=0
1 2v 0
t=
 v0t  2 gt2 = 0  g
During the same time t, the body moves in absence of gravity through a distance D  = v.t,
because in absence of gravity g = 0
2v 0 2v 20
 D =
v 0×( )
g
=
g …(1)
In presence of gravity the total distance covered is
2 2
v v
2 0= 0
= D = 2H = 2g g …(2)
(A)  (B)  D = 2D
Hence, (C) is correct

52. Velocity each type of physical quantity


(A) Vector (B) Scalar
(C) Tensor (D) None of these
Solution (a)

53 Which is correct ?
a
St th  u  (2n  3)
(A) 2 (B) V 2  u 2  2as
a a
St th  u  (2n  1) St th  u  (2n  1)
(C) 2 (D) 2
Solution (B) V  u  2as
2 2

54 Accretion is which type of physical quantity


(A) Vector (B) Scalar
(C) Tensor (D) None of these
Solution (a)

55 Which is correct ?
a a
St th  u  (2n  3) St th  u  (2n  1)
(A) 2 (B) 2
a a
St th  u  (2n  1) St th  u  (2n  1)
(C) 2 (D) 2

Join Elite Batch – Physics ( Batch -2 ) - Whatsapp -9264953797

Click Here - https://imjo.in/JKBjbX ( Batch Starting 01 March )


Final Revision Questions
2020
– Elite Batch

Solution (B)

56.: A particle starts from rest with acceleration  for some time and after achieving a
maximum velocity starts retarding at rate  and finally comes to rest. If total time
taken is t then Determine (a) maximum velocity (b) total distance travelled.
 t  t 2  t  t 2
, ,
          2    
(a) (b)
  t2 t2
t,      (   )t ,     
(c) 2 2 (d) 2

Solution: (b) Let the particle accelerate for a time t then maximum, velocity v   t1


t
t1 t-t1

0   t1    t  t1 
Since it retards at a rate  and finally comes to rest therefore
t  t
t1   vmax 
Or    
Distance travelled = area under (v -t) graph
1
 vmax .  total time 
2
1  t  t 2
  t=
2  2    

57. The displacement of a particle moving in one dimension is given by t  x  3 where


x is in meter and t in second. The displacement, when the velocity is zero is
(a) 3m (b) 1m

Join Elite Batch – Physics ( Batch -2 ) - Whatsapp -9264953797

Click Here - https://imjo.in/JKBjbX ( Batch Starting 01 March )


Final Revision Questions
2020
– Elite Batch

(c) 1.8 m (d) None of these

Key concept: Differentiate the relation t  x  3 & analyse.


Solution: (d)
dx
x   t  3 and  2  t  3
2

dt
dx
v  0 when t  3
dt
x   3  3  0
2

58. A body is moved along a straight line path by a machine delivering constant power.
The distance moved by the body in time t is proportional to
3 1
(a) t 2
(b) t 4

1 3
(c) t 2
(d) t 4

Key concept: Use formula, power = force velocity


Solution:
dV P.dt
Power P  F .v  m
dt
v  vdv   m
v2 P
or  t  C when t  0, v  0  C =0
2 m
2 Pt dx 2 Pt
or v  or 
m dt m

2 P 12 3
 dx  m  t dt or x  t 2

So option (a) is correct.

59. The radius vector of a point depends on time t as



  bt 2
r  ct 
2

Join Elite Batch – Physics ( Batch -2 ) - Whatsapp -9264953797

Click Here - https://imjo.in/JKBjbX ( Batch Starting 01 March )


Final Revision Questions
2020
– Elite Batch

 
where c and b are constant vectors. Find the modulus of velocity and acceleration at
any time t.
        
V  c . c  b . bt 2 +2c . bt a  b
(a) ,
        
V  c . c  b . bt 2 +2c . bt a  b
(b) ,
        
V  c . c  b . bt 2 +2c . bt a  b
(c) ,
(d) None of these

dr  

V  c  bt
Solution: (a) Velocity dt

V  c 2  b2t 2  2cbt cos 
Modulus of velocity vector will be
   
Here c and b are modulus of c and b and  is the angle between c and b which can
be written
  2    
As c 2
 c . c , b  b . b and cb cos   c .b
      
V  c . c  b . bt 2 +2c . bt
Hence,

 dv 
a b
(b) Acceleration dt
 
a  b
Hence,

60. A ball is thrown upwards from the top of a tower 40m high with a velocity of 10m/s.
Find the time when it strikes the ground.
(a) 2sec. (b) 4sec.
(c) 8sec. (d) None of these
Key concept: use proper sign rule
Solution: (B) s =ut+ ½ at2
s=ut+ ½ at2 u= +10m/s and g =-10m/s
s=0 at origin
o

- 40 = 10t -½10t 2
Hight of tower s= - 40m

Join Elite Batch – Physics ( Batch -2 ) - Whatsapp -9264953797

Click Here - https://imjo.in/JKBjbX ( Batch Starting 01 March )


Final Revision Questions
2020
– Elite Batch

after solving t= 4 sec and


t= -2 sec.
Taking positive value t= 4sec.

Objective
1. A glass marble projected horizontally from the top of a table falls at a distance x
from the edge of the table. If h is the highest of the table, then the velocity of
projection is
g g
h x
(A) 2x (B) 2h
(C) gxh (D) gx + h.
2h 2h g
t= ,x   or   x
Solution : (b) g g 2g

2. The trajectory of a projectile in a vertical plane is y = ax – bx 2, where a, b are constants, x and y are
respectively the horizontal and vertical distances of the projectile from the point of projection.
The maximum height attained is
a
2
a2
(A) 4b (B) 8b
a2 a 2

(C) 2b (D) 2 gb

Solution: y = ax- bx2 ;


dy d2 y
=0∧ 2
For maximum y, dx dx < 0
dy a
dx = a – 2bx = 0  x = 2b
2
d y a
=−2b <0
dx 2  for x = 2b , y is maximum.
2 2
a a a
 ymax = a
( ) ( )
2b
−b
2b
=
4b .

3. Which of the following parameters of a particle executing uniform ircular motion remains
constant?
(A) speed (B) radial acceleration (C) angular velocity (D) all of these

Join Elite Batch – Physics ( Batch -2 ) - Whatsapp -9264953797

Click Here - https://imjo.in/JKBjbX ( Batch Starting 01 March )


Final Revision Questions
2020
– Elite Batch

(A) A, B (B) B,C


(C) A,C (D) C, D

Solution: (C) The magnitude of the velocity of the particle, that is speed v remains constant.
The velocity vector changes from time to time. Therefore, it accelerates radically
v2
inwards. This is known as Centripetal acceleration, a r. Its magnitude r remains
constant whereas its direction changes from time to time. Therefore a
⃗ r is not
constant. Since the sense of rotation of the particle remains constant its angular
velocity ω⃗ remains constant in direction. Since ⃗v = Constant & ⃗v =⃗ω ×⃗r , ω ⃗
remains constant.
 A, & C are correct.

4. A particle is projected horizontally from the top of a cliff of height H with a speed √ 2gH . The
radius of curvature of the trajectory at the instant of projection will be
(A) H/2 (B) H
(C) 2H (D) 

Solution: (C) Since, ⃗g ⊥ {⃗v ¿


Radial acceleration ar = g
v 20
=g
 r where r is the radius of curvature.
2 gH
=g (∵ v =√ 2gH)
 r
 r = 2H

5. A body falling freely from a given height H hits an inclined plane in its path at a height ‘h’. As a
result of this impact the direction of the velocity of the body becomes horizontal. Find the total
time the body will take to reach the ground.
2
(A) √
2
g
( √ h+ √ H−h )
(B) √ 2gh
(C) √
g
( √ H −h )
(D) none of the above

Solution : (A) Time taken by the body to


H H-h
strike the inclined plane
2( H−h)
t1 = √
Now as after impact the velocity
g h

of the body is horizontal, so time

Join Elite Batch – Physics ( Batch -2 ) - Whatsapp -9264953797

Click Here - https://imjo.in/JKBjbX ( Batch Starting 01 March )


Final Revision Questions
2020
– Elite Batch

taken to reach the ground -


2h
t2 =
So total time of motion -
√ g

2
t = t1 + t2 = √ g
[ √h+ √ H −h ]

6. If R is the range for an angle of projection of 15 o with the horizontal, then the other angle of
projection for which the range is R, is
(A) 75o (B) 60o
(C) 45o (D) 30o.
Solution : (a)  = (90o – 15o) = 75o

7. An object is thrown along a direction inclined at an angle of 45 o with the horizontal direction.
The horizontal range of the particle is
(A) four times the vertical height (B) thrice the vertical height
(C) twice the vertical height (D) equal to vertical height.
2  2 sin2 45o  2
R ,hmax.  
Solution : (a) g 2g 4g
Clearly, R = 4 hmax.

8. A projectile thrown with a velocity  at an angle  has a range R on the surface of earth. For
same  and , its range on the surface of moon will be.
(A) 36 R (B) 60o
R R
(C) 6 (D) 36 .
1
R .g
Solution : (d) g is reduced by a factor of 6 on the moon.

9. If air resistance is ignored, then the horizontal motion of the oblique projectile takes place at
(A) uniform acceleration (B) variable acceleration
(C) uniform retardation (D) uniform velocity.
Solution : (d) The only force acting on a projectile is the force of gravity which acts vertically downwards.
It has no horizontal component. So, horizontal motion of the oblique projectile takes place at
uniform velocity.

10. A ball is projected with kinetic energy K at an angle of 45 o to the horizontal. At the highest point
during its flight, its kinetic energy will be
K
(A) K (B) 2
Join Elite Batch – Physics ( Batch -2 ) - Whatsapp -9264953797

Click Here - https://imjo.in/JKBjbX ( Batch Starting 01 March )


Final Revision Questions
2020
– Elite Batch

K
(C) 2 (D) zero.
K
Solution : (c) Kinetic energy at highest point = K cos 45 = 2 2 o

11. Two anti-aircraft guns fire two shells, one at an angle of 20 o with the horizontal. If muzzle
velocity is same in both the cases, then
(A) the first shell has a smaller horizontal range
(B) the second shell has a smaller horizontal range
(C) both the shells have to same vertical range
(D) both the shells have the same horizontal range.
Solution : (d) The horizontal range is the same whether the projectile is thrown at an angle  with the
horizontal or at an angle (90–) with the horizontal.

12. An aeroplane flying horizontally with a speed of 360 km h–1 releases a bomb at a height of 490
m from the ground. When will the bomb strike the bomb strike the ground ?
(A) 8s (B) 6s
(C) 7s (D) 10s.
2  490 2  49  100
t s s  100 s  10s
Solution : (d) 9.8 98

13. It is possible to project a body with a given speed in two possible ways so that
it has the same horizontal range R. The product of the times taken by it in the
two possible ways is (g is the acceleration due to gravity)
R 2R
(A) g (B) g
3R 4R
(C) g (D) g

Solution: If a body is projected with a given velocity u at angles  and (90–) to the horizontal,
it will have the same range R given by
u 2 sin2
R
g
The corresponding times of flight are
2u sin 2u sin  90    2u cos 
t1  t2  
g and g g

2u 2  2 sin cos   2u 2 sin2


 t1t2  
g2 g2

Join Elite Batch – Physics ( Batch -2 ) - Whatsapp -9264953797

Click Here - https://imjo.in/JKBjbX ( Batch Starting 01 March )


Final Revision Questions
2020
– Elite Batch

2R

g
Hence the correct choice is (b)

14. The resultant of velocity of two cars moving with equal speeds, Ratio of
R2
= 3
magnitude R 1 , when the direction of either car is reversed. The angle
between the direction of cars is
(A) 30° (B) 60°
(C) 120° (D) None of these

Solution: Let the vectors be


A⃗ & { ⃗B ¿
| A⃗ | = | B ⃗ | (given)
R2
=√ 3( given )
R1
R2 √ A 2+ B2 - 2 AB Cos θ = √3
 R1 = √ A2 + B2+ 2 AB Cos θ
Putting A = B we obtain  = 120°.

15. The locus of a projectile relative to another projectile is a


(A) straight line (B) circle
(C) ellipse (D) parabola

Solution: Let these projectiles be projected with velocities


⃗v 1=v x ^i + y y ^j=(v 1 cos θ1 ) ^i+(v 1 sin θ1 ) ^j
1 1
and
⃗v 2=v x ^i + y y ^j=(v 2 cos θ2 ) ^i+(v 2 sinθ 2 ) ^j
2 2

The velocities of the projectiles after a time t are given as


'
⃗v 1=(v 1 cosθ 1 ) ^i+(v 1 sin θ1 −gt ) ^j
'
⃗v 2=(v 2 cosθ 2 ) ^i+(v 2 sin θ 2−gt ) ^j
Relative velocity
' '
⃗v 1−⃗v 2 =( v 1 cosθ1 −v 2 cosθ2 ) ^i +( v 1 sinθ 1−v 2 sin θ2 ) ^j
Join Elite Batch – Physics ( Batch -2 ) - Whatsapp -9264953797

Click Here - https://imjo.in/JKBjbX ( Batch Starting 01 March )


Final Revision Questions
2020
– Elite Batch

' '
Since the relative velocity ⃗v 1−⃗v 2 is a constant and so does not vary with time, the
locus of one w.r.t. the other is a straight line.

ˆ ˆ ˆ
16. Given : 0.3i + 0.4j + nk is a unit vector. The value of n is

(A) 0.25 (B) 0.50

(C) 0.75 (D) 0.80

Solution: (C) (0.3)2 + (0.4)2 + n2 = 1 or 0.09 + 0.16 + n2 = 1

or n2 = 1 - 0.25 = 0.75 or n = 0.75 .

17. Two particles A and B move with constant velocities v 1 and v2 along two
mutually perpendicular straight lines towards the intersection point O. At
moment t = 0, the particles were located at distances l 1 and l2 from O,
respectively. The shortest distance between A & B is
|l 1 v 2 −l 2 v 1| |l 1 v 2 −l 2 v 1|

(A) √ v 21 + v 22 (B) √ v 21−v 22


|l 1 v 2 +l 2 v 1| ℓ1 v 2 v 1

(C) √ v 21+ v 22 (D) √ v 21 + v22


Solution: After time t, the position of the point A and B are (l1 - v1t) and (l2 - v2t), respectively.
The distance L between the points A and B are,
L2 = (l1 - v1t)2 + (l2 - v2t)2
Differentiating with respect to time,
l1
dL
2 L =2 ( l1−v 1 t ) (−v 1 )+2 ( l 2 −v 2 t ) (−v 2 ) A O
dt v1 A
dL
=0
For minimum value of L, dt L
2 2 l2
( v 1 + v 2 ) t =l 1 v1 +l 2 v 2
B
l 1 v 1 +l 2 v 2 v2
t=
or v 21 + v 22 B

Putting the value of t in equation


|l 1 v 2 −l 2 v 1|

Lmin = √ v 21 + v 22
18. A man walks 4 m towards East and then 3 m towards North and there he fixes a
pole 12 m high. The distance between the starting point and tip of the pole in space
is
Join Elite Batch – Physics ( Batch -2 ) - Whatsapp -9264953797

Click Here - https://imjo.in/JKBjbX ( Batch Starting 01 March )


Final Revision Questions
2020
– Elite Batch

(A) 7m (B) 11 m
(C) 13 m (D) 19 m
 ˆ ˆ ˆ
Solution: (C) s = 4i + 3 j + 12k

| s |= s = 42 + 32 + 122 m = 13 m

19. A particle is moving along a circular path of radius 5m and with uniform speed 5
m/s. What will be the average acceleration when the particle completes half
revolution?
(A) zero (B) 10 m/s2
2
(C) 10 nm/s (D) 10/nm/s2

Solution: The change in velocity when the particle completes half revolution is given by v = 5
m/s - (-5 m/s) = 1 0 m/s Time taken to complete half revolution
πr π ×5
t= = =π
v 5
v 10  m/s  10
     m/s
2

 Average acceleration t   s  
(D) is the correct choice.

20. A particle is projected horizontally from the top of a cliff of height H with a

speed 2gH . The radius of curvature of the trajectory at the instant of



projection will be

(A) H/2 (B) H


(C) 2H (D) 

Solution: (C) Since, ⃗g ⊥ {⃗v ¿


Radial acceleration ar = g
v 20
=g
 r where r is the radius of curvature.
2 gH
=g (∵ v =√ 2gH)
 r
 r = 2H

21. Which of the following graph correctly represents velocity-time relationship


for a particle released from rest to fall freely under gravity?

Join Elite Batch – Physics ( Batch -2 ) - Whatsapp -9264953797

Click Here - https://imjo.in/JKBjbX ( Batch Starting 01 March )


Final Revision Questions
2020
– Elite Batch

v v

(A) (B)
t t

v v

(C) (D)

t t

Solution: Releasing of the particle from rest means that v0 =0 at t = 0 and v =gt
at any time t.
 the slope of v/t graph is a constant.
 v/t graph is a straight line passing through the origin.
 (A)

22. A body is moving in a circle at a uniform speed  . What is the magnitude of the change in
velocity when the radius vector describes an angle 
 
2 cos  
(A)  cos  (B) 2

 
2 sin  
(C)  sin (D) 2
 
vf  vi  v 02  v 20  2v 20 cos 
Solution: (D)
 v 02 2(1  cos )
 v 02 2  2 sin2   / 2   2v 0 sin(  / 2)

23. What can be the possible velocity displacement (v – s) graph of a particle moving in a
straight line under constant acceleration
(A) straight line (B) parabola
(C) ellipse (D) circle
Solution: (B) v2  4as  parabola

Join Elite Batch – Physics ( Batch -2 ) - Whatsapp -9264953797

Click Here - https://imjo.in/JKBjbX ( Batch Starting 01 March )


Final Revision Questions
2020
– Elite Batch

24. A particle is moving eastward with a speed of 5 m/s. After 10 seconds, the direction changes
towards north, but speed remains same. The average acceleration in this time is
1
(A) zero (B) √2 m/s2 towards N-W
1 1
(C) √2 m/s towards N-E
2
(D) √2 m/s2 towards S-W
 
Solution 6:
v  5i,
(B) i  f
v  5j
v i  v f  5j  5i
 
v f  vi
a 
t (j - i) / 2 so direction is W  N
 1
a
2

25. If a particle takes t second less and acquires a velocity of v ms-2 more in falling through the
same distance on two planets where the accelerations due to gravity are 2g and 8g
respectively then
(A) v = 4gt (B) v = 5gt
(C) v = 2gt (D) v = l6gt

Solution: (A) V1 
4gH V '  16gH
H H
t t
g 4g
H H 1 H
 t t
g 4g  2 g

H  4t g
2

v ' v1  v  v  2 4gH  4gH


v  2 gH  2 4t 2g2
v  4gt m/s

26. An aeroplane flies along a horizontal circle of circumference 10 km, at a constant speed of
100 km/hr. The change in velocity in one fourth of a revolution is
(A) zero
(B) 141 km/hr at 90 from the original direction
(C) 141 km/hr at 135 from the original direction.
(D) 200 km/hr at 180 from the original direction.

Join Elite Batch – Physics ( Batch -2 ) - Whatsapp -9264953797

Click Here - https://imjo.in/JKBjbX ( Batch Starting 01 March )


Final Revision Questions
2020
– Elite Batch

 200 200


2v cos    2  100  cos 45    1.41kmh
Solution: (C)  2  2 1.41
135 to horizontal direction
vf Vi 135
vf
vi

27. Ratio or minimum kinetic energies of two projectiles of same mass is 4 : 1. The ratio of the
maximum height attained by them is also 4 : 1. The ratio of their ranges would be
(A) 16 : 1 (B) 4:1
(C) 8:1 (D) 2:1

Solution: (A) minimum K.E. = 1/2m


u12
u2 u12 u u
2
4  1 2 2 4
v1 v1 v2
2u 2u u
Range  u1  2  1 2
u1 g g
R1 u1u2
  8 :1
R2 v1v 2

28. A particle of mass m attached to a string of length I is describing circular motion on a sooth
plane inclined at an angle  with the horizontal. For the particle to reach the highest point its
velocity at the lowest point should exceed.

(A)
5gl

(B)
5gl(cos  +1)

(C)
5gltan 

(D)
5glsin 
Solution: (A) g effective  g sin
5geff l  5gl
V min 

29. A particle is projected from the ground with an initial speed of v at an angle  with horizontal.
The average velocity of the particle between its point of projection and highest point of
trajectory is
v
1 2cos2 
(A) 2

Join Elite Batch – Physics ( Batch -2 ) - Whatsapp -9264953797

Click Here - https://imjo.in/JKBjbX ( Batch Starting 01 March )


Final Revision Questions
2020
– Elite Batch

v
1 cos2 
(B) 2
v
1 3 cos2 
(C) 2
(D) v cos 
Solution: (C)
v 2 sin2 
v H
H 2g

R R v 2 sin2
2 
2 2g
R2
H2 
displacement = 4
Displacment
vavg = t
v
 1  3 cos2 
2

30. A particle is projected with a certain velocity at an angle  above the horizontal from the foot
of an inclined plane of inclination 30. If the particle strikes the plane normally then  is equal
to
 1 
30 + tan-1  
(A) 2 3
(B) 45
(C) 60

(D)
30 + tan-1 2 3  
Solution: (D) v = 0 = ucos( - 30) – gsin(30)t (along x-axis)
u cos(  30)
t
g sin30 …..(1)
0) along Y-axis

3 g sin 30 g cos 30
( 
30 1 2
S  ut  at  0
2
2u 2u sin(  30)
t 
a g sin30 …(2)
by (1) and (2)
u cos(  30) 1 3
 cot 30 
g sin30 2 2

Join Elite Batch – Physics ( Batch -2 ) - Whatsapp -9264953797

Click Here - https://imjo.in/JKBjbX ( Batch Starting 01 March )


Final Revision Questions
2020
– Elite Batch

 3
  30  tan1  
 2 

31. With what minimum speed must a particle be projected from origin so that it is able to pass
through a given point (30m, 40m). Take g  10 m/s2
(A) 60 m/s (B) 30 m/s
(C) 50 m/s (D) None of these
gx 2
y  x tan  2
Solution: (D) 2u cos2 
(x, y) = (30, 40)
g (sec 2  )  500
40 = 30 tan - 2u 2
90g sec 2 
 3 tan  4
2u 2
45 g sec 2  45g
u 2

3  sin u 3 sin cos   4cos2 
3sin2  cos2  1
f ( )  3 sin cos  4cos2    
2  2 
1

2
 3 sin 2  2cos 2   1
1
f '(0)   6 cos 2  4 sin 2   0
2
tan(2) = - 6/ 4 = -3/2

sin(2) = 3 / 13

cos(2) = -2 / 13
45g  2
u
13  2

32. A ball is dropped vertically from a height h above the ground. It hits the ground and
bounces up vertically to a height h/2. Neglecting subsequent motion and air resistance,
its velocity  varies with the height h as

Join Elite Batch – Physics ( Batch -2 ) - Whatsapp -9264953797

Click Here - https://imjo.in/JKBjbX ( Batch Starting 01 March )


Final Revision Questions
2020
– Elite Batch

(A) (B)

 
h h

(C) (D)

 
h h

Solution: (A) Velocity in downward motion is negative and upward motion will be positive. So either
any will be A or C but v2  4 as the graph will be a parabola.

33. A ball is projected at an angle of 45, so as to cross a wall at a distance from the point of
projection. It falls at a distance b on the other side of the wall. If h is the height of the wall
then

(A) ha 2 (B) hb 2

2ab ab
h h
(C) a+b (D) a+b
Solution: (D)
tan 45  tan1  tan 
1
h 2
a b h h ab
 h
1 a b ab

34. At what angle the forces of 2 N and 2 N act so that their combined effect is that of a single

force of 10 N ?
(A) 0 (B) 30
(C) 45 (D) 60
1
cos q =
Solution: (C) 10 = 4 + 2 + 4 2 cos q or 2 or  = 45

Join Elite Batch – Physics ( Batch -2 ) - Whatsapp -9264953797

Click Here - https://imjo.in/JKBjbX ( Batch Starting 01 March )


Final Revision Questions
2020
– Elite Batch

35. Two bodies fall freely from the same height, but the second body starts falling T seconds
after the first. The time (after which the first body begins to fall) when the distance between
the bodies equals L is
1 1
(A) gh (B) 2gh
1
(C) gt (D) None of these
Solution: (D)
After time T velocity of I’st ball = gT
Relative velocity between ball 1 and ball2 = Gt
1 L  gT 2
gT  gTt
2

L= 2  t = gT
Total time = T + t

36. Two projectiles, one fired from the surface of the earth with speed 5 m/s and the other fired
from the surface of a planet with initial speed 3 m/s, trace identical trajectories. Neglecting
air resistance, the value of acceleration due to gravity on the planet will be if g = 10 m/s 2 on
earth
(A) 5.9m/s2 (B) 3.6 ms/2
2
(C) 16.3 m/s (D) 8.5 m/s2
gx 2
y  x tan  2
Solution: (B) 2u cos2 
Trajectory is identical so
g1 g
2
 22
2u1 2u2
10 g 2
 
25 9
18
g2  m / s 2  3.6m / s 2
5

37. A particle is projected under gravity with


v 2a g

velocity
2ag from a point at a height h above
the level plane. The maximum range R on the O θ
ground is

Join Elite Batch – Physics ( Batch -2 ) - Whatsapp -9264953797 P

R
Click Here - https://imjo.in/JKBjbX ( Batch Starting 01 March )
Final Revision Questions
2020
– Elite Batch

(A) (a2  1)h

(B) a2 h

(C)
ah

(D)
2 a(a  h)
Solution: (D) Co–ordinates of point P are (R, –h). These coordinates should satisfy the
gR 2
h  R tan   (1 tan2 )
equation of projectile i.e. 2(2ag)
or R2 tan2 – 4aRtan  (R2 – 4ah)  0
For  to be real (4aR) 2  4R2(R2 – 4ah)
or 4a2  4a (a  h)

or R2  2
a(a  h)

 The maximum range is R max  2


a(a  h)

38. A projectile has the same range R for two angles of projections. If T1 and T2 be the times of
flight in the two cases, then

(A) T1T2  R (B) T1T2  R2


T T1
 tan   cot 
1

T T2
(C) 2
(D)
Solution: (C)
2usin 
T1 
g
2usin(90  ) 2ucos2  2a cos 2 
T2   
g g g

39. If T is the total time of flight, H the maximum height and R is the horizontal range of a
projectile. Then x and y co-ordinates at any time t are related as
 t  t   T  T 
y  4H   1  y  4H   1 
(A)  T  T  (B)  t  t 
 x  x   R  R 
y  4H   1  y  4H   1 
(C)  R  R  (D)  x  x 
sin2 gx 2
R  u2 and y  x tan   2
Solution: (C) g 2u cos2 

Join Elite Batch – Physics ( Batch -2 ) - Whatsapp -9264953797

Click Here - https://imjo.in/JKBjbX ( Batch Starting 01 March )


Final Revision Questions
2020
– Elite Batch

2usin  4H
T  tan 
g R
u2 sin2 
H
2g
4H 2u2 sin2   g2

R 2 4g  u4 sin2  cos2 
4H 4H
y   2 x2
By (C) R R

40. The horizontal range and maximum height attained by a projectile are R and H respectively.
If a constant horizontal acceleration a  g/4 is imparted to the projectile due to wind, then its
horizontal range and maximum height will be
H  H
(R +H),  R + 2  ,2H
(A) 2 (B)  
(C) (R + 2H), H (D) (R + H), H
Solution: (D) Maximum height will not effected.
So Range  ucos  T  1/ 2(g/4)T2
 R  gT2/ 8  R  g / 8  4u2sin2 / g2 = R + H

ˆ ˆ 
41. Given : = 3i - 4 j . Which of the following is perpendicular to
P P?

(A) 3iˆ (B) 4jˆ

(C) 4jˆ + 3iˆ 4ˆj - 3iˆ


(D)

Solution: (C) P is in fourth quadrant.
4iˆ + 3ˆj is in the first quadrant.

Clearly, 4iˆ + 3ˆj 


can be perpendicular to P . For confirmation, let us check whether
their dot product is zero.
( 3ˆj - 4iˆ) ×( 4iˆ + 3ˆj) = 12 - 12 = 0
ˆ ˆ
This shows that 4i + 3 j is perpendicular to
3ˆj - 4iˆ
.
( )
ˆ ˆ ˆ ˆ
42. The angle between i + j and j + k is
(A) 0 (B) 90
(C) 45 (D) 60
 
A ×B A xB x + A yB y + A zB z
cos q = =
AB A x 2 + A y 2 + A z 2 B x 2 + B y 2 + Bz 2
Solution: (D)
Join Elite Batch – Physics ( Batch -2 ) - Whatsapp -9264953797

Click Here - https://imjo.in/JKBjbX ( Batch Starting 01 March )


Final Revision Questions
2020
– Elite Batch

(1)(0) + (1)(1) + (0)(1) 1


= = Þ q = 60°
2
1 +1 1 +1 2 2 2 2

43. In projectile motion maximum horizontal range get at which angle ?


(A) 0 (B) 90
(C) 45 (D) 60

Solution: (C)

44.
^ ^
The velocity vectors of a point A varies with time as ⃗v =a i+bt { j¿ , where a and b
are positive constants. Find the equation of trajectory of the point.
2 2
1 x 1 x
y= b y= a
(A) 2 a (B) 2 b
1 x 1 x
y= a y= b
(C) 2 b (D) 2 a
d⃗r
⃗v = =a ^i +bt { ^j ¿
Solution: (A) dt
dx dy
∴ =a and =bt
dt dt
1
∴ x=at and y= bt 2
2
Eliminating t from these to equations, we get
1 x2
y= b
2 a

45.
^ ^
The velocity vectors of a point A varies with time as ⃗v =a i+bt { j¿ , where a and b
are positive constants. Find the time dependence of the angle 'α' between
acceleration and velocity vectors.
t bt
(A) √a 2+b 2 t2 (B) √a 2+b 2 t2
bt b
(C) √a +b 2 t
2
 (D) √a +b 2 t2
2

Solution: (B) acceleration


A  b ĵ
  
 
 A  v  bjˆ . aiˆ  btjˆ

⇒ b √ a2 +b 2 t 2 cosα=b2 t

Join Elite Batch – Physics ( Batch -2 ) - Whatsapp -9264953797

Click Here - https://imjo.in/JKBjbX ( Batch Starting 01 March )


Final Revision Questions
2020
– Elite Batch

bt
∴ cosα=
√ a +b 2 t 2
2

46.
^ ^
The velocity vectors of a point A varies with time as ⃗v =a i+bt { j¿ , where a and b
are positive constants. Find
the time dependence of magnitude of acceleration and velocity vectors.

(A) a2 + b2 t 2 (B) √ a+b 2 t2


(C) √ a2+b2 t 2 (D) √ a2+bt
Solution: (C) |⃗A|=b ∴ acceleration is independent of time
|⃗v|= √a 2 +b 2 t2
47. A point moves along a circle with a velocity v = at where a = 0.50 m/s 2. Find
the total acceleration of the point at the moment when it has covered the n th
(n = 0.10) fraction of the circle after beginning of the motion.

Solution: Let v be the velocity at the instant when the particle has covered the n th fraction of the
circle

dv
a
Tangential acceleration = dt
v 2  2as
v  2an  2 r
v2
 4 an
Radial acceleration = aN= r
a 2  16 2a 2n 2  a 1  16 2 n 2
Total acceleration =
 0.8m / s 2

48. A projectile is fired from the surface of level ground at an angle 0 above the
horizontal. Then elevation angle tan is
1
tan0 tan0
(A) (B) 4
1 2
tan0 tan0
(C) 2 (D) 1
H 2H
tan  
R R
Solution: (C) 2
u

Join Elite Batch – Physics ( Batch -2 ) - Whatsapp -9264953797 H

0 
O

Click Here - https://imjo.in/JKBjbX ( Batch Starting 01 March )


Final Revision Questions
2020
– Elite Batch

H = Maximum height
u 2sin 20

2g
R = Maximum range
u 2sin20

g
sin 20
tan 
sin20
sin0 1
tan   tan0
2cos0 2

49. A projectile is fired from the surface of level ground at an angle 0 above the
horizontal. Then calculate  for 0 = 45.
1 1
sin1 cos1
(A) 2 (B) 2
1 1
tan1 tan1
(C) 3 (D) 2

Solution: (D) 0 = 45


1
tan   tan 45
2
1

2
1
   tan1
2

A
50. A man is running with constant speed
along a circular path of radius 22m. He C

completes 1 round in 10 second. Find


B
average velocity in first 2.5 sec.

(A) 1. 6 m/ s (B) 2. 6 m/ s
(C) 1. 0 m/s (D) 3 .0 m/ s

Join Elite Batch – Physics ( Batch -2 ) - Whatsapp -9264953797

Click Here - https://imjo.in/JKBjbX ( Batch Starting 01 March )


Final Revision Questions
2020
– Elite Batch

A
51. A man is running with constant speed
along a circular path of radius 22m. He C

completes 1 round in 10 second. Find


B
instantaneous speed at 2.5 sec.

√2 π 2 √2 π
(A) 5 m/s (B) 5 m/s
2 √3 π 5 √2 π
(C) 5 m/s (D) 2 m/s
A
52. A man is running with constant speed
along a circular path of radius 22m. He C

completes 1 round in 10 second. Find


B
average velocity in 20 seconds

(A) 2 (B) 5
(C) 0 (D) 3
Solutions :(50-52)
50(a) Let man starts from A, distance covered in first 2 seconds = Path ACB

Displacement = AB= √ 8+8 =4m


r 2−r 1 4
= =1. 6 m/ s
Average velocity = t 2−t 1 2. 5

2π 2π 2√ 2 π
r 2 √ 2=
51(b) |⃗v| = r = T = 10 5 m/s
52(c) Net displacement = zero, so average velocity = 0

53. A rocket is fired vertically and ascends with constant vertical acceleration of 20m/s 2
for 1 minute. Its fuel is then all burnt and it continues to move as a free body.
Find the maximum height reached by the rocket
(A) 108000m (B) 106000m
(C) 108033m (D) 98000m

54. A rocket is fired vertically and ascends with constant vertical acceleration of 20m/s 2
for 1 minute. Its fuel is then all burnt and it continues to move as a free body.
Find the total time elapsed from the take off till the rocket strikes the earth.
(g=10m/s2).
(A) 526.76 s (B) 326.96 s
(C) 400.96 s (D) 226.96 s

Join Elite Batch – Physics ( Batch -2 ) - Whatsapp -9264953797

Click Here - https://imjo.in/JKBjbX ( Batch Starting 01 March )


Final Revision Questions
2020
– Elite Batch

Solution : 53(a)For the time interval from 0 to 60 seconds rocket accelerates and after then, it moves
freely under gravity. Distance moved by it in 60 seconds is given by
1 20 m
× 2 ×( 60 s )2
S1 = 2 s = 36000m.
20 m
×60 s=1200 m / s
v(60s) = s2
If H be the maximum height reached.
2
m
Then 0 =
( 1200
s)−2 g ( H −36000 )
, (v2 = u2 + 2as)
1200×1200
m
 H = 36000 + 2×10  H = 108000m
(b) time taken to ascend is
1200 u
s
t1 = 60s + 10 = 180 s, [t = t1 + g ]
Let time taken to descend is t2 then
1 2
gt 2
108000 = 2
2×108000
 t2 = √ 10
=146.96s
Total time T = t1 + t2 = 180 + 146.96 = 326.96 s

55. If the velocity of projection of a projectile is trebled, then its maximum range will be
(A) quadrupled (B) nine times
(C) six times (D) eight times.
(B) Max. range  .
2
Solution :

56. The times of flight of a projectile is maximum when angle of projection is


(A) 30o (B) 45o
o
(C) 60 (D) 90o.
2  sin 
T
Solution: g , For maximum T, sin should be maximum i.e.  should be 90 o.

57. If air resistance and air buoyancy are to be ignored, then which of the following factors
determines the time of flight of the projectile?
(A) intial velocity
(B) horizontal component of initial velocity
(C) vertical component of initial velocity
Join Elite Batch – Physics ( Batch -2 ) - Whatsapp -9264953797

Click Here - https://imjo.in/JKBjbX ( Batch Starting 01 March )


Final Revision Questions
2020
– Elite Batch

(D) vertical component of acceleration.


2  sin  2 
T =
Solution: (C) g g

58. A body is thrown with a velocity of 10 ms –1 at an angle of 60o with the horizontal. Its
velocity at the highest point is
(A) 7 ms–1 (B) 9 ms–1
–1
(C) 18.7 ms (D) 5 ms–1
o –1
Solution: (D)  cos  = 10 cos 60 = 5 ms

59. A ball is projected upwards. Its acceleration at the highest point is


(A) infinite (B) zero
(C) directed upwards (D) directed downwards.
Solution: (D) Acceleration due to gravity is always directed vertically downward.

60. A body is projected at an angle of 30o with the horizontal with momental with moment p.
At its highest point, the momentum is
p
(A) p (B) 2
3 2
p p
(C) 2 (D) 3 .
Solution: (C) p = m
3m 3p
=
P’ mcos 30o = 2 2

35. Two particles start simultaneously from the same point and move along two straight lines, one
with uniform velocity  and other with a uniform acceleration a. If  is the angle between the
lines of motion of two particles then the least value of relative velocity will be at time given by

 
sin  cos 
(A) a (B) a
 
tan  cot 
(C) a (D) a .

Sol. : (B)
Velocity of 1st particle at time t is v1 = v
Velocity of 2nd particle at time t is v2 = at
v r  v 22  2v1v 2 cos 
Relative velocity
 v r2  v 2  a2 t 2  2v(at)cos 
at
=
v2

Join Elite Batch – Physics ( Batch -2 ) - Whatsapp -9264953797


v1 = v

Click Here - https://imjo.in/JKBjbX ( Batch Starting 01 March )


Final Revision Questions
2020
– Elite Batch

For vr to be least or
vr2 to be least we must have
d 2
(v r )  0
dt
 0  2a2 t  2av cos   0
v cos 
 t
a

36. Two particles start simultaneously from the same point and move along two straight lines, one
with uniform velocity  and other with a uniform acceleration a. If  is the angle between the
lines of motion of two particles then, the least value of relative velocity is
(A)  sin  (B)  cos 
(C)  tan  (D)  cot .

Sol. : (A)
 v 2 cos2    v cos  
v r  v 2  a2  2   2v   acos 
 a   a 
 v r  v 2  v 2 cos2 
 v r  v sin 

39. A 2m wide car is moving with a uniform speed of 8 ms–1 along the edge of a straight horizontal
road. A pedestrian starts to cross the road with a speed  when the car is 12 m away from him.
The time to cross the moving vehicle safely is
24 37
s s
(A) 37 (B) 24
24 3
s s
(C) 49 (D) 2 .

Sol. : (B)
For pedestrian to cross the road safely, the time taken by the pedestrian to reach P equals the
time taken by the edge A of the car to reach P. To reach P total distance (12 + 2 cos ) is
travelled by the car with a velocity of 8 ms–1. Hence
12  2cot 
t
8
 1  1
12  2    12  3 
t 6   
8 8
37
t s
24

Join Elite Batch – Physics ( Batch -2 ) - Whatsapp -9264953797

Click Here - https://imjo.in/JKBjbX ( Batch Starting 01 March )


Final Revision Questions
2020
– Elite Batch

40. The velocity of a boat in still water is  times less than the velocity of flow of the river ( > 1).
The angle with the stream direction at which the boat must move to minimise drifting is
 1  1
sin1   cot 1  
(A)   (B) 
  1   1
 sin1    cot 1  
(C) 2   (D) 2   .

Sol. : (C)
Drift

Vr = v
vb = velocity of boat = v (say)
vr = velocity of river = v
To start with, we must observe that velocity of
Vb = v river flow is  times greater than the velocity of

x boat. So, the boat has to drift and in this problem we are to
minimise the drift. Let, the velocity of the boat make an angle  with the river velocity. Then v x =
vcos  is the component of the velocity of boat along the river flow, whereas v y = v sin q is the
component of the velocity of boat perpendicular to river flow. Due to this the v r + vx = v + v cos
 = V (say)
L L

v v sin 
t= y
During this time the drift equals (say x)
x = Vt
L
x  (v  v cos )
x= v sin 
x  ( cos ec   cot )L
To MINIMISE x, we must have
dx
0
d
(  cos ec cot )  cos ec 2  0
  cos ec cot )  cos ec 2  0
  cot    cos ec 2
cos  1
  
sin  sin 
1
 cos   

1
  cos  

  1
 sin     
 2 

Join Elite Batch – Physics ( Batch -2 ) - Whatsapp -9264953797

Click Here - https://imjo.in/JKBjbX ( Batch Starting 01 March )


Final Revision Questions
2020
– Elite Batch

  1
   sin1  
2 
  1
   sin1  
2 

41. An armoured car 2 m long and 3 m wide is moving at 10 ms –1 when a bullet hits it in a direction
3
tan1  
making an angle  4  with the length of the car as seen by a stationary observer. The bullet
enters one edge of the car at the corner and passes out at the diagonally opposite corner.
Neglecting any interaction between the car and the bullet, the time for the bullet to cross the car
is
(A) 0.20 s (B) 0.15 s
(C) 0.10 s (D) 0.50 s.

Sol. : (A)

44. A boar which has a speed of 5 kmh–1 in still water crosses a river of width 1 km along the
shortest possible path in 15 minute. The velocity of the river water in kmh –1 is
(A)  (B) 3
(C) 4 (D) 41 .

Sol. : (B)

45. A car A is going north east at 80 kmh–1 and another car B is going south east with a velocity of
60 kmh–1. The velocity of A relative to B makes an angle with the north equal to
2 7
tan1   tan1  
(A) 7 (B) 2
(C) tan1(7) (D)  cot .

Sol. : (D)
 1 ˆ 1 ˆ 
tan1  i j   vA
80 kmh–1 NE  2 2 
 1 ˆ 1 ˆ 
60kmh1SE  60  i j   vB
 2 2 
  
v A / B  velocity of A relative toB  v A  v B
Since

Join Elite Batch – Physics ( Batch -2 ) - Whatsapp -9264953797

Click Here - https://imjo.in/JKBjbX ( Batch Starting 01 March )


Final Revision Questions
2020
– Elite Batch

  
 v A / B  v A  vB  10 2iˆ  70 2jˆ
 1
   tan1   .
7

OBJECTIVE

1: A block of mass 10 kg accelerates uniformly from rest to a speed of 2 m/s in 20


sec. The average power developed in time interval of 0 to 20 sec is
(A) 10W (B) 1W
(C) 20W (D) 2W
Solution: (B)Average power
Net work done
Pav = Total time taken
Net work done = change in kinetic energy = Final energy – initial energy
1
×10 ×22
= 2 = 20J . . . (1)
20
Average power = 20 = 1 watt

2: An object is attached to a vertical spring and is allowed to fall under the gravity. What is the
distance traversed by the object before being stopped?
(A) mg/k (B) 2mg/k
(C) mg/2k (D) none of these.

1
Solution: (B)Apply COE; mgx= 2 kx2  x = 2mg/k

3: When a man walks on a horizontal surface with constant velocity work done by
the
(A) frictional force is zero (B) contact force is zero
(C) gravity is zero (D) man is zero

Join Elite Batch – Physics ( Batch -2 ) - Whatsapp -9264953797

Click Here - https://imjo.in/JKBjbX ( Batch Starting 01 March )


Final Revision Questions
2020
– Elite Batch

Solution : Since mg & N are perpendicular to velocity ⃗v N

and d ⃗s , work done by these forces is zero. v


Since no relative sliding occurs during walking, f
static friction comes into play. Hence the point ds
of application of static frictional force does not
mg
move. Therefore no work is done by frictional
force.
Man has to lose his body's (internal) energy E, hence performs work because W=
E (numerically). (A)

4: A pumping machine pumps water at a rate of 60 cc per minute at a pressure of


1.5 atm. The power delivered by the machine is
(A) 9 watt (B) 6 watt
(C ) 9 kW (D) None of these
Solution : (D)Power = F.v Where F = force imparted by the machine , v = velocity of the liquid
P = p.A.v, Where p = pressure & A = effective area
3 60
dV
= p dt = 2 (
×105
60 )(
×10−6 ) ( ∵ 1 atm  105 N/m2 )
= 0.15 w  none of these
5: A block of mass m moving on a smooth horizontal floor, with a constant
velocity v0 collides with a light spring of stiffness constant k which is rigidly
fixed horizontally with a vertical wall. If the maximum force imparted by the
spring on the block is F, then:
(A) F  √ m (B) F √ k
(C) F  v0 (D) all of these

Solution : (C)Energy conservation between the positions v0


k
A and B
m
1 1 2
PE+ KE = 0  2 kx  2 m v 0 =0
2

m B

 x= √ v
k 0 m

 Fmax = kx = √ km v 0 x

 Fmax  k
m
v0.

6: A block of mass m is moving with a constant acceleration a on a frictional plane. If the


coefficient of friction between the block and ground is , the power delivered by the
external agent after a time t from the beginning is equal to:

Join Elite Batch – Physics ( Batch -2 ) - Whatsapp -9264953797

Click Here - https://imjo.in/JKBjbX ( Batch Starting 01 March )


Final Revision Questions
2020
– Elite Batch

(A) ma2t (B) mgat


(C) m(a+g)gt (D) m(a+g)at
Solution : (D)Instantaneous power delivered = P = v a
⃗F .⃗v = Fv m F
f
where, F – f = ma
 F = f + ma
 P = (f + ma) v
Put f = mg
 P = (mg + ma)v = m(a + g).at
7: Find the ratio of work done by
gravity on the block under the 
/2
two conditions. Assume  = 0 h
h
between block and plane.
(A) 1 : 2  
(B) 2 : 1 Condition (1) Condition (2)
(C) 1 : 1
(D)  : 
Solution : (C)Work done by gravity will depend only upon the height ‘h’ and not on the length of
inclined plane. So work done in both the vases = mgh only.
ratio = 1 : 1

8: A spring, placed horizontally on a rough surface is compressed by a block of


mass m, placed on the same surface so as to store maximum energy in the
spring. If the coefficient of friction between the block and the surface is , the
potential energy stored in the spring is
μ 2 m2 g 2 2 μm2 g 2
(A) 2k (B) k
2 2
μ m g 3 μ mg2
2

(C) 2k (D) k
`Solution : (A)For equilibrium of the block Fmax
m
Fmax - mg = 0
 F = mg
mg
F 2 μ2 m 2 g2
=
 U = 2k 2k

9: An object of mass m is tied to a string of length L and a variable horizontal


force is applied on it which starts with zero magnitude and gradually increases
until the string makes an angle  with the vertical. Work done by the variable
force F is:
(A) mgL(1-sin) (B) mgL
(C) mgL(1-cos) (D) mgL(1+cos)

Join Elite Batch – Physics ( Batch -2 ) - Whatsapp -9264953797

Click Here - https://imjo.in/JKBjbX ( Batch Starting 01 March )


Final Revision Questions
2020
– Elite Batch

Solution : Work done by the external force =  E of the system L cos   L


(object) m
= PE ( ∵ Δ KE=0 ) h B
m
= mgh = mg (L – L cos ) = mgL (1 – cos ). A
(C)

10: When a body of mass M slides down an inclined plane of inclination , having
coefficient of friction  through a distance s, the work done against friction is:
(A)  Mg cos  s (B)  Mg sin  s
(C) Mg ( cos   sin )s (D) None of the above

Solution : (A)Work done against friction


= - work done by friction = mg cos  s.

11: An object is pushed and pulled along a certain pull


angle w.r.t. horizontal and is moved the some push
distance on the ground. Now the work done by

frictional force will be 
(A) push effect. (B) pull effect
(C) remain same. (D) data
insufficient.

Solution : (D)fr = (mg  F sin) pulling. f sin

Fv = (mg + F sin) pushing. 


 f cos

12: A body is acted upon by a force which is inversely proportional to the distance
covered. The work done will be proportional to:
(A) s (B) s2
(C) s (D) None of the above
s

∫ ks ds
Solution : (D)W =  F.ds= s1 = kln (s/s1)
13: A particle at rest is constrained is to move on a smooth horizontal surface.
Another identical particle hits the stationary particle with a velocity v at an
angle
 = 600 with horizontal. If the particles move together, the velocity of the
combination just after the impact is equal to
(A) v (B) v/2

(C) √ 3v/4 (D) v/4

Join Elite Batch – Physics ( Batch -2 ) - Whatsapp -9264953797

Click Here - https://imjo.in/JKBjbX ( Batch Starting 01 March )


Final Revision Questions
2020
– Elite Batch

Solution : (D)mv cos  = (M + m) v m


v v 
v
 v = 2 cos 60 = 4 .
0
m

14: Find the horizontal velocity of the particle when it reach the point Q. Assume
the block to be frictionless. Take g = 9.8 m/s 2.
(A) 4 m/s (B) 5 m/s
(C) 3.13 m/s (D) 3.6 m/s
m=2kg

1m
Q
0.5m

5m

1 1 g v2
Solution : (C)mg
( )
1−
2 = 2 mv2 2 = 2  v= √g  3.13 m/s.

15. A neutron mass 1.67  10–27 kg moving with a velocity 1.2  107 ms–1 collides head-on with a
deuteron of mass 3.34  10–27 kg initially at rest. If the collision is perfectly inelastic, the
speed of the composite particle will be
(A) 2  106 ms–1 (B) 4  106 ms–1
(C) 6  106 ms–1 (D) 8  106 ms–1
Sol: (B) Mass of neutron (m) = 1.67  10 kg. Speed of neutron (v) = 1.2  107 ms–1. Notice that
–27

the mass of deuteron (M) = 3.34  10–27 kg = 2m. If V is the speed of the composite particle,
the law of conservation of momentum gives
mv = (m + M)V
mv mv
V= =
or m + M m + 2m
v 1.2 ´ 107
V= = = 4 ´ 106 ms- 1
or 3 3 .

16. In above. Q. if the collision were perfectly elastic, what would be the speed of deuteron after
the collision ?
(A) 2  106 ms–1 (B) 4  106 ms–1
(C) 6  10 ms
6 –1
(D) 8  106 ms–1
Sol: (D) In an elastic collision, both momentum and energy are conserved. Using the two laws, it
is easy to see that (M = 2m), the deuteron will move forward with a speed
2 ´ 1.2 ´ 107
2v / 3 = = 0.8 ´ 107 ms - 1
3 .
Join Elite Batch – Physics ( Batch -2 ) - Whatsapp -9264953797

Click Here - https://imjo.in/JKBjbX ( Batch Starting 01 March )


Final Revision Questions
2020
– Elite Batch

17. A body of mass 5kg initially at rest, is subjected to a force of 40N. Find the kinetic energy
acquired by the body at the end of 5 seconds

(A) 4000 J (B) 6000 J


(C) 6249 J (D) 6145 N
Solution: (A)  Here we will calculate acceleration
F 40
a = m = 5 = 8 m/s2
Velocity at the end of 5 seconds is
v = u + at
v = 0 + 85 = 40 m/s
1 2 1
mv
 K.E. acquired = 2 = 2  5 (40)2 = 4000J

18. A body is constrained to move along z-axis of the co-ordinate system is being
applied by a constant force ⃗F = ( 2 ^i + 3 ^j + 4 k^ ) . Find the work done by this force in
moving the body over a distance of 5m along z-axis.
(A) 30 J (B) 20 J
(C) 40 J (D) 61 J
Solution: (B) Here displacement is only along z-axis

Hence, ⃗s = 5 k^ and
⃗F = ( 2 ^i + 3 ^j + 4 k^ )
 Work done by force W =
⃗F .⃗s
= ( 2 ^i + 3 ^j + 4 k^ ) . 5 k^ = 20J

19. A body moves from point A to point B F(N)


under the action of a force varying in 10 B

magnitude as shown in the force


5
displacement graph. Find total work
done by the force A 5 7 S(m)

(A) 50 J (B) 60 J
(C) 40 J (D) 45 J

Solution: (D) Work done dw =


⃗F .d⃗s

 W= ∫ ⃗F .d ⃗s
1
= Area under F-S curve = 2  5 10 + 210 = 45J

Join Elite Batch – Physics ( Batch -2 ) - Whatsapp -9264953797

Click Here - https://imjo.in/JKBjbX ( Batch Starting 01 March )


Final Revision Questions
2020
– Elite Batch

20. A body dropped forma height ‘H’ reaches the ground with a speed of 1.1 √ gH .
Calculate the work done by air friction?
(A) 0.395 mgH (B) 0.395 mgH
(C) 0.400 mgH (D) 0.400 mgH
Solution : (D) K.E. of particle on reaching the ground O
1 mg f = air friction

= 2 m.(1.21)gh = 0.605mgH
air Now since body moves opposite to the
Now KE = mgH  f r direction of force of air friction.
air
0.605 mgH = mgH  f r
 (air friction) = 0.395 mgH
air
 fr = 0.395 mgH
2kg
21. A block of mass m = 2kg is 100 N/m 125 N/m

attached to two unscratched


springs of force constant k1 = 100 0.1m
N/m and k2 = 125 N/m.
The block is displaced towards left through a distance of 10 cm and released.
Find the speed of the block as it passes through the mean position.

(A) 1.06 m/s (B) 1.02 m/s


(C) 1.04 m/s (D) 1.05 m/s
Solution: (A) Here effective spring constant of the combination = k 1 + k2 = 225 N/m
Now applying the energy conservation principle, we have
1 1
2 [100+125]  (0.1)2 = 2 mv2
1 1
2 [100+125]  (0.1)2 = 2  2  v2
 v2 = 1.125  v = 1.06 m/s at the mean position.

22. A 16kg block moving on a frictionless horizontal surface with a velocity of 4m/s
compresses an ideal spring and comes to rest. If the force constant of the
spring is 100N/m, then how much is the spring compressed?
(A) 1. 7 m (B) 1. 6 m
(C) 1. 4 m (D) 1. 8 m
Solution: (B) Here energy is conserved.
1 1
KX 2 = mv 2
 2 2
m 16
 X=
v
√ k =
4

100 = 1.6 m

Join Elite Batch – Physics ( Batch -2 ) - Whatsapp -9264953797

Click Here - https://imjo.in/JKBjbX ( Batch Starting 01 March )


Final Revision Questions
2020
– Elite Batch

23. A small block of mass 500 gm is pressed


against a horizontal spring fixed at one end to a
0.1m
distance of 10 cm. when released, the block
moves horizontally till it leaves the spring.
Where it will hit the ground 5 m below the 5m
spring. (Take k = 100 N/m of spring)
(A) 1.441 m (B) 1. 6 m
(C) 1.414 m (D) 1. 10 m
kx 2 /2
Solution: (C) v horizontal of block =
100×0 . 01
√ m/2 RH= 1.414m

vH = √1
0 .5 = √ 2 m/s

Now 2 gt2 = 5 mt = 10/10 = 1sec (where g = 10


m/s2)
so RH = vH  1 = √ 2 1 = 1.414 m
Block will hit the ground at 1.414 m from a point 5m below the spring.

24. A block of mass 0.5 kg is kept on a rough =0.2

inclined plane making an angle of 30 with


0.5kg
horizontal. What power will be required to move
the block up the plane (along the plane) with a P
g
velocity of 5m/s? (Take  = 0.2 between block 30

and plane)

(A) 16.825 N-m/s (B) 16.822 N-m/s


(C) 16.852 N-m/s (D) 16.528 N-m/s
Solution : (A)The total downward force acting on the block

= 0.510
( 12 +0 . 2× √23 ) mg sin

mg cos
= 5[0.5 + 0.173] = 3.365 N.
Now the power required to move up along the
inclined power = 3.365  5 = 16.825 N-m/s

25. A small body of mass m is located on a horizontal plane. The body acquires a
horizontal velocity v0 . Find mean power developed by the frictional force,
during the whole time of its motion. If coefficient friction x = 0.27, mass of body
m = 1kg, and V0 = 1.5 m/s
−μ mgv 0
(A)
−μmgv0 (B) 2

Join Elite Batch – Physics ( Batch -2 ) - Whatsapp -9264953797

Click Here - https://imjo.in/JKBjbX ( Batch Starting 01 March )


Final Revision Questions
2020
– Elite Batch

−μ mgv 0 −μ mgv 0
(C) 4 (D) 8
Solution: (B) Work done against friction w = - mgs
dw ds
P = dt = – mgs. dt = – mgs.v
V0

∫P dv
0 −μ mg V 0
V0
=
v0
∫0 v . dv
−μ mgv 0
∫ dv
Pav = 0 = 2

26. A block of mass m moving with a velocity v0 on a smooth horizontal floor


collides with a light spring of stiffness k that is rigidly fixed horizontally with a
vertical wall. If the maximum force imparted by the spring on the block is F,
then prove that
F = v0 √ km
(A) √ km v 0 (B)
4 √ km v 0
(C)
2 √ km v 0 (D) √ km
Solution : (A)Applying work energy theorem
PE+ KE = 0
1 1 2
 2 2
kx - 2 v
m 0 =0
m
 x= √ v
k 0
 Fmax = kx = √ km v 0
27. A bus of mass m produces a constant power P. If the resistance to motion is R.
Find the maximum speed at which the bus can travel on level road and
acceleration of bus when it is travelling at half of its maximum speed.
R
(A) a = R (B) a = m
R 1
(C) a = 2 m (D) a = m
Ans. (B)
Solution : Speed will be maximum, when engine of bus will produce maximum power i.e. P
P
 P = R.vmax  vmax = R

Join Elite Batch – Physics ( Batch -2 ) - Whatsapp -9264953797

Click Here - https://imjo.in/JKBjbX ( Batch Starting 01 March )


Final Revision Questions
2020
– Elite Batch

V max
Here v = 2
Force by which engine pulls the bus is
v max
P = F. 2
2P 2P
=
v max P
 F= R() =2R
 Since F – R = m.a
 2 R – R = m.a
R
a= m

28. A uniform rod of length L and mass m hinged at one end is hanging vertically.
The other end is now raised until it makes an angle 60 o with vertical. How much
work is required?
mgℓ
(A) mg ℓ (B) 8
mgℓ mgℓ
(C) 6 (D) 4
Ans. (D)
Solution: We can consider its whole mass is centre ted at its centre of mass O.

 work done = mgh


ℓ ℓ
= mg
( − cos60 °
2 2 ) 60

( 12 − 14 )
h
mgℓ O
= mg = 4

29- 32. A point mass m starts from rest and slides down the m
surface of a frictionless solid sphere of radius r as
shown in figure. Measure angles from the vertical and 
potential energy from the top. Find r
29. The change in potential energy of the mass with angle
(A) U =  mgR(1  cos) (B) U = + mgR(1  cos)
(C) U = + mgR(1 + cos) (D) U =  mgR(1 + cos)
Ans. (A)

Join Elite Batch – Physics ( Batch -2 ) - Whatsapp -9264953797

Click Here - https://imjo.in/JKBjbX ( Batch Starting 01 March )


Final Revision Questions
2020
– Elite Batch

A
Solution: Consider the mass when it is at the point B. R(1 - cos
UA (P.E. at A) = 0  B
r
UB(P.E. at B) = mgR (1  cos)
 U = UB  UA
 U =  mgR(1  cos)
Negative sign indicates that P.E. decreases
as particle slides down.
30. The kinetic energy as a function of angle
(A) T = mgR(1  cos) (B) T = mgR(1 + cos)
(C) T = – mgR(1  cos) (D) T = gR(1  cos)
Ans. (A)

Solution: Conserving energy at points A and B.


UA + TA = UB + TB
where UA = P.E. at A, UB = P.E. at B
TA = K.E. at A, TB = K.E. at B
 0 + 0 = mgR(1 cos) + TB
 T = mgR(1  cos)

31. The radial and tangential acceleration as a function of angle.


(A) at = sin (B) at = gsin
(C) at = g (D) at = –gsin
Ans. (B)
1 2
mv
Solution: Since T = 2
1 2
mv
 2 = mgR(1 - cos2mgR sin2

 v = 2 gR sin/2

 aradial = v2/R
 ar = 4g sin2/2
at = g sin
also, in circular motion velocity is along the tangent, therefore

vtangential = 2 √ gR sin/2
d
dt
( vt )
 at =

 at = (gR) cos/2 dt = (gR) cos/2
 at = (g/R) v cos/2 , as R = v

Join Elite Batch – Physics ( Batch -2 ) - Whatsapp -9264953797

Click Here - https://imjo.in/JKBjbX ( Batch Starting 01 March )


Final Revision Questions
2020
– Elite Batch

 at = gsin

A N
32. The angle at which the mass flies off the sphere.
(A) = cos-1(4/3) (B) = cos-
1 
(1/3) mgcos

(C) = cos-1(2/3) (D) = cos- mgsin


1
(2/8)
Ans. (C)

Solution : For circular motion


mv 2
mg cos - N = R
at the moment when the particle brakes off
the sphere N = 0.
mv 2
 mg cos = R
 g cos = v2/R

 v = 2 gR sin/2

g cos = 4g sin2/2 = 2g(1  cos)
 cos = 2/3
 = cos-1(2/3)

33-37. A block of ice mass 10 kg slides down an incline 5 m long and 3 m high. A man
pushes up on the ice block parallel to the incline so that it slides down at
constant speed. The coefficient of friction between the ice and the incline is
0.1. Find :
33. The work done by the man on the block
(A) Wm = –300 J (B) Wm = –260 J
(C) Wm = –400 J (D) Wm = –560 J
Ans. (B)

Solution : From the figure sin = 3/5 and cos =


4/5
F.B.D. of the block 5m
3m
F
N 

mgsin mgcos

F = force by the man

Join Elite Batch – Physics ( Batch -2 ) - Whatsapp -9264953797

Click Here - https://imjo.in/JKBjbX ( Batch Starting 01 March )


Final Revision Questions
2020
– Elite Batch

 = frictional force
N = normal reaction of the surface
mg = gravity
Since block slides with constant speed, hence
mg sin= F + 
 F = mg sin - 
3 4
−0 .1 ´ 10´ 10 ´ =52 N
= 10  10  5 5 ,
as f =  mg cos
(A) Wman = ⃗F . ⃗S =FS cos180=−FS

Here F = 52 N, and S = 5m.


 Wm = -52  5 J = -260 J
34. The work done by gravity on the block
(A) Wgravity = 300 J (B) Wgravity = 400 J
(C)Wgravity = 600 J (D) Wgravity = 500 J
Ans. (A)
Solution : Wgravity = mg S sin= 10  10 5  (3/5) J = 300 J
35. The work done by the surface on the block
(A) Wsurface = + 40 J (B) Wsurface = - 42 J
(C) Wsurface = - 41 J (D) Wsurface = – 40 J
Ans. (D)
Solution : Wsurface = WN + Wfriction = 0 + fS cos180o = 0 - mg cos S
= -0.1  10  10  (4/5)  5 J = -40 J
36. The work done by the resultant forces on the block
(A) W = 0 (B) W = 2
(C) W = 3 (D) W = 6
Ans. (A)
Solution : Work done by the resultant force is given by
W = Wm + Wg + WN + Wf = -260 J + 300 J + 0 - 40 J = 0

37. The change in K.E. of the block .


(A) K.E. = 0 (B) K.E. = 6
(C) K.E. = 4 (D) K.E. = 3
Ans. (A)

Solution : Since net work done by all the force is zero, hence change in K.E. = 0.

Join Elite Batch – Physics ( Batch -2 ) - Whatsapp -9264953797

Click Here - https://imjo.in/JKBjbX ( Batch Starting 01 March )


Final Revision Questions
2020
– Elite Batch

   

38: If a constant force F = 3 i+ 4 j+ 5 k is acting on a particle and displacement of


 
particle becomes r = 3 i+ 4 j+ 6 k . Find net work done by force F .
(A) 55 J (B) 80 J
(C) 90 J  (D) 100 J
w  F.r   3i  4j  5k  .  3i  4j  6k   9  16  30  55J
Solution: (A)

39: A particle is moving on circle of radius 5mt. A force of constant magnitude 10 N


is acting on particle along tangent find work done by this force when particle
will complete one circle.
(A) 100J (B) 200J
(C) 300J (D) 400J
Work is zero because displacement is zero.

F Correct solution: (A) Force F is varying because its direction is changing


dr
so w  F.r
  
dw  F.dr  Fdr
where dr is a small path length and small path length will
always be along tangent
 w   Fdr  F dr  F  2r
 10  2    5
 100J

F 40: A block of mass 10kg is pulled by a force F having


magnitude 20N. Find work done by force F if body
B moves 5mt in right direction. Given that l0 = 1 mt. and x
A
F l0
= 25mt initially.
 F  636  401 J
(A)  
x
 F  626  401 J
(B)  
 F  646  401 J
(C)  
 F  696  401 J
(D)  
Solution: In this case force F is variable
   

F   Fcos  i  F sin  j 
Common mistake:  

dw  F.dxi

Join Elite Batch – Physics ( Batch -2 ) - Whatsapp -9264953797

Click Here - https://imjo.in/JKBjbX ( Batch Starting 01 March )


Final Revision Questions
2020
– Elite Batch

= [F cos  i + F sin  j]. dx i


= F cos  dx
x
l0 2  x 2
cos  =
F

l0
text

y x

20
Fx
 dw   dx  F  l02  x 2   0
l0  x
2 2  
25

We know that in this case angle between F and displacement is acute so work
should be positive, contradiction has come.
Correct solution: (B) y = 25 – x  dy = – dx and
dw = (F cos  i + F sin  j). dy i
dw = –Fcos  dx
25
Fx
dx  F  l0 2  x 2 
20
w  
25
l0  x
2 2  
20

 F  626  401 J
 
20 m/s
41: A projectile is projected with initial velocity 20m/s
30 
and at an angle of 30 from horizontal. Find the

r total work done when it will hit the ground.
(A) 0 (B) 10
(C) 20 (D) 30
Solution: (A)work = (mg j) . (r i) = 0

42. A particle of mass 1 kg is moving along x-axis and a force F is also acting along x-axis
and a force F is also acting along x-axis in such a way such that its displacement is
varying as: - x = 3t2. Find work done by force F when it will move 2mt.
(A) 12 J (B) 16 J
(C) 32 J (D) 42 J

Solution: (A) x = 3t2

Join Elite Batch – Physics ( Batch -2 ) - Whatsapp -9264953797

Click Here - https://imjo.in/JKBjbX ( Batch Starting 01 March )


Final Revision Questions
2020
– Elite Batch

dx
 vel.  v  6t m / s
dt
dv
Acceleration   6m / s2

dt
r Force acting on particle = ma = 1  6 = 6N

F
2
Displacement = 2
work = work  F.r  Fr  6  2  12J

43: If a projectile is projected with initial speed u and angle  from horizontal
then what will be its average power up to time when it will hit the ground
again.
(A) 100 (B) 200
(C) 300 (D) 0

Solution:(D)
20 m/s

30

r
When it will hit the ground the net work done is zero. So
 w 0
P  0
T T

44: If a particle is moving on straight line and a constant instantaneous power is


supplying on the particle then find displacement of particle as a function of
time.
3/2
 1  3c 1/ 3 
x  t

 3  m  
(A)
3/2
 5  3c 1/ 3 
x   t

 3  m  
(B)
3/2
 2  3c 1/ 3 
x   t

 3  m  
(C)
3/2
 10  3c 1/ 3 
x   t
(D)  3  m  

Solution: (C)
Fv = constant = c

Join Elite Batch – Physics ( Batch -2 ) - Whatsapp -9264953797

Click Here - https://imjo.in/JKBjbX ( Batch Starting 01 March )


Final Revision Questions
2020
– Elite Batch

 mAv  c
c c
v  A
mA mv
v x
dv c c
v    v dv   dx
2

dx mv 0 0
m
1/ 3
v3 c  3c 
  x  v   x
3 m m 
1/ 3 x 1/ 3
dx  3c  dx t  3c 
 x   1/ 3    
dt  m  0
x 0 m
 
3/2
 2  3c 1/ 3 
 x     t
 3  m  

45. A fast moving neutron suffers one-dimensional elastic collision with a nucleus
7 N 14 . Wha approximate percentage of energy is lost by the neutron in the
collision ?
(A) 5% (B) 10 %
(C) 25 % (D) 0%
4x 4  14 5600
 100   100   24.9  25
Solution: (C) (1 x)2
(1 14) 2
225

46. An 8 kg block accelerates uniformly from rest to a velocity of 4 ms –1 in 40


second. The instantaneous power at the end of 8 second is
(A) 0.64 W (B) 0.32 W
(C) 0.16 W (D) 0.08 W.
4  0 2 1
a ms  ms2
Solution: (A) 40 10
1
0  8  0.8ms1
Velocity at the end of 8 second 10
1
 8  8  0.8 W  0.64 W
Required power = (ma) 10

47: Work is which type of physical quantity.


(A) vector (B) scalar
(C) tenser (D) None of these.
Solution : (B)

Join Elite Batch – Physics ( Batch -2 ) - Whatsapp -9264953797

Click Here - https://imjo.in/JKBjbX ( Batch Starting 01 March )


Final Revision Questions
2020
– Elite Batch

48: An engine is working at a constant power draws a load of mass m against a resistance
r. Find maximum speed of load and time taken to attain half this speed.
Pm Pm
t t
(A) 8r 2 (B) 8r
Pm Pm
t 2 t 2
(C) r (D) 9r
Solution: (A)
 f v  CP
P
 f
V
At the time of maximum velocity f = r. i.e.,
 net force onload  0
P P
r  Vmax 
Vmax r
P
F
v
dv P
m. 
dt v
P.2r
P t
0 v.dv   m  0 dt
2
P P
 2r    m  t
   
Pm
t 2
8r

49. Block m has given a velocity v0 towardly right.


m What will be the maximum power
smooth
delivered by the spring force during the
entire motion.
5 3
(A) 4 (B) 7
3 1
(C) 4 (D) 4

Solution: In the given case v = v0 cos(t) ……(i)


 v0 
   sin t
x=   …….(ii)
Acceleration = - v0  sin(t) …….(iii)

Join Elite Batch – Physics ( Batch -2 ) - Whatsapp -9264953797

Click Here - https://imjo.in/JKBjbX ( Batch Starting 01 March )


Final Revision Questions
2020
– Elite Batch

Force = mass  acceleration = mv0 sin(t)


k
m Power = - force v
= mv02 sin(t) cos (t)
mv 02 
sin(2t)
P= 2  max power
mv 2
 3
 0 ,
P max = 2 for t = 4 4
 minimum power
3
So Ans = 4 (C)

50: A particle of mass 10kg is moving with velocity


F

2i + 3j + 4k what will be its kinetic energy. If A constant force = i - j - 2k
F
t4

t2
10 U
t5 starts acting on body then find after 2 second its power and work done by
F t3
t1
t6 this force in 2 second.
t(sec) 74 84
0 1 2 4
 J  J
(A) 5 (B) 5
105 84
 J  J
(C) 5 (D) 5

F i  j  2k
 m / s2
Solution: Acceleration = mass 10
  1  1  i  j  2k 
S  at  at 2  (2i  3j  4k)  2   4 2   
2 2  10 
21 29 38
 i j k
5 5 5
21 29 76 84
     J
Work = F.S = 5 5 5 5

ui  4  9  16  29

ui  v 0
84 1
  k.E   10(v 02  29)
5 2
84
v 02  29 
25
641
v m/s
5 (D)

Join Elite Batch – Physics ( Batch -2 ) - Whatsapp -9264953797

Click Here - https://imjo.in/JKBjbX ( Batch Starting 01 March )


Final Revision Questions
2020
– Elite Batch

. If a spring mass system is placed on a horizontal v0


smooth surface. Then what will be the net work done k smooth
by spring force after one time period.
1 1
km2  km2
(A) 2 (B) 2
1 1
 km2  km2
(C) 4 (D) 4

1
spring  friction  0  mv 02
Solution: 2
1 1
  kx 2  mgx   mv 02
2 2
 kx  2mgx  mv 0  0
2 2

2mg  4 2m2g2  4kmv 02


x
2k
mg  2m2 g2  4kmv 02
x
k
v0
k

v=0
k

1
 km2
So work by spring = 2 (B)

Check whether the force F = xi+ yj + zk is conservative or not.
(A) conservative
(B) non conservative
(C) variable
(D) none of these.

Solution: Let a particle on which force F is acting moving from
(x1, y1, t1) to (x2, y2, t2) then
dw = F .d r and d r = dxi + dyj + dzk
dw = (xi + yj = zk).(dxi + dyj + dzk)
( x 2 ,y 2 z2 )
 x 2 y 2 z2 
w     
2 2 2 ( x ,y z )
! 1 1

Join Elite Batch – Physics ( Batch -2 ) - Whatsapp -9264953797

Click Here - https://imjo.in/JKBjbX ( Batch Starting 01 March )


Final Revision Questions
2020
– Elite Batch

So above work depend only on the coordinates of point (x 1, y1, z1) and (x2, y2, z2).
So force is conservative.
(A)

53: Prove that the central force which follows the inverse square law are
Y conservative forces.
1 1 1 1
A B k   k  
r r r r
(A)  1 2  (B)  1 2 
 1 1
k  
r r k  r1  r2 
X (C)  1 2  (D)
k
F
Solution: Let consider a central force r 2 . Now we will prove that if
k
F
a body on which the force is acting r 2 , will move from A to B then work done
 k(xi  yj)
F  2 2 3/2 
on it will not depend on both (x  y ) , r  xi  yj  dr  dxi  dyj
k
0 dw  ( x1,y1 ) (x2  y 2 )3 / 2  x dx  y dy 
w ( x2 ,y 2 )

 1 1  1 1
k    k  
 x1  y12 x 2  y 2   r1 r2 
2 2 2
(B)
 2

2

2

54: Prove that F = x i+ y j+ z k is a conservative force.


1 2
(A) w = 2 2

x  x12  y 22  y12  z22  z12 
1 2
(B) w = 4 2

x  x12  y 22  y12  z22  z12 
1 2
(C) w = 6  
x 2  x12  y 22  y12  z22  z12
1
(D) w = 10 2

x 2  x12  y 22  y12  z22  z12
 2
Solution: F  x i  y 2 j  z2k
Let particle moves from (x1, y1, z1) to (x2, y2, z2)
dw = x2 dx + y2dy + z2dz
1 2
w= 2 
x 2  x12  y 22  y12  z22  z12 
which depend only on (x1, y1, z1) and (x2, y2, z2)
Join Elite Batch – Physics ( Batch -2 ) - Whatsapp -9264953797

Click Here - https://imjo.in/JKBjbX ( Batch Starting 01 March )


Final Revision Questions
2020
– Elite Batch


So F will be conservative.
(A)

55: A particle of mass m is moving in a circular path of constant radius r


such that centripetal acceleration ac is varring with time t as ac= k2r t2
where k is constant what is the power delivered to the particle by the
forces acting on it.
mk 4r 2t 5 mk 4 r 2t 5
(A) 4 (B) 5
mk 4 r 2t 5 mk 4 r 2t 5
(C) 3 (D) 9
Solution
dv
 k 2rt 2
dt
k 2rt 3
 v
3
Centripetal force will not supply the power and power by the tangential force =
dv
v
m dt (P = FV)
2 3
k rt
= mk2rt2 3
mk 4 r 2t 5
= 3 (C)

56: Find the total compression in the spring.


V0
m` k
  ax
x=0
L
Smooth

mV02  amgL2
x 
(A) k
mV02  amgL2
x 
(B) k
mV02  amgL2
x 
(C) k2

Join Elite Batch – Physics ( Batch -2 ) - Whatsapp -9264953797

Click Here - https://imjo.in/JKBjbX ( Batch Starting 01 March )


Final Revision Questions
2020
– Elite Batch

mV02  amgL2
x 
k
(D) 2

Solution: E Ti = ETf – work done by nonconservative force


1
mv 02
E Ti = 2
1 2
kx
E Tf = 2
L amgL2
w=-
0
ax mg dx  
2
1 1 2 amgL2
mV0  kx 
2

2 2 2

mV02  amgL2
x 
k (A)

57. Power applied to a particle varies with time as P = (4t3 – 5t + 2)watt, where t is in
second. Find the change is its K.E. between time t = 2 and t = 4 sec.
(A) 212 J (B) 213 J
(C) 214 J (D) 215 J
Solution: (C)
P = maV
dV
= mv dt = 4t3 – 5t + 2
mv 2 5t 2
K. E. = 2 = t4 – 2 + 2t
So change in K.E. will be 214 J. (C)

58: An engine develops 10KW of power. How much time will it take to lift a mass of
200kg to a height 40m. (g = 10 ms-2)
(A) 4s (B) 5s
(C) 8s (D) 10s
W
P
Solution: (C) t
W mgh 200  10  40
t    8s
P P 10  100 (C)

59: What is work done when body will displace perpendicular to the force.

Join Elite Batch – Physics ( Batch -2 ) - Whatsapp -9264953797

Click Here - https://imjo.in/JKBjbX ( Batch Starting 01 March )


Final Revision Questions
2020
– Elite Batch

(A) zero (B) maximum


(C) minimum (D) None of these.
Solution: (A) W = F.d = Fd cos
 = 90o
then W = 0 (A)

60: A tube-well pump out 2400kg of water per minute. If water is coming out with a
velocity of 3m/s, the power of the pump is
(A) 120W (B) 180 W
(C) 240 W (D) 90 W
Solution: (B) Mass of water pumped per second
2400
m  40kg
60
v  3m / s
Kinetic energy of water coming out per second
1 1
 mv 2   40  3  3  180J
2 2
 Power of pump = 180 J /s = 180 W (B)

ASSIGNMENT

1. A planet of mass m is revolving round the sun (of mass M s) in an elliptical orbit. If  is the
velocity of the planet when its position vector from the sun is r , then areal velocity of the
planet is.
   
(A)   r (B) r 
1   1  
(C) 2
(
n´ r ) (D) 2
(r´n )
MCQ - PG- 159 - Q-1
Ans. (D)
 
DA L
= =
Solution: Areal velocity Dt 2m
1   1  
Þ Areal velocity = éëm ( r ´ v ) ù
û = ( r ´ v)
2m 2

2. A system consists of n identical particles each of mass m. The total number of interactions
possible is.
1
n(n + 1)
(A) n(n + 1) (B) 2

Join Elite Batch – Physics ( Batch -2 ) - Whatsapp -9264953797

Click Here - https://imjo.in/JKBjbX ( Batch Starting 01 March )


Final Revision Questions
2020
– Elite Batch

1
n(n - 1)
(C) n(n –1) (D) 2
MCQ - PG- 159 - Q-4
Ans. (D)
n ( n - 1)
= nC 2 =
Solution: Total number of interactions 2

0
3. The ratio of the time period of a simple pendulum of length with a pendulum of infinite
length is.
0
(A) zero (B) R
0 + R R
R 0 + R
(C) (D)
(Where, R is the radius of earth)
MCQ - PG- 160 - Q-5
0 R
t1 = 2 p t 2 = 2p
Solution: (B) g and g

4. The rotation of the earth having R radius about its axis speeds up to a value such that a man
at latitude angle 60 ° feels weightlessness. The duration of the day in such a case is.
R R
2p 4p
(A) g (B) g
g g
2p 4p
(C) R (D) R
MCQ - PG- 160 - Q-6

Solution: (B) 0 = g - Rw2 cos2 60°


R 2 w2
Þ =g
4
g
Þ w=2
R
R
Þ T = 4p
g
5. At what height the gravitational field reduces by 75% the gravitational field at the surface of
earth?
(A) R (B) 2R
(C) 3R (D) 4R
Join Elite Batch – Physics ( Batch -2 ) - Whatsapp -9264953797

Click Here - https://imjo.in/JKBjbX ( Batch Starting 01 March )


Final Revision Questions
2020
– Elite Batch

MCQ - PG- 160 - Q-7


æ 3g ö gR 2
çè g - ÷ =
4 ø ( R + h) 2
Solution: (A)
Þ h=R

k 
I   
6. In a certain region of space gravitational field is given by  r  . Taking the reference
point to be at r = ro with v = vo, the potential (V) is given by
r
log  vo log
r
 vo
(A) V=K ro (B) V=K ro
r ro
log o  v o log  vo
(C) V=K r (D) V=K r
dV
I
Solution: (C) We know that dr
v r
k
 
dV ∫ dv =K ∫ drr
 r dr  Vo ro

r
V  K log  vo
 ro

7. Two different planets have same density but different radii. The acceleration due to gravity
(g) on the surface of the planets is dependent on its radius (R) as.
1
gµ g µ R2
(A) R2 (B)
1

(C) R (D) gR
MCQ - PG- 160 - Q- 9.
æ4 p R3 ö
Gç r
GM è 3 ÷ ø
g= 2 =
Solution: (D) R R2
4 p RGr
Þ g=
3

8. A particle of mass m lies at a distance r from the centre of earth. The force of attraction the
particle and earth is F(r).
1 1
F(r) µ F(r) µ
(A) r 2 for r < R (B) r 2 for r R

Join Elite Batch – Physics ( Batch -2 ) - Whatsapp -9264953797

Click Here - https://imjo.in/JKBjbX ( Batch Starting 01 March )


Final Revision Questions
2020
– Elite Batch

1
F(r) µ
(C) F(r) µ r for r < R (D) r for r < R
MCQ - PG- 161 - Q-10
ì GMm
ïï r 2 r³ R
F=í
ï 4 p Gr rm r <R
Solution: (B, C) ïî 3
where  is density of earth.

9. A shell of mass M and radius R has another point mass m placed at a distance r from its
centre (r > R). The force of attraction between the shell and point mass is.
GMm GMm
F F
(A) r (B) r2
(C) F = zero (D) None of above
MCQ - PG- 161 - Q-13
éGMm
ê 2 r³ R
F=ê r ( outside and at surface)

êë0 r <R
( inside)
Solution: (B)

10. If gh and gd be the accelerations due to gravity at a height h and at depth d, above and below
the surface of earth respectively. Assuming h << R and d <<R and if g h = gd then,
(A) d=h
(B) d = 2h
(C) h = 2d
(D) Data is insufficient to arrive at a conclusion.
MCQ - PG- 161 - Q-14
æ 2h ö æ dö
g ç1 - ÷ @g ç1 - ÷
Solution: (B) è R ø è Rø
Þ d = 2h

11. A particle of mass m is located at a distance r from the centre of a shell of mass M and
radius R. The force between the shell and mass is F(r). The plot of F(r) vs r is.
F(r) F(r)

r r
O R O R

Join Elite Batch – Physics ( Batch -2 ) - Whatsapp -9264953797

Click Here - https://imjo.in/JKBjbX ( Batch Starting 01 March )


Final Revision Questions
2020
– Elite Batch

F(r) F(r)

r
O R
r
O R
MCQ - PG- 162 - Q-19
ì GMm
ï , r³ R
Þ F = í r2
ïî 0 , r <R
Solution: (A)

12. In PROBLEM 19, if the shell is replaced by a sphere of same mass and radius then the
graph of F(r) vs r will be (select your answer from answers of PROBLEM 19).
(A) (A) (B) (B)
(C) (C) (D) (D)
MCQ - PG- 162 - Q-20
ì GMm
ïï r 2 , r³ R
F( r) = í
ï 4 p Gr rm , r < R
Solution: (C) ïî 3
where  is density of sphere.

13. A shell of mass M and radius R has a point mass m placed at a distance r from its centre.
The gravitational potential energy U(r) vs r will be.
O r O r

U(r) U(r)

Join Elite Batch – Physics ( Batch -2 ) - Whatsapp -9264953797

Click Here - https://imjo.in/JKBjbX ( Batch Starting 01 March )


Final Revision Questions
2020
– Elite Batch

O R O R
r r

U(r) U(r)
MCQ - PG- 163 - Q-21
ì GMm
ïï - r , r³ R
U( r) = í
ï - 4Gm , r <R
Solution: (C) ïî R

14. A satellite is revolving round the earth in an orbit of radius r with time period T. If the satellite
r  r  r  r 
is revolving round the earth in an orbit of radius with time period
T  T  T  T 
then.
DT 3 Dr DT 2 Dr
= =
(A) T 2 r (B) T 3 r
DT Dr DT Dr
= =-
(C) T r (D) T r
2 3
Solution: (A) Since, T = kr
DT Dr
Þ 2 =3
T r
DT 3 Dr
Þ =
T 2 r
MCQ - PG- 163 - Q-24

15. A body of mass m is situated on the earth in the gravitational field of sun. For the body to
escape from the gravitational pull of the solar system the body must be imparted an escape
velocity of (assume earth to be stationary).
(A) 11.2 kms – 1 (B) 22.4 kms – 1
(C) 33.6 kms – 1 (D) 42 kms – 1
MCQ - PG- 164 - Q-25
2GMs
v=
Rse
Solution: (D)
2 ( 6.67 ´ 10 - 11 ) ( 2 ´ 1030 )
Þ v=
1.5 ´ 1011
Join Elite Batch – Physics ( Batch -2 ) - Whatsapp -9264953797

Click Here - https://imjo.in/JKBjbX ( Batch Starting 01 March )


Final Revision Questions
2020
– Elite Batch

Þ v = 42.174 kms- 1

16. Consider an infinite plane sheet of mass with surface mass density . The gravitational field
intensity at a point P at perpendicular distance r from such a sheet is
(A) Zero (B) – G
(C) 2G (D) – 4G
MCQ - PG- 164 - Q-26
Solution: (C) Use Gauss Theorem for Gravitation.

17. In problem 26, the gravitational potential at the point P is


(A) Zero (B) – G
(C) 2G (D) – 4G
MCQ - PG- 164 - Q-27
dV
E=-
Solution: (C) dr

18. Consider a circular disc of mass M, radius R with surface mass density . The gravitational
potential due to the disc at a point P lying on its axis at distance r from the centre is
(A) – 2G (B) – 4G
æ r ö æ r ö
çè1 - 2 2 ÷
ø çè1 - 2 2 ÷
ø
(C) – 2Gr r +R (D) – 4Gr r +R
MCQ - PG- 164 - Q-28
Solution: (C)
x

a
a

dx

E = - 2ps Gòsin qdq


0

Þ E = - 2ps G ( 1 - cos a )
æ r ö
Þ E = - 2ps G ç1 - ÷
è r 2 + R2 ø
æ r ö
V = - 2ps rG ç1 -
2 ÷
è r +R2
ø

19. A satellite is orbiting round the earth. While in orbit a small part separates from the satellite. The
separated part
(A) falls directly to the earth.

Join Elite Batch – Physics ( Batch -2 ) - Whatsapp -9264953797

Click Here - https://imjo.in/JKBjbX ( Batch Starting 01 March )


Final Revision Questions
2020
– Elite Batch

(B) moves in a spiral path and reaches the after few revolutions about the earth.
(C) Continuous to move in the same orbit.
(D) Move gradually farther from the earth.
MCQ - PG- 165 - Q-32
Ans. (C)

20. Two identical thin rings each of radius R are coaxially placed at a distance R. If the rings
have a uniform mass distribution and each has mass m 1 and m2 respectively, then the work
done in moving a mass m from centre of one ring to that of the other is
Gm(m1 - m2 ) ( )
2-1

(A) Zero (B) 2R

Gm 2(m1 + m2 ) Gmm1 ( 2 +1 )
(C) R (D) m2R
MCQ - PG- 165 - Q-33
Solution: (B)

( ) (
VA = Potential at + Potntial at
A due to A A due to B )
Gm1 Gm2
Þ VA = - - and
R 2R
Similarly,
( ) (
VB = Potential at + Potntial at
B due to A B due to B )
WA ® B = m ( VB - VA )
Since

Þ WA ® B =
Gm ( m1 - m2 ) ( )
2- 1
2R

21. A point P
(R 3, 0, 0 )
lies on the axis of a ring of mass M and radius R. The ring is located in y
- z plane with its centre at origin O. A small particle of mass m starts from P and reaches O
under gravitational attraction only. Its speed at O will be.
GM Gm
(A) R (B) R

Join Elite Batch – Physics ( Batch -2 ) - Whatsapp -9264953797

Click Here - https://imjo.in/JKBjbX ( Batch Starting 01 March )


Final Revision Questions
2020
– Elite Batch

GM Gm
(C) 2R (D) 2R
MCQ - PG- 165 - Q-34
æ Total ö æ Total ö
çMechanical ÷ = çMechanical ÷
è Energy øP è Energy øO
Solution: (A)
1 GMm 1 GMm
m ( 0) -
2
Þ = mv 2 -
2 2 R
( )
2
3R + R2
GMm 1 GMm
Þ - = mv 2 -
2R 2 R
GM
Þ v=
R

22. An artificial satellite moving in circular orbit around the earth has a total (kinetic + potential)
energy E0. Its potential energy and kinetic energy respectively are
(A) 2E0 and –2E0 (B) –2E0 and 3E0
(C) 2E0 and –E0 (D) –2E0 and –E0
MCQ - PG- 165 - Q-36
GMm GMm
K.E. = = - E0 , and P.E. = - = 2E 0
Solution: (C) 2r r
GMm
Þ T.E. = K.E. + P.E. = - = E0
2r

23. The ratio of Earth's orbital angular momentum (about the Sun) to its mass is 4.4  1015 m2s–1.
The area enclosed by the earth's orbit is approximately.
(A) 1  1022 m2 (B) 3  1022 m2
(C) 5  1022 m2 (D) 7  1022 m2
MCQ - PG- 166 - Q-37
Area Swept
Since Areal velocity =
Time for one Revolution of
Solution: (D) Earth about the sun
L
Further Areal velocity = 2M
æ L ö æ Time for one ö
Þ Area Swept = ç Revolution of Earth
ø çè about the sun ÷
è 2M ÷ ø
1
=
2
( )
4.4 ´ 1015 ( 365 ´ 24 ´ 60 ´ 60 )
Area Swept
Þ Area Swept = 7 ´ 1022 m2

Join Elite Batch – Physics ( Batch -2 ) - Whatsapp -9264953797

Click Here - https://imjo.in/JKBjbX ( Batch Starting 01 March )


Final Revision Questions
2020
– Elite Batch

24. A particle is projected vertically upwards from the surface of earth (radius Re) with a kinetic
energy equal to half of the minimum value needed for it to escape. The height to which it
rises above the surface of earth is
(A) Re (B) 2Re
(C) 3Re (D) 4Re
MCQ - PG- 166 - Q-38
2
1 æ 2GM ö GMm
( K.E.) escape = mç =
2 è Re ÷ø Re
Solution: (A)
1 GMm
( K.E.) body =
2 Re
initially

By Law of Conservation of Energy


æTotal Initial ö æTotal Final ö
çMechanical ÷ = çMechanical ÷
è Energy ø è Energy ø
( K.E. + P.E.) surface = ( K.E. + P.E.) at height h
1 GMm GMm GMm
Þ - = 0-
2 Re Re Re + h
{ velocity at maximum height is zero}
Þ h = Re

CMP: A satellite of mass 5000kg is projected in space with an initial speed of 4000m/s making an
angle of 30o with the radial direction from a distance 3.6107m away from the center of the
earth.

25. The angular momentum of satellite


(A) 3.6107 joule sec (B) 4.9107 joule sec
(C) 9.210 joule sec
7
(D) 3.61014 joule sec
Solution: (D) Given, r = 3.6 × 107 m, m = 5000 kg, v = 4000 m s–1,  = 30°
Angular momentum of satellite L = mvr sin
= 5000 × 4000 × 3.6 × 107 × sin 30°
= 3.6 × 1014 joule × second.

26. The semi-major axis of the orbit of satellite


(A) 6.6107 m (B) 14.9107 m
(C) 19.21017 m (D) 1.6104 m
GMm 6.67  10 11  5.97  1024  5000

Solution: (A) Now semi–major axis a = 2E 2  ( 1.5  1010 )

Join Elite Batch – Physics ( Batch -2 ) - Whatsapp -9264953797

Click Here - https://imjo.in/JKBjbX ( Batch Starting 01 March )


Final Revision Questions
2020
– Elite Batch

= 6.6 × 107m.

27. Semi-minor axis of the orbit of satellite


(A) 16.6107 m (B) 3.92107 m
17
(C) 10.210 m (D) 2.6104 m
 2EL2 
e  1 2 2 3 
 GMm 
Solution: (B) Eccentricity of the orbit
2  ( 1.5  1010 )  (3.6  1014 )2
 1
(6.67  10 11 )2  (5.97  1024 )2  (5000)3 = 0.804

Hence, semi–minor axis b = a 1  e  6.6  10 1  (0.804)  3.92  10 m


2 7 2 7

28. The minimum distance of satellite from earth


(A) 66.6107 m (B) 14.9107 m
(C) 1.29107 m (D) 1.6104 m
Solution: (C) Minimum distance rmin = a(1 –e) = 6.6 × 107 × (1 – 0.804) = 1.29 × 107m.

29. The maximum distance of satellite from earth.


(A) 6.6107 m (B) 24.9107 m
(C) 11.9107 m (D) 1.6104 m
Solution: (C) Maximum distance rmax = a (1 + e) = 6.6 × 107 × (1 + 0.804) = 11.9 × 107m

30. The energy of satellite


(A) 1.6107 joule (B) 4.9107 joule
(C) 0.2107 joule (D) -1.51010 joule
1 GMm
m2 
Solution: (D) Energy of the satellite E = 2 r
1 (6.67  10 11)  (5.97  10 24 )  5000
 500  (4000) 
2

=2 3.6  107
= –1.5 × 1010 joule.

For Test Paper

1. Imagine a light planet revolving around a very massive star in a circular orbit of radius R with
a period of revolution T. If the gravitational force of attraction between the planet and the star
is proportional to R-5/2, then,
(A) T2 is proportional to R2 (B) T2 is proportional to R7/2
2 3/2
(C) T is proportional to R (D) T2 is proportional to R3.75
MCQ - PG- 166 - Q-41
2 -5/2
Solution: (B) mRw µ R

Join Elite Batch – Physics ( Batch -2 ) - Whatsapp -9264953797

Click Here - https://imjo.in/JKBjbX ( Batch Starting 01 March )


Final Revision Questions
2020
– Elite Batch

Þ T 2 µ R7 / 2

2. If the earth suddenly stopped in its orbit (assume orbit to be circular) the time that would
elapse before it falls into the sun is
1 1
T T
(A) 2 (B) 2 2
1 1
T T
(C) 4 2 (D) 8 2
MCQ - PG- 167 - Q-44
t
Solution: (C) The time 0 taken by earth to fall into the sun can be calculated by considering a very
elongated ellipse having major axis equal to the radius of orbit of earth about sun (r).
Þ 2a = r
According to Kepler's Third Law,
( 2t 0 )
2
µ a3
3
ær ö
( 2t 0 )
2
Þ µç ÷
è2 ø
2 3
Since, T µ r
2t 0 1
Þ =
T 8
T 365
Þ t0 = = 64.53 days
4 24 2

3. A projectile is fired upwards from the surface of the earth with a velocity kv e where ve is the
escape velocity and k < 1. If r is the maximum distance from the centre of the earth to which
it rises and R is the radius of the earth, then r is
R 2R
(A) k2 (B) 1- k 2
2R R
(C) k2 (D) 1- k 2
MCQ - PG- 167 - Q-45
æ Total ö æ Total ö
çMechanical ÷ = çMechanical ÷
è Energy øsurface è Energy ør
Solution: (D)
GMm 1 GMm
+ m ( kv e ) = -
2
Þ - +0
R 2 r
2GM
ve =
where R

Join Elite Batch – Physics ( Batch -2 ) - Whatsapp -9264953797

Click Here - https://imjo.in/JKBjbX ( Batch Starting 01 March )


Final Revision Questions
2020
– Elite Batch

R
Þ r=
1- k2

4. Two satellites S1 and S2 revolve round a planet in coplanar circular orbits in the same sense.
Their periods of revolution are 1 hour and 8 hour respectively. The radius of the orbit of S 1 is
104 km. The speed of S2 relative to S1 when they are closet (in kmh-1) is
(A) 104 (B) 2  104
1
(C) 2  104 (D) 4  104
MCQ - PG- 167 - Q-46
2 3
Solution: (A) Since T µ r
r13 T12 1
Þ 3
= 2 =
r2 T2 64
r1 1
Þ =
r2 4
Þ r2 = 4 ´ 10 4 km
2p r1 2p r2
v1 = and v 2 =
T1 T2
2p ´ 104 2p ´ 4 ´ 104
Þ v1 = and v 2 = ( in kmph)
1 8
Þ v1 = 2p ´ 104 kmh- 1 and v 2 = p ´ 10 4 kmh - 1

So, speed of S2 with respect to S1 is p ´ 10 4 kmh- 1 .

5. Two satellites S1 and S2 revolve round a planet in coplanar circular orbits in the same sense.
Their periods of revolution are 1 hour and 8 hour respectively. The radius of the orbit of S 1 is
104 km., the angular speed of S2 as observed by an astronaut in S1 is
p p
(A) 2 (B) 3
p p
(C) 4 (D) 6
MCQ - PG- 167 - Q-47
v - v1
w= 2
r2 - r1
Solution: (B)
10 4 p
Þ w=
3 ´ 10 4
p
Þ w= rads- 1
3

Join Elite Batch – Physics ( Batch -2 ) - Whatsapp -9264953797

Click Here - https://imjo.in/JKBjbX ( Batch Starting 01 March )


Final Revision Questions
2020
– Elite Batch

6. Two particles having masses m1 and m2 start moving towards each other from the state of
rest from infinite separation. Their relative velocity of approach when they are interacting
gravitationally at a separation r will be
G(m1 + m2 ) 2G(m1 + m2 )
(A) r (B) r
3G(m1 + m2 ) 4G(m1 + m2 )
(C) r (D) r
MCQ - PG- 168 - Q-48
Solution: (B) CONCEPT OF REDUCED MASS ()
Let m, be at rest and think that m 2 has been replaced by  and is moving with velocity v.
Then by Law of Conservation of Energy
1 1 Gm1m2
m ( 0 ) + mv 2 = -
2
+0
2 1 2 r
m1m2
=
where,  = reduced mass of system m1 + m2
2G ( m1 + m2 )
Þ v=
r

7. The gravitational potential on the surface of a planet of radius R mass M is


gM
(A) g (B) R
(C) –GM (D) – gR
MCQ - PG- 168 - Q-49
GM
V=-
Solution: (D) R
GM
Þ V=- 2 R
R
Þ V = - gR

8. A body is imparted a velocity v from the surface of the earth. If v 0 is orbital velocity and v e be
the escape velocity then for
(A) v = v0, the body follows a circular track around the earth.
(B) v > v0 but < ve, the body follows elliptical path around the earth.
(C) v < v0, the body follows elliptical path and returns to surface of earth.
(D) v > ve, the body follows hyperbolic path and escapes the gravitational pull of the earth.
(A) A, B (B) B, C
(C) A, B, C (D) A, B, C, D
MCQ - PG- 169 - Q-35
Solution: (D)

Join Elite Batch – Physics ( Batch -2 ) - Whatsapp -9264953797

Click Here - https://imjo.in/JKBjbX ( Batch Starting 01 March )


Final Revision Questions
2020
– Elite Batch

Velocity of Nature of Path


Satellite
1. v = v0 Circular path around the earth.
2. v < v0 Elliptical path and body returns to earth.
3. v > v0 but < ve Elliptical path around the earth and will not escape.
4. v = ve Parabolic path and it escapes from the earth.
5. v > ve Hyperbolic path and escapes from earth.

9. A particle is launched from the surface of earth with speed v. For the particle to move as a
satellite, which statement is correct?
ve ve
(A) 2 < v < ve (B) 2 < v < ve
ve ve
2v e 2<v< 2
(C) ve < v < (D)
MCQ - PG- 169 - Q-55
Solution: (B) From SOLUTION to PROBLEM 8, we have
v0 < v < ve
ve
Þ < v < ve
2
ìï GM 2GM üï
í  v0 = and v e = ý
îï r r ïþ

10. Two bodies of masses m and M are placed a distance d apart. The gravitational potential at
the position where the gravitational field due to them is zero is V.
G Gm
V=- (m + M) V=-
(A) d (B) d
GM G
( )
2
V=- V=- m+ M
(C) d (D) d
MCQ - PG- 169 - Q-56
Solution: (D) Let gravitational field be zero at a point lying at distance x from M. Then,
GM Gm
=
x 2
( d - x) 2
d- x m
Þ =
x M
d m
Þ - 1=
x M

Join Elite Batch – Physics ( Batch -2 ) - Whatsapp -9264953797

Click Here - https://imjo.in/JKBjbX ( Batch Starting 01 March )


Final Revision Questions
2020
– Elite Batch

æ M ö
Þ x=ç ÷d
è M+ mø
...(1)
æ M ö
Þ ( d - x) = ç ÷d
è M+ mø
...(2)
Gm Gm
Vp = - -
Since, d- x x ...(3)
Substituting (1) and (2) in (3), we get
G
( )
2
Vp = - m+ M
d

11. The orbital period of revolution of a planet round the sun is T 0. Suppose we make a model of
Solar system scaled down in the ratio  but of materials of the same mean density as the
actual material of planet and the sun has. The new orbital period is.
(A) T0 (B) 2T0
(C)  T0
3
(D) T0
MCQ - PG- 170 - Q-58
æ r3 ö
T 2 = 4p 2 ç
è GM ÷
ø
Solution: (D)

12. P is a point at a distance r from the centre of a solid sphere of radius R 0. The gravitational
potential at P is V. If V is plotted as a function of r, then the curve representing the plot
correctly is
O r  R0 O r  R0
r r

V V

Join Elite Batch – Physics ( Batch -2 ) - Whatsapp -9264953797

Click Here - https://imjo.in/JKBjbX ( Batch Starting 01 March )


Final Revision Questions
2020
– Elite Batch

O r  R0 O r  R0
r r

V V
MCQ - PG- 170 - Q-59
ì GM
ïï - r , r ³ R0
V =í
GM
ï-
ïî 2R0
3 (
3R02 - r 2 ) , r <R
Solution: (D)

13. A point P is lying at a distance r (< a) from the centre of shell of radius a. If E and V be the
gravitational field and potential at the point P then E = ?
GM
E=-
(A) E=0 (B) r2
GM
V=-
(C) v=0 (D) a
MCQ - PG- 170 - Q-60
Solution: (A) The field inside the shell is zero.

14. A point P is lying at a distance r (< a) from the centre of shell of radius a. If E and V be the
gravitational field and potential at the point P then, V = ?
GM
E=-
(A) E=0 (B) r2
GM
V=-
(C) v=0 (D) a
MCQ - PG- 170 - Q-60
Solution: (D) The field inside the shell is zero and so potential inside the shell is constant equal to
GM
- .
the value that exists at the surface i.e. a

15. A geostationary satellite orbits around the earth in a circular orbit of radius 36000 km. Then
the time period of a spy satellite orbiting a few hundred kilometre above the earth's surface
(Rearth = 6400 km) will approximately be

Join Elite Batch – Physics ( Batch -2 ) - Whatsapp -9264953797

Click Here - https://imjo.in/JKBjbX ( Batch Starting 01 March )


Final Revision Questions
2020
– Elite Batch

1
(A) 2 h (B) 1h
(C) 2h (D) 4h
MCQ - PG- 170 - Q-61

Solution: (C)
v = gR = 7.9 kms- 1, and
2p R 2p 6400 ´ 100
T= =
v 7900
= 84.79 minute
is the time period of satellite revolving very close to earth's surface. So time period of spy
satellite must be slightly greater and so C is the appropriate answer.

16. Consider a thin uniform spherical layer of mass M and radius R. The potential energy of
gravitational interaction of matter forming this shell is
GM2 1 GM2
- -
(A) R (B) 2 R
2
3 GM 2 GM2
- -
(C) 5 R (D) 3 R
MCQ - PG- 171 - Q-63
Solution: (B) Let us consider the shell when a mass m is already piled on it by the agency. If V is
the potential on the shell, then
Gm
V=-
R
To add a mass dm further we have
dW = Vdm
Gm
Þ dW = - dm
R
M
G

Þ W=- m dm
0

1 GM2
Þ W=-
2 R
= Potential Energy of Interaction.

17. Consider a thin uniform spherical layer of mass M and radius R, if we consider a solid sphere
of mass M and radius R, then the potential energy of gravitational interaction of matter
forming this solid sphere is
GM2 1 GM2
- -
(A) R (B) 2 R

Join Elite Batch – Physics ( Batch -2 ) - Whatsapp -9264953797

Click Here - https://imjo.in/JKBjbX ( Batch Starting 01 March )


Final Revision Questions
2020
– Elite Batch

3 GM2 3 GM2
- -
(C) 5 R (D) 2 R
MCQ - PG- 171 - Q-64
Solution: (C)

Gmdm
dU = -
r
æ4 ö
G ç p r 3r ÷( 4 p r 2drr )
è3 ø
Þ dU = -
r
16 p 2Gr 2 4
Þ dU = - r dr
3
2
æ ö
R
16 2 ç M ÷ 4
Þ U=-
3
p Gç
4 3
÷ òr dr
çè p R ÷ ø
0
3
æ ö
2
æ16 2 ö ç M ÷æR5 ö
Þ U = - ç p G ÷ç
è3 ø 16 2 6 ÷çè 5 ÷ ø
çè p R ÷ ø
9
3 GM2
Þ U=-
5 R

18. What should be the period of rotation of earth so as to make any object on the equator weigh
half of its present value?
(A) 2 hrs (B) 24 hrs
(C) 8 hrs (D) 12 hrs
Solution: (A) ge = g0  R  go/2 = go  R
2 2

g0
 R= 2
2

g0
 = 2R
2R
g0
 T = 2 , putting R = 6.4  106 m and g0 = 9.8 m/sec2, we obtain,
T = 1.99 hrs  2 hrs
Join Elite Batch – Physics ( Batch -2 ) - Whatsapp -9264953797

Click Here - https://imjo.in/JKBjbX ( Batch Starting 01 March )


Final Revision Questions
2020
– Elite Batch

19. An artificial satellite is describing an equatorial orbit at 3600 km above the earth’s surface.
Calculate its period of revolution ?
(A) 8.71 hrs (B) 9.71 hrs
(C) 10.71 hrs (D) 11.71 hrs
Solution: (A) The time period of satellite is given by
4 2 4 2 (R + h)3
T2 = GM (R + h)3 = g R2
3
2  (R  h) 2 1021
T= R g = 6400 10
3
9.8 [as R+h = (6400 + 3600)  103 m = 107m]
=31360.78 sec = 8.71 hrs

20. An artificial satellite is describing an equatorial orbit at 3600 km above the earth’s surface.
Calculate its orbital speed.
(A) 6.335 km/sec (B) 7.335 km/sec
(C) 8.335 km/sec (D) 9.335 km/sec

Solution: (A) The time period of satellite is given by


4 2 4 2 (R  h)3
T2 = GM (R + h)3 = g R2
3
2  (R  h) 2 1021
T= R g = 6400 10
3
9.8 [as R+h = (6400 + 3600)  103 m = 107m]
=31360.78 sec = 8.71 hrs
Orbital speed is
GM gR2 9.8 6400 10 
3 2

 m/s
Vo = (R h) =
(R h) 107 = 6335 m/s
= 6.335 km/sec

21. Three particles each having a mass of 100 gm are placed on the vertices of an equilateral
triangle of side 20 cm. The work done in increasing the side of the triangle to 40 cm is
2
é - 11 N - m ù
êG = 6 .67 ´ 10
ë kg 2 úû
(A) 5.0  10-12J (B) 2.25  10-10J
(C) 4.0  10-11J (D) 6.0  10-15J

Join Elite Batch – Physics ( Batch -2 ) - Whatsapp -9264953797

Click Here - https://imjo.in/JKBjbX ( Batch Starting 01 March )


Final Revision Questions
2020
– Elite Batch

Ufinal Uinitial m
Solution: (A) W =
 G mm G mm 600
3   
 r   r  d
=  f i

Putting rf 0.4m , ri 0.2


m and
2
600 600
11 N m
m
m
G  6.67 10
kg2 and m = 0.1 kg
12
We get W 5.0 10 J

22. If the time of revolution of a satellite is T, then Kinetic energy is proportional to


(A) 1/T (B) 1/T2
3
(C) 1/T (D) T-2/3
2r
v
Solution: Orbital velocity T
1
2
Hence, KE  T

23. The magnitude of gravitational potential energy of the earth-satellite system is U with zero
potential energy at infinite separation. The kinetic energy of satellite is K. Mass of satellite
<< mass of the earth. Then
U
K=
(A) K = 2U (B) 2
(C) K = U (D) K = 4U
GMm
K
Solution: (A) Kinetic energy of the satellite 2r while magnitude of the potential energy
GMm
U
r
U
K
Hence 2

24. A Saturn year is 29.5 times the earth year. How far is the Saturn from the sun if the earth is 1.5 x
108 km away from the sun?
(A) 1.43  109 km (B) 2.43  109 km
(C) 3.43  109 km (D) 4.43  109 km
Solution: (A) It is given that
Ts = 29.5 Te; Re = 1.5 x 1011 m
Now, according to Kepler’s third law
2
TS2 R3S æT ö3
= Re ç S ÷
Te2 R3e  Rs = èT ø e

Join Elite Batch – Physics ( Batch -2 ) - Whatsapp -9264953797

Click Here - https://imjo.in/JKBjbX ( Batch Starting 01 March )


Final Revision Questions
2020
– Elite Batch

æ29.5 Te ö 32
çè T ÷
ø
= 1.5  1011 e
= 1.43  1012 m = 1.43  109 km

25. A spherical planet for out in space has a mass M o and diameter Do. A particle of mass m
falling freely near the surface of this planet will experience an acceleration due to gravity
which is equal to
GMo 4GmMo
2
(A) Do (B) Do2
4GMo GmM
(C) Do2 (D) Do2
GMom
mgo = 2
æDo ö
çè 2 ÷
ø
Solution: (C) Let go to the acceleration thin
4GMo
go =
Do 2

26. A body falls freely towards the earth from a height 2R, above the surface of the earth, where
initially it was at rest. If R is the radius of the earth then its velocity on reaching the surface of
the earth is
4 2 4
gR gR gR
(A) 3 (B) 3 (C) 3 (D) 2gR
GMm
Solution: (A) Initial energy of the body = 
( R + 2R)
GMm 1
+ mv 2
Final energy of the body =  R 2
where m is the mass of the body and M is the mass of the earth.
Applying conservation of mechanical energy
GMm GMm 1
+ mv 2
 3R = R 2
1 2GMm
 2 mv =
2 3R

4GM 4GM
 v2 = 3R  v= 3R
4GMR 4
2
= gR
= 3R 3

Join Elite Batch – Physics ( Batch -2 ) - Whatsapp -9264953797

Click Here - https://imjo.in/JKBjbX ( Batch Starting 01 March )


Final Revision Questions
2020
– Elite Batch

27. Two satellite (I) and (II) are moving round a planet in circular orbit having radii R and 3R
respectively, if the speed of satellite (I) is v the speed of satellite B will be

(A) v /3 (B) v 3
(C) 3v (D) data insufficient
Solution: (C) T = 2r/v
T1 = 2R/v1 and T2 = 23R/v2
T1 1 æv 2 ö
=
T2 3 çè v1 ÷
ø
2 2
æT1 ö 1 æv 2 ö
çè T ÷ = ç ÷
2 ø 9 è v1 ø
3 3
T12 æR1 ö æ1 ö
= =ç ÷
T22 çè R2 ÷
ø è3 ø
But by Kepler’s third law
3 2
æ1 ö 1 æv 2 ö
çè ÷ =
3 ø 9 çè v1 ÷
ø v1 3
 v2 =

28. The radius of a planet is R. A satellite revolves around it in a circle of radius r with angular
speed . The acceleration due to gravity on planet’s surface will be:
r 3w r 2w 3
(A) R (B) R
r 3w 2 r 2w 2
(C) R2 (D) R
Solution: (C) Let M be the mass of the planet and m the mass of satellite. Then
GMm
mrw 2 = 3 2
r2  GM = r w
GM r 3w 2
g= 2 = 2
Now R R

29. The gravitational field in a region is given by
(
I = 4iˆ + ˆj )
N/kg. Work done by this field is
zero when the particle is moved along the line
(A) y+4x = 2 (B) 4y+x = 6
(C) x+y = 5 (D) all of the above
Solution: (A) Work done will be zero when displacement is perpendicular to the filed.
The field makes an angle
æ1 ö
θ1 = tan-1 ç ÷
è 4 ø with positive x-axis

Join Elite Batch – Physics ( Batch -2 ) - Whatsapp -9264953797

Click Here - https://imjo.in/JKBjbX ( Batch Starting 01 March )


Final Revision Questions
2020
– Elite Batch

While the line y + 4x = 2 makes an angle


θ2 = tan-1 ( -4)
with positive x–axis
o
θ1 + θ2 = 90 i.e. the line y + 4x = 2
is perpendicular to I

30. A particle of mass m is placed inside a spherical shell, away from its centre. The mass of the
shell is M.
(A) The particle will move towards the centre.
(B) The particle will move away from the centre, towards the nearest wall.
(C) The particle will move towards the centre if m < M and away from the centre if m > M.
(D) The particle will remain stationary.

Solution: (D) As the gravitational field inside the shell is zero, the particle will remain stationary

57: Three solid spheres each of mass m and radius R are released 
from the position shown in figure. The speed of any one sphere at
d d
the time of collision would be
æ1 3 ö æ3 1 ö
Gm ç - ÷ Gm ç - ÷
èd R ø èd R ø  
(A) (B) d
æ2 1 ö æ1 2 ö
Gm ç - ÷ Gm ç - ÷
èR d ø èR d ø
(C) (D)

Solution: From conservation of mechanical energy


2R
2 2
ì1 ü ì Gm Gm ü
3 í mv 2 ý = 3 í - ý 2R
î2 þ î 2R d þ
2R
ì 1 2ü
v 2 = Gm í - ý
î R dþ
ì 1 2ü
\ v = Gm í - ý
î R dþ
Hence, (D) is correct.

CMP 1. A satellite of mass 5000kg is projected in space with an initial speed of 4000m/s making an
angle of 30o with the radial direction from a distance 3.6107m away from the centre of the
earth.

58: The angular momentum of satellite


(A) 3.6107 joule sec (B) 4.9107 joule sec
(C) 9.2107 joule sec (D) 3.61014 joule sec

Join Elite Batch – Physics ( Batch -2 ) - Whatsapp -9264953797

Click Here - https://imjo.in/JKBjbX ( Batch Starting 01 March )


Final Revision Questions
2020
– Elite Batch

Solution: (D) Given, r = 3.6 × 107 m, m = 5000 kg, v = 4000 m s–1,  = 30°
Angular momentum of satellite L = mvr sin
= 5000 × 4000 × 3.6 × 107 × sin 30°
= 3.6 × 1014 joule × second.

59: The semi-major axis of the orbit of satellite


(A) 6.6107 m (B) 14.9107 m
17
(C) 19.210 m (D) 1.6104 m
- GMm 6.67 ´ 10 - 11 ´ 5.97 ´ 10 24 ´ 5000
=-
Solution: (A) Now semi–major axis a = 2E 2 ´ (- 1.5 ´ 1010 )
= 6.6 × 107m.

60: Semi-minor axis of the orbit of satellite


(A) 16.6107 m (B) 3.92107 m
(C) 10.21017 m (D) 2.6104 m
2
æ 2EL ö
e = ç1 + 2 2 3 ÷
è GMm ø
Solution: (B) Eccentricity of the orbit
2 ´ ( - 1.5 ´ 1010 ) ´ (3.6 ´ 1014 )2
= 1+
(6.67 ´ 10- 11 )2 ´ (5.97 ´ 1024 )2 ´ (5000)3 = 0.804
2 7 2 7
Hence, semi–minor axis b = a 1 - e = 6.6 ´ 10 1 - (0.804) = 3.92 ´ 10 m

61: The minimum distance of satellite from earth


(A) 66.6107 m (B) 14.9107 m
(C) 1.29107 m (D) 1.6104 m
Solution: (C) Minimum distance rmin = a(1 –e) = 6.6 × 107 × (1 – 0.804) = 1.29 × 107m.

62: The maximum distance of satellite from earth.


(A) 6.6107 m (B) 24.9107 m
(C) 11.9107 m (D) 1.6104 m
Solution: (C) Maximum distance rmax = a (1 + e) = 6.6 × 107 × (1 + 0.804) = 11.9 × 107m

63: The energy of satellite


(A) 1.6107 joule (B) 4.9107 joule
(C) 0.2107 joule (D) -1.51010 joule
1 GMm
mn 2 -
Solution: (D) Energy of the satellite E = 2 r
1 2 (6.67 ´ 10 - 11) ´ (5.97 ´ 10 24 ) ´ 5000
´ 500 ´ (4000) -
=2 3.6 ´ 107
= –1.5 × 1010 joule.

Join Elite Batch – Physics ( Batch -2 ) - Whatsapp -9264953797

Click Here - https://imjo.in/JKBjbX ( Batch Starting 01 March )


Final Revision Questions
2020
– Elite Batch

OBJECTIVE PROBLEMS
1: If the elastic limit of a typical rock is 3 × 10 8 Nm–1 and its mean density 3 × 10 3 kgm–3,
estimate the maximum height of a mountain on earth. Take g as 10 ms –2

(A) 6 km (B) 12 km
(C) 8 Km (D) 10 km
Solution: (D) Let maximum stress of a mountain of height y be g
Then weight of rock in mountain
= lm2 × y × 3 × 103 × 10
Stress on base of 1m2
= y × 3 × 103 × 10
As per problem,
3 × 108 = y × 3 × 104
or y = 104 m = 10 km

2: The pressure of water in a water pipe when tap is open and closed are respectively 3 × 10 5
Nm–2 and 3.5 × 105 Nm–2. With open tap, the velocity of water flowing is

(A) 10 ms–1 (B) 5 ms–1


(C) 20 ms–1 (D) 15 ms–1
1 2
  P1  P2
Solution: (A)Here P = 2
or v2 = 2(P1 – P2)/ 
= 2 (3.5 × 105 – 3 × 105)/1000
2  0.5  105
= 1000
2  0.5  105
or = 1000 = 10 ms–1

Join Elite Batch – Physics ( Batch -2 ) - Whatsapp -9264953797

Click Here - https://imjo.in/JKBjbX ( Batch Starting 01 March )


Final Revision Questions
2020
– Elite Batch

3: Two soap bubbles of radii R and r come in contact. R is more than r. Radius of curvature of
common surface is

R
r

C o m m o n S u r fa c e

Rr Rr
(A) Rr (B) Rr
Rr Rr
(C) Rr (D) Rr
4S
Solution: (B) Using excess pressure = R , we get
4S
P1 – Patm = R
4S
P2 –Patm = r
4S 4S

or P2 – P 1 = r R
4S
 4S 4S

or rcurvature R R
1 1 1 R r
  
or rcurvature r R rR
rR
or rcurvature = Rr

4: A conical metallic body is heated at middle. If the body was voiced from its ends then
R
L

Heat

(A) There will be no effect on the body.


(B) Body will contract from middle.
(C) No strain is produced in the body.
(D) Maximum compressive stress will be on the side of body towards R.

Join Elite Batch – Physics ( Batch -2 ) - Whatsapp -9264953797

Click Here - https://imjo.in/JKBjbX ( Batch Starting 01 March )


Final Revision Questions
2020
– Elite Batch

F
Solution: (D) Stress = A
Since compressive force is same throughout the body, right side has less area of cross–
section, so stress is maximum.

5: A barometer kept in an elevator accelerating downward read x cm. The air pressure in the
elevator is

(A) x cm of mercury column


(B) less than x cm of mercury column
(C) greater than x cm mercury column
(D) nil.
Solution: (B) Net acceleration = (g –a)
 g  a x
 Pressure =   cm of mercury
ga
Since g <1
 here pressure in lift is less than x cm of
mercury.

6: A cubical block of wood 10 cm on a side floats at the interface between oil and water with its
lower surface horizontal and 4 cm below the interface. The density of oil is 0.6 g cm –1. The
mass of block is
(A) 706 g (B) 607 g
(C) 760 g (D) 670 g
Solution: (C) Volume of block = 10 × 10 × 10 = 1000 cm3
Volume of block in water = 10 × 10 × 4 = 400 cm3
6 cm Volume of block in oil = 10 × 10 × 6 = 600 cm3
Weight of block = Weight of oil displaced + Weight of water displaced
4 cm or mg = (600 × 0.6)g + (400 × 1) g
or m = 360 + 400 = 760g

7: A stone of 0.5 kg mass is attached to one end of a 0.8m long aluminium wire of 0.7 mm
diameter and suspended vertically. The stone is now rotated in a horizontal plane at a rate
such that wire makes an angle of 85 o with the vertical. If Y = 7 × 10 10 Nm–2, sin 85o = 0.9962
and cos 85o = 0.872, the increase in length of wire is

Join Elite Batch – Physics ( Batch -2 ) - Whatsapp -9264953797

Click Here - https://imjo.in/JKBjbX ( Batch Starting 01 March )


Final Revision Questions
2020
– Elite Batch

(A) 1.67 × 10–3 m (B) 6.17 × 10–3 m


(C) 1.76 × 10–3 m (D) 7.16 × 10–3 m
(A) Here T cos  = mg and T sin  = mR 
2
Solution:
T l
Using Y = A l we get
mgl
l = 2
r Y cos 
0.5  9.8  0.8
 
2
3.14  0.35  10 3  7  1010  0.872 R
m g
=
= 1.67 × 10–3 m

8: A metallic plate having shape of a square is suspended X

as shown in figure. The plate is made to dip in


water such that level of water is well above that of
the plate. the point X is then slowly raised at
constant velocity then curve between tension T in
string and displacement S of point X is given by

T T T
T S S S
S
(A) (B) (C) (D)

Solution: (B) Tension in the string is constant so long as plate remains immersed in liquid. Tension
increases linearly as the plate starts coming out of the liquid. When the plate is completely
out of the liquid, the tension in string becomes constant.

9: A large block of ice of thickness t and density  has a large


vertical hole along it axis. This block is floating in a lake. The
length of rope required to raise a bucket of water through the
hole is
(A) (  –l) (B) (1+  ) l
(C) l (D) l(1–  )
Solution: (D) Let the block has height h above the surface of water and A be the area of block.
Hereweight of ice block = weight of liquid displaced
 .A.l.g = (l – h) Ag = lAg – hAg
–h = (  –1) l or h = l (1–  )
Which is the least length of rope.

Join Elite Batch – Physics ( Batch -2 ) - Whatsapp -9264953797

Click Here - https://imjo.in/JKBjbX ( Batch Starting 01 March )


Final Revision Questions
2020
– Elite Batch

10: A ball of radius r and density  falls freely under gravity through a distance h before entering
water. Velocity of ball does not change even on entering water. If viscosity of water is  , the
value of h is given by
2 2  1   2   1 
gr   gr 2  
(A) 9    (B) 81   
2 2
2 4   1  2g 4    1 
r   g r  
(C) 81    (D) 9   

Solution: (C) Here velocity of ball =


2gh, but this velocity is
the same as terminal velocity
2 2    1
gr  
Given by 9   
2g 2    1 
2gh  r  
Then 9   
2
 2 2    1 1 2g 4    1 
2

 gr    r  
or h = 9     2g = 81   

11: A wire of brass and another of steel support a horizontal for as shown

B ra s s
S te e l

T T

A x B

Here s = 1.5 m, Ysteel = 2 × 1011 Nm–2, Ybrass = 1 × 1011 Nm–2, Area of cross sections a steel = 1 ×
10– m2, abrass = 2 × 10–5 m2. Both wires have same length. Then tension produced in both
wires ?
(A) if the rod remains horizontal the extensions in the wires have to be different.
(B) tension produced in both wires is same for the bar to remain horizontal
(C) distance x = 0.75m
(D) tensions produced in both wires is different for the far to remain horizontal.
Solution: (B) Elongation of both wires must be same for horizontal position of rod.
Tsteell Tbrass l
i.e. l = asteel Ysteel = abrass Ybrass

Tsteel A Y  1 10   2  10 
5 11

 steel steel
 Tbrass A brass Ybrass =
 2  10   1 10 
5 11

Join Elite Batch – Physics ( Batch -2 ) - Whatsapp -9264953797

Click Here - https://imjo.in/JKBjbX ( Batch Starting 01 March )


Final Revision Questions
2020
– Elite Batch

i.e. Tsteel =Tbrass


Taking moment Tbrass = × x Tsteel (s–x)
i.e. x = (s–x) i.e., 2x = s
s
i.e. 2x = s i.e., x = 2 = 0.75m.

12: A wire of brass and another of steel support a horizontal for as shown

B ra s s
S te e l

T T

A x B

Here s = 1.5 m, Ysteel = 2 × 1011 Nm–2, Ybrass = 1 × 1011 Nm–2, Area of cross sections a steel = 1 ×
10– m2, abrass = 2 × 10–5 m2. Both wires have same length.Then distance x is
(A) if the rod remains horizontal the extensions in the wires have to be different.
(B) tension produced in both wires is same for the bar to remain horizontal
(C) distance x = 0.75m
(D) tensions produced in both wires is different for the far to remain horizontal.
Solution: (C) Elongation of both wires must be same for horizontal position of rod.
Tsteell Tbrass l
i.e. l = asteel Ysteel = abrass Ybrass
Tsteel A steel Ysteel  1 105   2  1011 

A brass Ybrass =  2  10   1 10 
5 11
T
 brass
i.e. Tsteel =Tbrass
Taking moment Tbrass = × x Tsteel (s–x)
i.e. x = (s–x) i.e., 2x = s
s
i.e.2x = s i.e., x = 2 = 0.75m.

13: A wall of width  at an angle  with the normal is subject to water pressure in a vessel. The
height of water in the vessel is H and  is the density of water. Then Average pressure on the
wall is ?
P a t


H
P a v

Join Elite Batch – Physics ( Batch -2 ) - Whatsapp -9264953797

Click Here - https://imjo.in/JKBjbX ( Batch Starting 01 March )


Final Revision Questions
2020
– Elite Batch

1
Pat   gH
(A) Average pressure on the wall is 2
1
 Pat   gH 
(B) Average pressure on the wall is 2
1 H
 Pat   gH 
(C) Force exerted on wall is 2 sin 
 1  H
 Pat   gH 
(D) Force exerted on the wall is  2  cos 
Solution: (B) Pressure on bottom = Pat + h  g.
Pressure on top = Pat d
Pat   Pat  hg
Average pressure on bottom = 2
1
gH
Using Pascal’s law, average pressure on the wall P at + 2
Area of the wall in contact with water
h
= cos 
 1  H
 Pat  2 gH  cos 
Force exerted on wall is F = Pav A =  

14: A wall of width  at an angle  with the normal is subject to water pressure in a vessel. The
height of water in the vessel is H and  is the density of water. Then Force exerted on the
wall is ?
P a t


H
P a v

P
1
Pat   gH
(A) Average pressure on the wall is 2
1
 Pat   gH 
(B) Average pressure on the wall is 2
1 H
 Pat   gH 
(C) Force exerted on wall is 2 sin 
 1  H
 Pat   gH 
(D) Force exerted on the wall is  2  cos 
Solution: (D) Pressure on bottom = Pat + h  g.
Pressure on top = Pat d
Join Elite Batch – Physics ( Batch -2 ) - Whatsapp -9264953797

Click Here - https://imjo.in/JKBjbX ( Batch Starting 01 March )


Final Revision Questions
2020
– Elite Batch

Pat   Pat  hg


Average pressure on bottom = 2
1
gH
Using Pascal’s law, average pressure on the wall P at + 2
Area of the wall in contact with water
h
= cos 
 1  H
 Pat  2 gH  cos 
Force exerted on wall is F = Pav A =  

15: A mercury filled U–tube arrangement is connected to a bulb containing gas. Atmosphere
pressure is 1.012 × 105 Pa and H = 0.05 m. Gauge pressure at R is ?

(A) gauge pressure at R is nil


(B) gauge pressure at R is 6.56 × 103 Pa
(C) gauge pressure at R is 1.08 × 105 Pa
(D) pressure at R, Q and inside bulb are same.
Solution: (A) Pressure at R = Atmosphere pressure = 1.013 × 10 5 Pa
Gauge pressure = nil

16: A mercury filled U–tube arrangement is connected to a bulb containing gas. Atmosphere
pressure is 1.012 × 105 Pa and H = 0.05 m. Gauge pressure at Q is ?

(A) gauge pressure at Q is nil


(B) gauge pressure at Q is 6.56 × 103 Pa
(C) gauge pressure at Q is 1.08 × 105 Pa
(D) pressure at R, Q and inside bulb are same.
Solution: (B) Pressure at R = Atmosphere pressure = 1.013 × 10 5 Pa
Join Elite Batch – Physics ( Batch -2 ) - Whatsapp -9264953797

Click Here - https://imjo.in/JKBjbX ( Batch Starting 01 March )


Final Revision Questions
2020
– Elite Batch

Gauge pressure = nil


At Q, gauge pressure =  gH
= (1.36 × 103) (9.8) (0.05) = 6.564 × 103 Pa

17: A mercury filled U–tube arrangement is connected to a bulb containing gas. Atmosphere
pressure is 1.012 × 105 Pa and H = 0.05 m. Absolute pressure in bulb is ?

(A) absolute pressure in bulb is nil


(B) absolute pressure in bulb is 6.56 × 103 Pa
(C) absolute pressure in bulb is 1.08 × 105 Pa
(D) pressure at R, Q and inside bulb are same.
Solution: (C) Pressure at R = Atmosphere pressure = 1.013 × 10 5 Pa
Gauge pressure = nil
At Q, gauge pressure =  gH
= (1.36 × 103) (9.8) (0.05) = 6.564 × 103 Pa
Absolute pressure = Pat+  gH
= (1.013 × 105) + (6.564 × 103) = 1.08 × 105 Pa
 Pressure in bulb = 1.08 × 105 Pa.

18: A mercury filled U–tube arrangement is connected to a bulb containing gas. Atmosphere
pressure is 1.012 × 105 Pa and H = 0.05 m. Pressure at R, Q is ?

(A) pressure at R, Q is nil


(B) pressure at R, Q is 6.56 × 103 Pa
(C) pressure at R, Q is 1.08 × 105 Pa
(D) pressure at R, Q and inside bulb are same.
Solution: (D) Pressure at R = Atmosphere pressure = 1.013 × 10 5 Pa
Gauge pressure = nil
At Q, gauge pressure =  gH
Join Elite Batch – Physics ( Batch -2 ) - Whatsapp -9264953797

Click Here - https://imjo.in/JKBjbX ( Batch Starting 01 March )


Final Revision Questions
2020
– Elite Batch

= (1.36 × 103) (9.8) (0.05) = 6.564 × 103 Pa


Absolute pressure = Pat+  gH
= (1.013 × 105) + (6.564 × 103) = 1.08 × 105 Pa
 Pressure in bulb = 1.08 × 105 Pa.

19: Density of sea water is 1.03 × 103 kg m–3. A ship of weight 10.1 × 106 N floats on it. The ship
then enters the fresh water and some cargo is unloaded. Then volume of sea water
displaced ?
(A) volume of sea water displaced is 103 m3
(B) volume of sea water displaced 3 × 104 kg
(C) A is incorrect
(D) B is incorrect. W  s = 1 .0 3 × 1 0 3k g m -3

Solution: (A) Weight of ship = volume of sea water displaced


W
 volume of sea water displaced, V = g

10.1 106
 1 103 m3
= 1.03  10  9.8
3

Mass of volume V of fresh water


= 1 × 103 × 1 × 103 = 106 kg
Mass of cargo to be unloaded
= 1.03 × 106 –1 × 106 = 3 × 104 kg

20: Density of sea water is 1.03 × 103 kg m–3. A ship of weight 10.1 × 106 N floats on it. The ship
then enters the fresh water and some cargo is unloaded. Then Mass of volume V of fresh
water ?
(A) Mass of volume V of fresh water = 104 kg
(B) Mass of volume V of fresh water = 105 kg
(C) Mass of volume V of fresh water = 106 kg
(D) Mass of volume V of fresh water = 108 kg W  s = 1 .0 3 × 1 0 3k g m -3

Solution: (C) Weight of ship = volume of sea water displaced


W
 volume of sea water displaced, V = g

10.1 106
 1 103 m3
= 1.03  10  9.8
3

Mass of volume V of fresh water


= 1 × 103 × 1 × 103 = 106 kg
Mass of cargo to be unloaded
= 1.03 × 106 –1 × 106 = 3 × 104 kg

Join Elite Batch – Physics ( Batch -2 ) - Whatsapp -9264953797

Click Here - https://imjo.in/JKBjbX ( Batch Starting 01 March )


Final Revision Questions
2020
– Elite Batch

21: Density of sea water is 1.03 × 103 kg m–3. A ship of weight 10.1 × 106 N floats on it. The ship
then enters the fresh water and some cargo is unloaded. Then Mass of cargo to be unloaded
?
(A) Mass of cargo to be unloaded = 4 × 104 kg
(B) Mass of cargo to be unloaded = 3 × 104 kg
(C) Mass of cargo to be unloaded = 6 × 104 kg
(D) Mass of cargo to be unloaded = 8 × 104 kg W  s = 1 .0 3 × 1 0 3k g m -3

Solution: (B) Weight of ship = volume of sea water displaced


W
 volume of sea water displaced, V = g
10.1 106
 1 103 m3
= 1.03  10  9.8
3

Mass of volume V of fresh water


= 1 × 103 × 1 × 103 = 106 kg
Mass of cargo to be unloaded
= 1.03 × 106 –1 × 106 = 3 × 104 kg

22: A 4.0m long copper rod of cross sectional area 1 cm 2 is stretched by a force of 4.8103N. If
Young’s modulus for copper is Y = 1.2  1011 N/m2. Calculate Stress.
(A) 2.0  107 N/m2 (B) 4.0  107 N/m2
(C) 1.0  107 N/m2 (D) 6.0  107 N/m2

Solution : (B) When a force F is applied at the cross-sectional area A of a wire, then
F
Stress = A = 4.0  107 N/m2

23: A 4.0m long copper rod of cross sectional area 1 cm 2 is stretched by a force of 4.8103N. If
Young’s modulus for copper is Y = 1.2  1011 N/m2. Calculate strain.
(A) 1.310-4 (B) 2.310-4
(C) 3.310-4 (D) 4.310-4

Solution : (C) When a force F is applied at the cross-sectional area A of a wire, then
F
Stress = A = 4.0  107 N/m2
Stress
Young’s modulus Y = Strain
stress
 strain = Y = 3.310-4

24: A 4.0m long copper rod of cross sectional area 1 cm 2 is stretched by a force of 4.8103N. If
Young’s modulus for copper is Y = 1.2  1011 N/m2. Calculate Increase length of the wire.
(A) 0.32 mm (B) 3.32 mm
Join Elite Batch – Physics ( Batch -2 ) - Whatsapp -9264953797

Click Here - https://imjo.in/JKBjbX ( Batch Starting 01 March )


Final Revision Questions
2020
– Elite Batch

(C) 4.32 mm (D) 1.32 mm

Solution: (D) When a force F is applied at the cross-sectional area A of a wire, then
F
Stress = A = 4.0  107 N/m2
Stress
Young’s modulus Y = Strain
stress
 strain = Y = 3.310-4
increase in length
Longitudinal strain = initial length
 Increase in length = longitudinal strain  initial length
= (3.3  10-4)4.0 =13.210-4m = 1.32mm

25. A beam of metal supported at two ends is loaded at the centre. The depression at the centre
is proportional to Yn. The value of n is
(A) 1 (B) -1
(C) 2 (D) 4
WL3 1
y or y 
Solution:(B) 12Yr 4
Y

26. When a certain weight is suspended from a long uniform wire, its length increases by one
cm. If the same weight is suspended from another wire of the same material and length but
having a diameter half of the first one, the increase in length will be
(A) 0.5 cm (B) 2 cm
(C) 4 cm (D) 8 cm
Fl 1 1
y or l  ; l  2
Solution:(C) l  D
l2 D12
  4 or l2  4l1  4cm
l1 D22

27. A force F is needed to break a copper wire having radius R. The force needed to break a
copper wire of same length and radius 2R will be
(A) F/2 (B) 2F
(C) 4F (D) F/4
Solution:(C) Breaking force  cross-sectional area.
F  r2
When r is doubled, F increases four times.

28. A string 1 mm in diameter breaks if the tension in it exceeds 80 N. The maximum tension that
may be given to a similar string of diameter 2 mm is

Join Elite Batch – Physics ( Batch -2 ) - Whatsapp -9264953797

Click Here - https://imjo.in/JKBjbX ( Batch Starting 01 March )


Final Revision Questions
2020
– Elite Batch

(A) 40 N (B) 80 N
(C) 320 N (D) 1640 N
Solution:(C) Breaking force = Breaking stress  area
F  r2
When r is doubled, F increases four times.

29. A wire can sustain a weight of 10 kg before breaking. If the wire is cut into two equal parts,
then each part can sustain a weight of
(A) 2.5 kg (B) 5 kg
(C) 10 kg (D) 15 kg
Solution:(C) Maximum load is not related to length.

30. Two rods of different materials having coefficients of linear expansion 1 and 2 and Young’s
modulli, Y1 and Y2 respectively are fixed between two rigid massive walls. The rods are
heated such that they undergo the same increase in temperature. There is no bending of
1 2

rods. If  2 3 , then the thermal stresses developed in the two rods are equal, provided
Y1
Y2 is equal to
(A) 2:3 (B) 1:1
(C) 3:2 (D) 4:9
Solution:(C) Thermal strees = Yt
In the given problem,
Y  cons tan t
Y1 1 3
 
Y2  2 2

31. Two wires of the same material and length are stretched by the same force. Their masses
are in the ratio 3 : 2, Their elongations are in the ratio
(A) 3:2 (B) 9:4
(C) 2:3 (D) 4:9
Solution:(C)
Fl 1
Y or l 
l 
Again,m  l or m  
1
 l 
m
l1 m2 2
 
l2 m1 3

Join Elite Batch – Physics ( Batch -2 ) - Whatsapp -9264953797

Click Here - https://imjo.in/JKBjbX ( Batch Starting 01 March )


Final Revision Questions
2020
– Elite Batch

32. A slightly concal wire of length L and radii r 1 and r2 is stretched by two forces F, F applied
parallel to length in opposite directions and normal to end faces. If Y denotes the Young’s
modulus, then extension produced is
FL FL
(A)  r12 Y (B)  r1Y
FLY FL
(C) r1r2 (D) r1r2 Y
r1r2
Solution:(D) Mean radius is .

33. The upper end of a wire of radius 4 mm and length 100 cm is clamped and its other end is
twisted through an angle of 30. Then angle of shear is
(A) 12 (B) 0.12
(C) 1.2 (D) 0.012
r 0.4
   30  0.12
Solution:(B) l 100

34. Forces of 100 N each are applied in opposite directions on the upper and lower faces of a
cube of side 20 cm. The upper face is shifted parallel to itself by 0.25 cm. If the side of the
cube were 10 cm, then the displacement would be

(A) 0.25 cm (B) 0.5 cm


(C) 0.75 cm (D) 1 cm
Fl Fl F 1
  2  or l 
Solution:(B) Al l l ll l
Fig. 1 If l is halved, then l is doubled.

35. A copper wire 2 m long is stretched by 1 mm. If the energy stored in the stretched wire is
converted to heat, calculate the rise in temperature of the wire. (Given : Y  12  10 dyne
11

cm-2, density of copper = 9 g cm-3 and specific heat of copper = 0.1 cal g-1C-1)
(A) 252C (B) (1/252)C
(C) 1000C (D) 2000C
1 Yl2
E
Solution:(B) 2 l

Join Elite Batch – Physics ( Batch -2 ) - Whatsapp -9264953797

Click Here - https://imjo.in/JKBjbX ( Batch Starting 01 March )


Final Revision Questions
2020
– Elite Batch

m
But m  ld or  
ld
Yml2
 E
2l2d
Ym l2
Eincalorie  2
2l dJ
Ym l2 Yl2
Now, mS  or  
2l2 dJ 2l2dJS
12  1011  10 1  10 3  10 3
or 
2  2  2  9  103  4.2  0.1 103
12  105 1
  C
72  42  10 5
252

36. A 1000 kg lift is tied with metallic wires of maximum safe stress of 1.4  108 N m-2. If the
maximum acceleration of the lift is 1.2 m s-2, then the minimum diameter of the wire is
(A) 1m (B) 0.1 m
(C) 0.01 m (D) 0.001 m
Solution:(C) T  1000[9.8  1.2]N  11000N
11000  7
Maximum stress 
22  r 2min.
11000  7 502
or r 2min.  
22  1.4  108 (10 4 )2
or r 2min.  50  10 4 m  0.01 m

37. Two blocks of masses 1 kg and 2 kg are connected by a metal wire going over
a smooth pulley as shown. The breaking stress of the metal is
40
 106 N m2
3 . If g = 10 m s-2, then what should be the minimum radius of
the wire used if it is not to break?
(A) 0.5 mm (B) 1 mm
1 kg (C) 1.5 mm (D) 2 mm
2m1m2 2  1 2 40
2 kg T g  10N  N
Fig. 2 Solution:(B) m1  m2 1 2 3
If r is the minimum radius, then
40
40 40
 32 or  106 
Breaking stress r 3 3r 2

Join Elite Batch – Physics ( Batch -2 ) - Whatsapp -9264953797

Click Here - https://imjo.in/JKBjbX ( Batch Starting 01 March )


Final Revision Questions
2020
– Elite Batch

1 1
or r2  6
or r  3 m
10 10
1
or r  3  103 mm  1mm
10

38. Given :  is the compressibility of water,  is the density of water and K is the bulk modulus
of water. What is the energy density of water at the bottom of a lake ‘h’ metre deep ?
1 1
( hg)2 (hg)
(A) 2 (B) 2
1 hg hg
(C) 2  (D) 
1
u stress  strain
Solution:(A) Energy density, 2
1 stress
or u stress 
2 Bulk modulus
1
or u   compressibility  (stress)2
2

39. A body weighs 160 g in air, 130 g in water and 136 g in oil. The specific gravity of oil is
(A) 0.2 (B) 0.6
(C) 0.7 (D) 0.8
Loss of weight in oil

Solution:(D) Specific gravity of oil Loss of weight in water
160  136 24 8
    0.8
160  130 30 10

40. A diver is 10 m below the surface of water. The approximate pressure experienced by the
diver is
(A) 105 Pa (B) 2  105 Pa
(C) 3  105 Pa (D) 4  105 Pa
5
Solution:(B) 1 atmosphere = 10 Pa
Also, p= hg
= 10100010Pa = 105Pa
So, total pressure is nearly 2  105Pa

41. A penguin floats first in a fluid of density 0, then in a fluid of density 0.950 and then in a fluid
of density 1.10. Which of the following is correct ?
(A) maximum buoyant force in the fluid of density of 0.950
(B) maximum buoyant force in the case of fluid of density 1.10
(C) maximum fluid is displaced in the case of density 1.10
(D) maximum fluid is displaced in the case of density 0.950
Join Elite Batch – Physics ( Batch -2 ) - Whatsapp -9264953797

Click Here - https://imjo.in/JKBjbX ( Batch Starting 01 March )


Final Revision Questions
2020
– Elite Batch

Solution:(D) The buoyant force on the penguin is the same in all the cases. Maximum fluid is
displaced in the case of least density.

42. A piece of brass (Cu and Zn) weighs 12.9 g in air. When completely immersed in water, it
weighs 11.3 g. The relative densities of Cu and Zn are 8.9 and 7.1 respectively. The mass of
copper in the alloy is
(A) 4.6 g (B) 5.6 g
(C) 7.6 g (D) 8.6 g
Solution:(C) Loss of weight = (12.9 – 11.3) gf = 1.6 gf
Weight of water displaced = 1.6 gf
If m is the mass of Cu, then
 m 12.9  m 
 8.9  7.1  g  1.6 g
 
or 7.1m + 12.9  8.9 – 8.9m = 1.6  8.9  7.1
or 1.8m = 114.8 – 101.1
or m = 7.6 gram

43. A cubical block of wood 10 cm on a side floats at the interface between


Oil oil and water, as in Fig.3, with its lower face 2 cm below the interface.
10 cm
The density of the oil is 0.6 g cm-3. The mass of the block is
Wood
(A) 340 g (B) 680 g
Water 10 cm (C) 80 g (D) 10 g
Solution:(B) mg = [10021 + 10080.6] g
Fig.3
 m = (200 + 480) g = 680 g
1 cm2
44. A tube 1 cm2 in cross-section is attached to the top of a vessel 1 cm
high and of cross-section 100cm2. Water is poured into the system
filling it to a depth of 100cm above the bottom of the vessel as shown in
99 cm Fig.4. Take g = 10 m s-2. Now,
(A) The force exerted by the water against the bottom of
the vessel is 100 N.
1 cm
(B) The weight of water in the system is 1.99 N.
Fig.4
(C) Both (a) and (b) are correct.
(D) Neither (a) nor (b) is correct.
Solution:(c) P = 100 cm1 g cm-31000 cm s-2 = 105 dyne cm-2
F = 105100 dyne = 100 N
Again, V = 199 cm3
Weight = 19911000 dyne = 1.99 N

45. An inverted vessel (bell) lying at the bottom of a lake, 47.6 m deep has 50 cm 3 of air trapped
in it. The bell is brought to the surface of the lake. If atmospheric pressure is 70 cm of Hg, the
volume of trapped air when the vessel is brought to the surface of the lake is
(A) 100 cm3 (B) 200 cm3

Join Elite Batch – Physics ( Batch -2 ) - Whatsapp -9264953797

Click Here - https://imjo.in/JKBjbX ( Batch Starting 01 March )


Final Revision Questions
2020
– Elite Batch

(C) 300 cm3 (D) 500 cm3

Solution:(C) [47.6  102  g  70  13.6  g]50  70  13.6  gV


[4760  952]50 5712  50
or V cm3  cm3  300 cm3
952 952

1 1
and
46. When a loaded test tube floats vertically with 3 4 of the lengths inside two liquids, then
the ratio of the densities of the liquids is
(A) 3:4 (B) 4:3
(C) 9 : 16 (D) 16 : 9
l l  l 3 3
A  2g  A  2g or 1   
Solution:(A) 3 4 2 4 l 4

47. A hollow cylinder of mass m made heavy at its bottom is floating vertically in water. It is tilted
from its vertical position through an angle  and is left. The restoring force acting on it is
mg
(A) mg cos  (B) cos
 1   1 
mg   1 mg   1
(C)  cos   (D)  cos  
Solution:(C) Let l be the length of the cylinder, when vertical, in water. Let A be the cross-sectional
area of the cylinder. Equating weight of the cylinder with the upthrust, we get
mg  Alg or m  A / 
1

When the cylinder is tilted through an angle , length of cylinder in water cos 
1
 Ag
Weight of water displaced cos 
lAg
  lAg
Restoring force cos 
 1   1 
 lAg   1  mg   1
 cos    cos  

48. Energy needed in breaking a drop of radius R into n drops of radii r is given by
4
(A) 4T (nr2-R2) (B) 3  (nr3-R2)
2 2
(C) 4T (R - nr ) (D) 4T (nr2+R2)
Solution:(A) Increase in surface area = n  4 r2- 4 R2
Required energy is equal to the product of surface tension and increase in surface area.

Join Elite Batch – Physics ( Batch -2 ) - Whatsapp -9264953797

Click Here - https://imjo.in/JKBjbX ( Batch Starting 01 March )


Final Revision Questions
2020
– Elite Batch

49. The surface tension of a liquid is 5 N m-1. If a thin film is formed on a loop of area 0.02 m 2,
then its surface energy will be
(A) 5  10-2 J (B) 2.5  10-2 J
(C) 2  10 J
-1
(D) 3
Solution:(C) Effective area = 2  0.02 m2 = 0.04 m2
Surface energy = 5 N m-1 0.04 m2 = 2  10-1 J

50. Two soap bubbles, each with a radius r, coalesce in vacuum under isothermal conditions to
form a bigger bubble of radius R. Then R is equal to
(A) 2-1/2 r (B) 21/3 r
1/2
(C) 2 r (D) 2r
Solution:(C) Since conditions are isothermal, therefore, energy will be conserved.
2[2  4r2] = 2  4R2
or R2 = 2r2 or R = 21/2r

51. Two water drops, each of radius r coalesce to form a bigger drop of radius R. Then R is
equal to
1 1

(A) 2 2
r (B) 23 r
1

(C) 2 3r (D) 2r
4 4
 R 3  2   r 3 or R=21/ 3 r
Solution:(B) 3 3

52. A disc of paper of radius R has a hole of radius r. It is floating on a liquid of surface tension T.
The force of surface tension on the disc is
(A) T.2R (B) T.2 (R + r)
(C) T.4 (R + r) (D) T.2 (R - r)
Solution:(B) Effective length = 2r + 2R

53. If FC and FA denote cohesive and adhesive force on a liquid molecule near the surface of a
solid, then the surface of liquid is convex if
FC FC
FA  FA 
(A) 2 (B) 2
FC
FA 
2 FA  FC
(C) (D)
FC
FA 
Solution:(C) 2 or FC  2FA

54. Two soap bubbles, of radii 3 cm and 4 cm, coalesce in vacuum under isothermal conditions
to form a bigger bubble of radius R. Then R is equal to
Join Elite Batch – Physics ( Batch -2 ) - Whatsapp -9264953797

Click Here - https://imjo.in/JKBjbX ( Batch Starting 01 March )


Final Revision Questions
2020
– Elite Batch

(A) 3 cm (B) 4 cm
(C) 5 cm (D) 7 cm
Solution:(C) 2  4  32 + 2  4  42 = 2  4 R2
or R2 = 9 + 16 = 25 or R = 5 cm

55. Water rises to a height of 10 cm in a glass capillary tube. If the area of cross-section of the
tube is reduced to one-fourth of the former value, the water will rise to
(A) 20 cm (B) 5 cm
(C) 2.5 cm (D) 40 cm
1
h
Solution:(A) Area reduced to one-fourth. Radius reduced to one-half. Since r , therefore, h is
doubled.

56. A straw 6 cm long floats on water. The water film on one side has a surface tension of 50
dyne cm-1. On the other side, camphor reduces the surface tension to 40 dyne cm -1. The
resultant force acting on the straw is
(A) (50  6 – 40  6) dyne (B) 10 dyne
 50 40 
 6  6  dyne
(C)   (D) 90 dyne
Solution:(A) F = (1 - 2)l

57. A wire ring of diameter 14 cm is gently lowered on to a liquid surface and then pulled up.
When the film just breaks, the force required is 0.0616 N. The surface tension of the liquid is
(A) 70 N m-1 (B) 7 N m-1
-1
(C) 70 dyne cm (D) None of these
Solution:(C) 2  2 r    0.0616  10
5

6160  7
or   dyne cm 1  70 dyne cm 1
2  22  14

58. When two soap bubbles of radii r 1 and r2 (r2 > r1) coalesce, the radius of curvature of common
surface is
r2  r1
r2  r1 r1r2
(A) (B)
r1r2
r2  r1 r2  r1
(C) (D)
4 4 4 1 1 1 rr
  or   or R  1 2
Solution:(C) R r1 r2 R r1 r2 r2  r1

59. If work W is done in blowing a bubble of radius R form a soap solution, then the work done in
blowing a bubble of radius 2R form the same solution is

Join Elite Batch – Physics ( Batch -2 ) - Whatsapp -9264953797

Click Here - https://imjo.in/JKBjbX ( Batch Starting 01 March )


Final Revision Questions
2020
– Elite Batch

(A) W/2 (B) 2W


1
2 W
(C) 4W (D) 3
Solution:(C) W = 2  4R  ; R is increased by a factor of 2. So, W is increased by a factor of 4.
2

60. Water rises to a height of 2 cm in a capillary tube. If the tube is tilted 60 from the vertical,
water will rise in the tube to a length
(A) 4.0 cm (B) 2.0 cm
(C) 1.0 cm (D) water will not rise at all
Solution:(A) l cos 60 = 2 or l = 2  2 cm = 4 cm

2. SOLVED OBJECTIVE PROBLEMS

1. A flat horizontal board moves up and down in SHM of amplitude. Then the shortest
permissible time period of the vibration such that an object placed on the board may not lose
contact with the board is
g 
2 2
(A)  (B) g
1 g
(C)
2 g   (D) 2 

Solution:(B) m  mg
2

g


2 g g
 or T  2
T  

2. A uniform spring of force constant k is cut into two pieces, the lengths of which are in the
ratio 1 : 2. The ratio of the force constant of the shorter and the longer pieces is
(A) 1:2 (B) 2:3
(C) 2:1 (D) 1:3
F /  Fl F l
Y  or Y   
Solution:(C) l / l l  l  

Join Elite Batch – Physics ( Batch -2 ) - Whatsapp -9264953797

Click Here - https://imjo.in/JKBjbX ( Batch Starting 01 March )


Final Revision Questions
2020
– Elite Batch

kl
or Y or kl  cons tan t

k1 l2 k1 2
or  or 
k 2 l1 k2 1

3. The displacement of a particle moving in SHM at any instant is given by y =  sin t. The
T
t
acceleration after time 4 is (where T is the time period)
(A) 2 (B) -2
(C) - (D) 
T
t
Solution:(B) When 4 , the particle is at the extreme position. At the extreme position, the
particle has maximum acceleration -2.

4. A simple pendulum is suspended from the roof a trolley which moves in a horizontal direction
l
with an acceleration . Then the time period is given by T = 2 g , where g’ is equal to
a (A) g (B) g-

(C) g +  (D) g2   2
g g` 2

Solution:(D) g'  g2   2
or g'  g2   2
Fig.1

5. A pendulum suspended from the ceiling of a train has a period T when the train is at rest.
When the train is accelerating with a uniform acceleration, the period of oscillation will
(A) increase (B) decrease
(C) remain unaffected (D) become infinite
l 1
y  2 ; T '  2
g g2   2
Solution:(B) Comparing with
Clearly, T’<T

6. What fraction of the total energy is kinetic when the displacement is one-half of the
amplitude?
1 2
(A) 4 (B) 4
3 3.5
(C) 4 (D) 4

Join Elite Batch – Physics ( Batch -2 ) - Whatsapp -9264953797

Click Here - https://imjo.in/JKBjbX ( Batch Starting 01 March )


Final Revision Questions
2020
– Elite Batch

1 2
m2 ( 2   2 )  2

Ek  2  2 4 3
 2  
E 1  2
 2
4
m2  2
Solution:(C) 2

7. What fraction of the total energy is potential when the displacement is one-half of the
amplitude?
1 2
(A) 4 (B) 4
3 3.5
(C) 4 (D) 4
2
1 1 
Ep 2 m    2 
2 2
2   1
  2 
1
m2 2   2
E 4
Solution:(A) 2

8. At what displacement, the kinetic and potential energies are equal? Given :  = amplitude.

(A)  (B) 2
 
(C) 2 (D) 3
1 1 
m2 ( 2  2 )  m22 or  2  22 or  
Solution:(C) 2 2 2

9. What is the phase difference between velocity and displacement in SHM?



(A) 0 (B) 2

(C) 4 (D) 
Solution:(B)  =  sin t …(i)
 =  cos t
 
 t  
or  =  sin  2 …(ii)
It follows from (i) and (ii) that velocity is /2 ahead of the displacement in phase.

10. A simple pendulum with a bob of mass m swings with an angular amplitude of 40. When the
angular displacement is 20, the tension in the string is
(A) mg cos 20 (B) > mg cos 20
(C) < mg cos 20 (D) mg cos 40

Join Elite Batch – Physics ( Batch -2 ) - Whatsapp -9264953797

Click Here - https://imjo.in/JKBjbX ( Batch Starting 01 March )


Final Revision Questions
2020
– Elite Batch

Solution:(B) In this case, T- mg cos 20 = centripetal force


 T> mg cos 20

11. Two masses m1 and m2 are suspended together by a massless spring of


spring constant K. When the masses are in equilibrium, m 1 is removed without
disturbing the system. The angular frequency of m 2 is
K
K K
m1 m2
(A) (B)
m1
m2 K K
Fig.2 m1  m2 m1  m2
(C) (D)
Solution:(B) (m1+m2)g = K(l1 + l2) …(i)
After the removal of mass m1,
m2g = Kl2 …(ii)
m1g
(i) – (ii) gives m1g = Kl1 or l1= k
This gives the amplitude of oscillation of mass m2.
m2 2 K
  
K T m2
Now, T = 2

12. In Q.11, The amplitude of oscillation of m2 is


m1g m2g
(A) k (B) k
(m1  m2 )g (m1  m2 )g
(C) k (D) k
Solution:(A) (m1+m2)g = K(l1 + l2) …(i)
After the removal of mass m1,
m2g = Kl2 …(ii)
m1g
(i) – (ii) gives m1g = Kl1 or l1= k
This gives the amplitude of oscillation of mass m2.
m2 2 K
  
K T m2
Now, T = 2

13. A block of mass 0.1 kg is held between two rigid supports


by two springs of force constants 8 N m -1 and 2 N m-1. If the
block is displaced along the direction of the length of the
springs, then the frequency of vibration is
5 4
Fig.3 Hz Hz
(A)  (B) 

Join Elite Batch – Physics ( Batch -2 ) - Whatsapp -9264953797

Click Here - https://imjo.in/JKBjbX ( Batch Starting 01 March )


Final Revision Questions
2020
– Elite Batch

(C) 5  Hz (D) 4 Hz
Solution:(A) Springs are connected in parallel.
 Combined spring constant,
k = (8 + 2)Nm1  10N m1
1 k 1 10 5
v  Hz  Hz
2 m 2 0.1 

14. Two identical springs, each of spring constant K, are connected in series and parallel as
shown in fig. 4. A mass m is suspended form them. The ratio of their frequencies of vertical
oscillations will be
(A) 2 : 1 (B) 1:1
K (C) 1 : 2 (D) 4:1
K 1 K
K v
Solution:(C) 2 m
K
v K
K'
m
m In the first case,v 1 
2
(A) (B)
Fig.4 In the sec ondcase,v 2  2K
v1 1 1
   or v1 : v 2 : : 1: 2
v2 4 2

15. Refer to Fig. 5. One kg block performs vertical harmonic oscillations with amplitude 1.6 cm
and frequency 25 rad s-1. The maximum value of the force that the system exerts on the
surface is
1 kg

4.10 kg

(A) 20 N (B) 30 N
(C) 40 N (D) 60 N
Solution:(D) Weight = 5.1  9.8 N = 49.98 N = 50 N
1.6
 1  25  25N  10N
Maximum restoring force 100
Maximum force on the surface = (50 + 10) N = 60 N.

Join Elite Batch – Physics ( Batch -2 ) - Whatsapp -9264953797

Click Here - https://imjo.in/JKBjbX ( Batch Starting 01 March )


Final Revision Questions
2020
– Elite Batch

16. A small spherical heavy ball of radius r is placed on a smooth concave mirror of radius of
curvature R placed on a horizontal table. If the sphere is displaced slightly form the position
of rest, then it executes SHM with a period of
r R
2 2
(A) g (B) g
Rr Rr
2 2
(C) g (D) g
Solution:(C) The effective radius is R – r. For calculation of time period, refer to important terms,
facts and formulae.

17. A pendulum clock that keeps correct time on the Earth is taken to the Moon. It will run
(A) at correct rate (B) 6 times faster

(C) 6 times faster (D) 6 times slower.

Solution:(D) g decreases by a factor of 6. T increases by a factor of 6 . So, the clock is 6 times


slower.

18. A horizontal platform with an object placed on it is executing SHM in the vertical direction.
The amplitude of oscillation is 2.5 cm. What must be the least period of these oscillation so
that the object is not detached ? (Given : g = 10 m s -2)

s
(A) s (B) 5
 
s s
(C) 10 (D) 15
g
m 2a  mg or  
Solution:(C) a
T a a 2.5 
or  or T  2  2  s
2 g g 1000 10

19. A forced oscillator is acted upon by a force F = F 0 sin t. The amplitude of the oscillator is
55
given by 2  36  9 . What is the resonant angular frequency ?
2

(A) 2 units (B) 9 units


(C) 18 units (D) 36 units
Solution:(B) At resonance, amplitude should be maximum.
This is possible if 22 - 36 + 9 is minimum.
d
 [22  36  9]  0
d
or 4  36  0 or   9 units

Join Elite Batch – Physics ( Batch -2 ) - Whatsapp -9264953797

Click Here - https://imjo.in/JKBjbX ( Batch Starting 01 March )


Final Revision Questions
2020
– Elite Batch

20. Two simple harmonic motions are represented by :




π

y1 = 10 sin  4πt +  and y 2 = 5 sin 4 πt + 3cos 4πt
4

. The ratio of the amplitudes of two SHM
is
(A) 1:1 (B) 1:2

(C) 2:1 (D) 1: 3

Solution:(A) In the second case, amplitude is 52 + (5 3 )2

i.e. 25 + 75 i.e. 100 or10units.


So, the ratio of amplitudes is 1 : 1.

21. The vertical extension in a light spring by a weight of 1 kg is 9.8 cm. The period of oscillation
is
 2
second second
(A) 5 (B) 5
 5
second second
(C) 10 (D) 
1 9.8 2 
T = 2  2 s s s
Solution:(A) g 980 10 5

22. The period of a simple pendulum, whose bob is a hollow metallic sphere, is T. The period is
T1 when the bob is filled with sand, T 2 when it is filled with mercury and T 3 when it is half filled
with sand. Which of the following is true?
(A) T = T1 = T2>T2 (B) T1 = T2 > T3>T
(C) T > T3 > T1=T2 (D) T = T1 = T2<T3
Solution:(D) In the following cases, the location of centre of gravity is same.
(i) hollow sphere
(ii) sphere filled with sand
(iii) sphere filled with mercury.
So, effective value of l is same and hence T is same. In the case of half-filled sphere, the
C.G. is below the geometrical centre. So, l and hence T increases.

23. What is the spring constant for the combination of springs shown in Fig.9?
(A) k (B) 2k
2K
5k
(C) 4k (D) 2
m Solution:(C) Treat the given system as a parallel combination of
springs.

K K

Join Elite Batch – Physics ( Batch -2 ) - Whatsapp -9264953797


Fig.9

Click Here - https://imjo.in/JKBjbX ( Batch Starting 01 March )


Final Revision Questions
2020
– Elite Batch

24. A particle executes simple harmonic motion under the restoring force provided by a spring.
The time period is T. If the spring is divided in two equal parts and one part is used to
continue the simple harmonic motion, the time period will
(A) remain T (B) become 2T
(C) become T/2 (D) become T/ 2
Solution:(D) k si doubled.
m m
T = 2 ; T '  2
k 2k
T' 1 T
 or T ' 
T 2 2

25. A mass M is suspended by two springs A and B of force constants k1 and k2


respectively as shown in the Fig. 10. The total stretch of springs is
A K1 Mg Mg( k1  k 2 )
(A) k1  k 2 (B) k1k 2
Mgk1  k 2 k1  k 2
B K2 (C) k1  k 2 (D) k1k 2 Mg
Mg

M Solution:(B) Extension of first spring k1 ; Extension of second
Fig.10 Mg

spring k 2 .
Mg Mg  k  k2 
   Mg  1 
k1 k2  k1k 2 
Net extension

26. Three masses of 500 g, 300 g and 100 g are suspended at the end of a
spring as shown, and are in equilibrium. When the 500 g mass is
removed, the system oscillates with a period of 2 second. When the 300 g
mass is also removed, it will oscillate with a period of
(A) 2s (B) 4s
(C) 8s (D) 1s
400 100
500 g T  2 ; T '  2
300 g Solution:(D) k k
100 g
T' 100 1 T 2
Fig.11   or T '   s  1s
T 400 2 2 2

Join Elite Batch – Physics ( Batch -2 ) - Whatsapp -9264953797

Click Here - https://imjo.in/JKBjbX ( Batch Starting 01 March )


Final Revision Questions
2020
– Elite Batch

27. If the mass shown in Fig.12 is slightly displaced and then let go, then the
system shall oscillate with a time period of
K
m 3m
2 2
(A) 3k (B) 2k
2m 3k
2 2
K K (C) 3k (D) m
Solution:(B) The equivalent situation is a series combination of
two spring of spring constants k and 2k.
m If k’ is the equivalent spring constant, then
Fig.12
( k )(2k ) 2k
k'  
3k 3
3m
T  2
2k

28. When the potential energy of a particle executing simple harmonic motion is one-fourth of its
maximum value during the oscillation, the displacement of the particle from the equilibrium
position in terms of its amplitude a is
(A) a/3 (B) a/4
(C) 2a/3 (D) a/2
1 1 a 22
E P  m22 or E P  m2 2
Solution:(D) 2 2 a
2 2 a 
2 2
1 1 2 2 1
Now  m a   m a
42  2 a2
a2 a
or 2  or   
4 2

29. A block of mass m, attached to a spring of spring constant k, oscillates on a smooth


horizontal table. The other end of the spring is fixed to a well. The block has a speed  when
the spring is at its natural length. Before coming to an instantaneous rest, if the block moves
a distance x from the mean position, then

(A) x  m / k (B) x m/k


1
x m/k
(C)  (D) x   m/k
1 2 1 2 m
m  kx or x  
Solution:(D) 2 2 k

Join Elite Batch – Physics ( Batch -2 ) - Whatsapp -9264953797

Click Here - https://imjo.in/JKBjbX ( Batch Starting 01 March )


Final Revision Questions
2020
– Elite Batch

30. A 1 kg block is executing simple harmonic motion of amplitude 0.1


m on a smooth horizontal surface under the restoring force of a
spring of spring constant 100 N/m. A block of mass 3 kg is gently
placed on it at the instant it passes through the mean position.
Assuming that the two blocks move together, the amplitude of the
motion is
Fig.13 (A) 1 cm (B) 2 cm
(C) 3 cm (D) 5 cm
Solution:(D) Gentle Placing. No external force. Apply conservation servation of momentum.
Now, m  a  ( m  m ')a '  '
k k
or ma  ( m  m ')a '
m m  m'
100 1
or 1 0.1  (1  3)a '  m or a '  5 cm
1 20

31. A particle moves on the X-axis according to the equation x = A + B sin t. The motion is
simple harmonic with amplitude
(A) A (B) B

(C) A+B (D) A2  B2


Solution:(B) Compare with  = 0 + a sin t

32. A block of mass m is suspended by different springs of force constant shown in


figure. Let time period of oscillation in these four positions be T 1, T2, T3 and T4.
Then

2k

2k

(iv)
(A) T1 = T2 = T4 (B) T1 = T2 and T3 = T4
(C) T1 = T2 = T3 (D) T1 = T3 and T2 = T4
Solution: (B) Effective force constant in case (iii) and (iv) is
keff = 2k + 2k = 4k
m m
2 
Therefore, T1 = T2 = k k

-β  x - 2 
33. A particle moves such that its acceleration is given by : a =

Join Elite Batch – Physics ( Batch -2 ) - Whatsapp -9264953797

Click Here - https://imjo.in/JKBjbX ( Batch Starting 01 March )


Final Revision Questions
2020
– Elite Batch

Here  is a positive constant and x the position from origin. Time period of
oscillations is
1
2
(A)
2  (B) 
1
2
(C)
2   2 (D)  2
Solution: (B) a = o at x = 2 i.e. x = 2 is the mean position.
Assuming x–2= X
a=  X
i.e. a  –X
oscillations are simple harmonic. Time period of which will be given by :
X ml
2  2
 T= a YA

34. A wire of length l, area of cross section A and Young’s modulus of


elasticity Y is suspended from the roof of a building. A block of mass m is
attached at lower end of the wire. If the block is displaced from its mean
position and then released the block starts oscillating. Time period of these
oscillations will be
Al AY
2 2
(A) mY (B) ml
ml m
2 2
(C) YA (D) YAl
YA
Solution: (C) Force constant of a wire is : k = l
m ml
2  2
 T= k YA

35. Let T1 and T2 be the time periods of two springs A and B when a mass m is
suspended from them separately. Now both the springs are connected in
parallel and same mass m is suspended with them. Now let T be the time period
in this position. Then
T1T2
(A) T=T+T 1 2 (B) T = T1  T2
1 1 1
 2 2
(C) T2 =
T T
1
2
2
2
(D) T 2
T1 T2

Join Elite Batch – Physics ( Batch -2 ) - Whatsapp -9264953797

Click Here - https://imjo.in/JKBjbX ( Batch Starting 01 March )


Final Revision Questions
2020
– Elite Batch

m 42m
2 2
k1
Solution: (D) T1= or k1= T1
m 42m
2 2
k2
T2 = or k = T2
2

m 42m
2 2
k2
Now T = or k = T2
In parallel k = k1 + k2
Substituting the values of k, k1 and k2 we get :
1 1 1
2
 2 2
T T1 T2

36. When a particle oscillates simple harmonically, its kinetic energy varies
periodically. If frequency of the particle is ‘n’, the frequency of the kinetic energy
is
(A) n/2 (B) n
(C) 2n (D) 4n
mω2 2 2 mω2 2
(A −y ) [ A −( A sin (ωt+ φ ))2 ]
Solution: K.E. = 2 = 2
2

[ A2 − A2 sin2 (ωt +φ )]
= 2
mω2 A 2
[ cos2 ( ωt+ φ)]
= 2
mω2 A 2
[1+cos (2 ωt+ 2 φ)]
= 4

 Frequency = 2 π = 2n

37. An accurate pendulum clock is mounted on the ground floor of high building.
How much time will it lose or gain in one day if it is transferred to top storey of
a building which is h = 200m higher than the ground floor. Radius of earth is 6.4
× 106m.
(A) it will lose 6.2s (B) it will lose 2.7s
(C) it will gain 5.2s (D) it will gain 1.6s
l 1
2
or T 
g g
Solution: (B) T =
T' g

 T g'

Join Elite Batch – Physics ( Batch -2 ) - Whatsapp -9264953797

Click Here - https://imjo.in/JKBjbX ( Batch Starting 01 March )


Final Revision Questions
2020
– Elite Batch

g
2
 h
1 R 
But g’ =  
1
g'  h
2
 1 
Or g  R

T'  h
 1 
 T  R
 h
T 1 
or T’ =  R
Since T’>T, the clock will lose the time.
h
T ' T  T  
 T = R
 Time lost in t = 1 day is
 t 
 t
t   T' 
t  h / R h
 t 
=  1 h / R   R 
 24  3600   200 
= 6.4  106 s = 2.7 s

38. A pendulum has time period T for small oscillations. An


obstacle P is situated below the point of suspension O at
3l
.
a distance 4 The pendulum is released from rest.
Throughout the motion the moving string makes small
angle with vertical. Time after which the pendulum returns
back to its initial position is
3T
(A) T (B) 4
3T 4T
(C) 5 (D) 5
l
Solution: (B) After P length of pendulum becomes 4

Now as T  l , so after P time period will become T’ = T/2, Therefore, the desired
time will be :

Join Elite Batch – Physics ( Batch -2 ) - Whatsapp -9264953797

Click Here - https://imjo.in/JKBjbX ( Batch Starting 01 March )


Final Revision Questions
2020
– Elite Batch

T T ' T T 3T
   
t = 2 2 2 4 4

39. The two block of mass m 1 and m2 are kept on a smooth horizontal table as
shown in figure. Block of mass m 1 but not m2 is fastened
to the spring. If now both the blocks are pushed to the
m1 m left so that the spring is compressed a distance d. The
2
amplitude of oscillation of block of mass m1, after the
system is released is
m1 m2
d d
m1  m2 m1  m2
(A) (B)
2m2 2m1
d d
m1  m2 m1  m2
(C) (D)
Solution: (A) Block of mass m2 shoots off carrying some kinetic energy away from the system.
To find its speed. Potential energy of spring = maximum kinetic energy of blocks.

kd2 v2
  m1  m2 
 2 4
(k = force constant of spring)
kd2
or v2 = m1  m2
with m1 alone on the spring :
maximum potential energy = maximum kinetic energy of m 1.
1 2 1
kA  m1v 2
or 2 2
km1d2
or KA2 = m1  m2
m1
d
m2  m 2
or A=

40. A particle starts SHM from the mean position. It’s amplitude is a and total
3E
energy E. At one instant its kinetic energy is 4 its displacement at this
instant is
a a
(A) y = √2 (B) y= 2

Join Elite Batch – Physics ( Batch -2 ) - Whatsapp -9264953797

Click Here - https://imjo.in/JKBjbX ( Batch Starting 01 March )


Final Revision Questions
2020
– Elite Batch

a
(C) y = √3 / 2 (D) y=a
Solution: y = A sin t
1
mω2 A 2
E= 2
1 3 1
mω2 ( A2 − y 2 ) mω2 A 2
K.E. = 2 = 4 2
3
 A2  y 2 = 4 A2
2
A A
= y2
4  y= 2

41. A particle executing SHM while moving from one extremity is found at distance
x1, x2, and x3, from the centre at the end of three successive seconds. The time
period of oscillation is
2 
(A) θ (B) 

(C)  (D) 2θ
Solution: (B) Displacement time equation of the particle will be
x = A cos t
Given that x1 = A cos 
x2 = A cos 2 
and x3 = A cos 3 
x1+ x 2 A  cos   cos3
Now 2x 2 = 2A cos 2
2A cos 2 cos 
 cos 
= 2A cos 2
 x  x3  2
cos1  1 
   2x 2  T
2  x  x3 
  cos1  1 
or T=  where  2x 2 

42. Two masses M and m are suspended together by a massless spring of force
constant k. When the masses are in equilibrium, M is removed without
disturbing the system. The amplitude of oscillation is
Mg mg
(A) k (B) k

Join Elite Batch – Physics ( Batch -2 ) - Whatsapp -9264953797

Click Here - https://imjo.in/JKBjbX ( Batch Starting 01 March )


Final Revision Questions
2020
– Elite Batch

 M  m g  M - m g
(C) k (D) k
Solution: (D) For equilibrium of (M + m)
 M  m g
X1 = k
and for equilibrium of m
mg
x2 = k
 amplitude of oscillation will be
Mg
A = x 1 – x2 = k
43. A block is kept on a rough horizontal plank. The coefficient of friction between
the block and the plank is 1/2. The plank is undergoing SHM of angular
frequency 10 rad/s. The maximum amplitude of plank in which the block does
not slip over the plank is : (g = 10 m/s2)
(A) 4cm (B) 5cm
(C) 10 cm (D) 16 cm
Solution: (B) Maximum acceleration in SHM is
amax =  A = g
2

this will be provided to the block by friction.


Hence amax = g or  A = g
2

 1
   10 
2
g
 10 
2

or A= 2 =
= 0.05 m = 5 cm

44. A particle executes SHM between x = -A and x = +A. The time taken for it to go
from 0 to A/2 is T1 and to go from A/2 to A is T2. Then
(A) T1 < T2 (B) T1 > T2
(C) T1 = T2 (D) T1 = 2T2
Solution: Let x = A sin t
From 0 – A/2
 A/2 = A sin T1
1 π
or sin T1 = 2 = sin 6
2π π T
. T1=
 T 6 or T1 = 12
From 0 – A
A = A sin T’1
Join Elite Batch – Physics ( Batch -2 ) - Whatsapp -9264953797

Click Here - https://imjo.in/JKBjbX ( Batch Starting 01 March )


Final Revision Questions
2020
– Elite Batch

or, sin T’1 =1 = sin ( / 2)


2π ' π T
.T =
or, T 1 2 or T’1 = 4
 From A/2 – A :
T T T

T2 = T’1  T1 = 4 12 = 6
T T
∵ T1 = 12 and T2 = 6
 T1 < T2

45. A simple pendulum of length 1m with a bob of mass m swings with an angular
amplitude 30o. Then tension in the string at angular displacement 15 o: (g = 9.8
m/s2)
(A) greater than mg sin15oC
(B) greater than mg cos15oC
(C) greater than mg cos75oC
(D) greater than mg cot75oC

46. A simple pendulum of length 1m with a bob of mass m swings with an angular
amplitude 30o. Then rate of change of speed at angular displacement 15 o: (g =
9.8 m/s2)
(A) g cos 15o
(B) g cot 25o
(C) g sin 15o
(D) g sin 75o
Solution: (45-46) (B,C) Simple pendulum of length 1 m is called second’s pendulum whose time
period is T = 2s. But this time period is for small oscillations. In this case angular
amplitude is 30o. Therefore, time period will not be 2s.
At angular displacement 15o.
mv 2
T – mg cos15o = l
Or T > mg cos15o.
And tangential acceleration at = g sin 15o = rate of change of speed.

47. A constant force F is applied on a spring block system as shown


in figure. The mass of the block is m and spring constant is k.
F k
m The block is placed over a smooth surface. Initially the spring
was unstretched. Then motion is which execute type?
(A) SHM (B) Circular
(C) straight line (D) rotation

Join Elite Batch – Physics ( Batch -2 ) - Whatsapp -9264953797

Click Here - https://imjo.in/JKBjbX ( Batch Starting 01 March )


Final Revision Questions
2020
– Elite Batch

48. A constant force F is applied on a spring block system as shown


in figure. The mass of the block is m and spring constant is k.
F k
m The block is placed over a smooth surface. Initially the spring
was unstretched. Then what is time period of oscillation?
m m
 2 5
(A) k (B) k
m k
2 2
(C) k (D) m

49. A constant force F is applied on a spring block system as shown


in figure. The mass of the block is m and spring constant is k.
F k
m The block is placed over a smooth surface. Initially the spring
was unstretched. Then what is the maximum speed of block?
Fx  kx 2 2Fx  kx 2
(A) m (B) m
2Fx  mx 2
2Fx  kx 2
(C) k (D) m

Solution: (47-49)(A,C,D) The situation is similar as if a block of mass m is suspended from a


vertical spring and a constant force mg acts downwards. Therefore, in this case also block
will execute SHM with time period:
m
2
T= k
At compression x where
F = kx
F
X= k
This is also the amplitude of oscillation.
Hence
F
A= k
At mean position speed of the block will be maximum. Applying work energy theorem
1 2 1
kx  mv 2
F. x = 2 2
2Fx  kx 2
or v = m

Join Elite Batch – Physics ( Batch -2 ) - Whatsapp -9264953797

Click Here - https://imjo.in/JKBjbX ( Batch Starting 01 March )


Final Revision Questions
2020
– Elite Batch

50: If a SHM is represented by the equation


x = 10sin [ p t + p /6 ] in S.I. Units Determine its
amplitude?
(A) 10 m (B) 1m
(C) 6m (D) 14 m

51: If a SHM is represented by the equation


x = 10sin [ p t + p /6 ] in S.I. Units Determine its
time period?
(A) 6s (B) 2s
(C) 10 s (D) 3s

52: If a SHM is represented by the equation


x = 10sin [ p t + p /6 ] in S.I. Units Determine its
maximum velocity Q?
(A) 1 m /s (B) 5 m / s
(C) 10  m / s (D) 50  m / s
Solution 1:(50-52)(A, B, C) Comparing the above equation with
x = A sin ( wt + f 0 ) ,
we get,
A = 10 m
w = p - 1s and f 0 = p /6
 T = 2p / w
 T  2s
v max = w A = 10 p m / s.

53. The spring mass system is shown in the figure. The


spring stretches 2 cm from its free length when a force of
10 N is applied. This spring is stretched 10 cm from its
free length, a body of mass m = 2kg attached to it and
released from rest at time t = 0. Find the force constant
of the spring.
Write the equation of motion of the body in the from x = A sin (  + ) where x is the
displacement from the equilibrium position. Express the spring force as a
function of time.
(A) 500 N/m (B) 800 N/m
(C) 300 N/m (D) 100 N/m

54. The spring mass system is shown in the figure. The


spring stretches 2 cm from its free length when a Free length
k
force of 10 N is applied. This spring is stretched 10
cm from its free length, a body of mass m = 2kg
m
attached to it and released from rest at time t = 0.
Find the time period

Join Elite Batch – Physics ( Batch -2 ) - Whatsapp -9264953797

Click Here - https://imjo.in/JKBjbX ( Batch Starting 01 March )


Final Revision Questions
2020
– Elite Batch

Write the equation of motion of the body in the from x = A sin (  + ) where x is the
displacement from the equilibrium position. Express the spring force as a
function of time.
(A) 0.307 s (B) 0.397 s
(C) 0.297 s (D) 5.300 s

55. The spring mass system is shown in the figure. The


spring stretches 2 cm from its free length when a Free length
force of 10 N is applied. This spring is stretched 10 k
cm from its free length, a body of mass m = 2kg
m
attached to it and released from rest at time t = 0.
Find the frequency of vibration
Write the equation of motion of the body in the from x = A sin (  + ) where x is the
displacement from the equilibrium position. Express the spring force as a
function of time.
(A) 7.20 Hz (B) 5.51 Hz
(C) 2.51 Hz (D) 0.55 Hz

56. The spring mass system is shown in the figure. The spring
stretches 2 cm from its free length when a force of 10 N is Free length
applied. This spring is stretched 10 cm from its free k
length, a body of mass m = 2kg attached to it and released
m
from rest at time t = 0. Find the amplitude of vibration.
Write the equation of motion of the body in the from x = A sin (  + ) where x is the
displacement from the equilibrium position. Express the spring force as a
function of time.
(A) 3.06 m (B) 1.00 m
(C) 0.06 m (D) 8.03 m

57. The spring mass system is shown in the figure. The spring
stretches 2 cm from its free length when a force of 10 N is Free length
applied. This spring is stretched 10 cm from its free k
length, a body of mass m = 2kg attached to it and released
m
from rest at time t = 0. Find the initial velocity and
acceleration
Write the equation of motion of the body in the from x = A sin (  + ) where x is the
displacement from the equilibrium position. Express the spring force as a
function of time.
(A) 1, 1 m/s2 upward (B) 2, 7 m/s2 upward
(C) 3, 10 m/s2 upward (D) 0, 15 m/s2 upward

Join Elite Batch – Physics ( Batch -2 ) - Whatsapp -9264953797

Click Here - https://imjo.in/JKBjbX ( Batch Starting 01 March )


Final Revision Questions
2020
– Elite Batch

58. The spring mass system is shown in the figure. The spring
stretches 2 cm from its free length when a force of 10 N is Free length
applied. This spring is stretched 10 cm from its free k
length, a body of mass m = 2kg attached to it and released
m
from rest at time t = 0. Find the maximum velocity and
acceleration
Write the equation of motion of the body in the from x = A sin (  + ) where x is the
displacement from the equilibrium position. Express the spring force as a
function of time.
(A) 0.95 m/s, 15 m /s2 (B) 0.95 m/s , 25 m /s2
(C) 0.95 m/s , 5 m /s2 (D) 0.95 m/s , 2 m /s2

59. The spring mass system is shown in the figure. The spring
stretches 2 cm from its free length when a force of 10 N is Free length
applied. This spring is stretched 10 cm from its free k
length, a body of mass m = 2kg attached to it and released
m
from rest at time t = 0. Find the spring force at the two
extreme position of the body.
Write the equation of motion of the body in the from x = A sin (  + ) where x is the
displacement from the equilibrium position. Express the spring force as a
function of time.
(A) x = 00.06 sin (-t + /2) (B) x = 00.06 sin (t + /2)
(C) x = 00.06 sin (t - /2) (D) x = 00.01 sin (t + /2)

60. The spring mass system is shown in the figure. The spring
stretches 2 cm from its free length when a force of 10 N is Free length
applied. This spring is stretched 10 cm from its free length, a k
body of mass m = 2kg attached to it and released from rest at
m
time t = 0. Find the time taken by the body to move half way
towards the equilibrium position from its initial position.
Write the equation of motion of the body in the from x = A sin (  + ) where x is the
displacement from the equilibrium position. Express the spring force as a
function of time.
(A) 20 - 10 cos t N (B) 20 + 30 sin t N
(C) 20 + 30 cos t N (D) 50 - 30 cos t N

Join Elite Batch – Physics ( Batch -2 ) - Whatsapp -9264953797

Click Here - https://imjo.in/JKBjbX ( Batch Starting 01 March )


Final Revision Questions
2020
– Elite Batch

10 N
Free length
Solution 7: 53(a) k = 0. 02m = 500 N/m k
m 2
54(b) time period T = 2 √ 1
k =2 √ 500 =0.397 s Equilibrium
position
0

x
i

55(c)frequency f = T = 2.51 Hz Initial position


angular frequency= 15.8 rad/s
 = 2f
56(c) In equilibrium position acceleration = 0
k0 – mg = 0
mg 2(10 )
= =0. 04
0 = k 500 m
amplitude = maximum displacement from the equilibrium position
A = I  0 = 0.10 – 0.04 = 0.06 m.

57(d) Initial velocity = 0 (given)


kδi −mg 500(0 .1 )−2(10 )
Initial acceleration = m = 2 = 15 m/s2 upward.
58(a) Maximum velocity =A
= 0.06  15.8 = 0.95 m/s
Maximum acceleration = A2 = 15 m /s2

59(b) At one extreme position


spring force Fs = kI = 500 (0.1) = 50 N
At the other extreme position
Fs = k(A - 0) = 500 (0.06 – 0.04) = 10 N compression.
The equation of motion in SHM is
x = A sin (t + )
 dx/dt = A  cos (t + )
initial condition t = 0, x = 10 – 4 = 6 cm = 0.06 m.
dx
=0
dt
A sin  = 0.06
And A  cos  = 0
π
  = 2 and A = 0.06 m
 x = 00.06 sin (t + /2)
60(c) At the given position x = A/2 = 0.03 m
 0.03 = 0.06 sin (t + /2)
t = /3  t = /3 = 0.066 s
The instantaneous velocity

Join Elite Batch – Physics ( Batch -2 ) - Whatsapp -9264953797

Click Here - https://imjo.in/JKBjbX ( Batch Starting 01 March )


Final Revision Questions
2020
– Elite Batch

v= √ A 2−x2 = √ A 2−A 2/ 4


√3
= A 2 = 0.82 m/s upward
or v = A cos (t + /2)
= A  cos (/3 + /2)
√3
= A  cos (5/3) =  A 2 = 0.82 m/s upward]
Instantaneous acceleration
a = - 2x= (A/2) 2 = 7.5 m/s2 i.e. 7.5 m/s2 upwards
spring force Fs = k (0 + x)
= mg + kA sin (t + )
= 20 + 30 sin (t + /2)
= 20 + 30 cos t N

--
ASSIGNMENT
 
 50  t + 2 
1. A wave disturbance in a medium is described by y(x, t) = 0.02 cos   cos(10x)
where x and y are in metre and t is in second:
(A) A node occurs at x = 0.15 m (B) An antinode occurs at x = 0.3m
-1
(C) The speed wave is 5 ms (D) The wave length is 0.3 m
Solution: (C)
Comparing it with y(x, t) = A cos(t + /2)cos kx.
If kx = /2, a node occurs; 10x = /2  x = 0.05m
If kx = , an antinode occurs  10x = x  x = 0.1 m
50
 5m / s
Also speed of wave = /k = 10 and  = 2/k = 2/10= 0.2

2. The extension in a string, obeying Hooke’s law, is x. The speed of sound in the stretched
string is v. If the extension in the string is increased to 1.5x, the speed of sound will be
(A) 1.22v (B) 0.61v
(C) 1.50v (D) 0.75v
Solution: (A)
According to Hooke’s law FR  x [Restoring Force FR =T, tension of ring]
Velocity of sound by a stretched string
T
v
m where m is the mass per unit length

Join Elite Batch – Physics ( Batch -2 ) - Whatsapp -9264953797

Click Here - https://imjo.in/JKBjbX ( Batch Starting 01 March )


Final Revision Questions
2020
– Elite Batch

v T T 1.5x
 v  v v  1.22v
 v T  T x

3. A whistle giving out 450 Hz approaches a stationary observer at a speed of 33 m/s. The
frequency heard by the observer in Hz is
(A) 409 (B) 429
(C) 517 (D) 500
Solution: (D)
33m / s V0  0
   
VS 0
(Source) (Observer)
 v   330 
      450    500Hz
 v  vs   330  33 

4. A travelling wave in a stretched string is described by the equation y = A sin (kx – t). The
maximum particle velocity is
(A) A (B) /k
(C) d/dk (D) x/t
Solution: (A)
dy
V   A cos(kx  t)
dt
 Vmax = A

5. A siren placed at a railway platform is emitting sound of frequency 5kHz. A passenger sitting
in a moving train A records a frequency of 5.5 kHz while the train approaches the siren.
During his return journey in a different train B he records a frequency of 6.0 kHz while
approaching the same siren. The ratio of the velocity of train B to that train A is
(A) 242/252 (B) 2
(C) 5/6 (D) 11/6
Solution: (B)
VA  V 5.5 VB  V 6
VB
  2
V 5 and V 5  VA

6. An organ pipe P1 closed at one end vibrating in its first harmonic and another pipe P 2 open at
ends vibrating in its third harmonic are in resonance with a given tuning fork. The ratio of the
length of P1 to that of P2 is:
(A) 8/3 (B) 3/8
(C) 1/6 (D) 1/3
Solution: (C)

Join Elite Batch – Physics ( Batch -2 ) - Whatsapp -9264953797

Click Here - https://imjo.in/JKBjbX ( Batch Starting 01 March )


Final Revision Questions
2020
– Elite Batch

 

v
41 for first harmonic
3
v
22 for third harmonic
 3 1 1
 
 41 22  2 6

7. Two whistles A and B have frequencies 600Hz and 590Hz reactively. An observer is
standing in the middle of the line joining the two sources. Source B and observer are moving
towards right with velocity 30m/s and A is standing at left side. If the velocity of sound in air is
300m/s, the number of beats listened by the observer are:
(A) 2 (B) 4
(C) 6 (D) 8
Solution: (B) Frequency of A as heard by the observer
300−30
N1 = 300 660 = 594Hz.
Frequency of B as heard by observer = 594Hz, as there is no relative motion between source
and observer.
 Beats frequency = (594  590)= 4 per/s

8. A cylindrical tube, open at both ends has a fundamental frequency 'f' in air. The tube is
vertically dipped in water so that half of it is in water, the fundamental frequency of the air
column is
(A) f/2 (B) 3f/4
(C) f (D) 2f
Solution: (C) For open tube f = f = v/2. On dipping the tube in water, it becomes a closed tube.
v ℓ v
For closed tube f = 2 ℓ
' ()
=v / 4
2 = 2ℓ = f

9. The amplitude of a wave disturbance propagating in the positive x-direction is given by y =1/ (1 +
x2) at time t = 0 and by y = 1/[1 + (x – 1) 2 at t = 2 seconds, where x and y are in metres. The
shape of the wave disturbance does not change during the propagation. The velocity of the wave
is
(A) 1 ms1 (B) 0.5 ms1
(C) 1.5 ms1 (D) 2 ms1
Solution: (B) Writing the general expression for y in terms of x as

Join Elite Batch – Physics ( Batch -2 ) - Whatsapp -9264953797

Click Here - https://imjo.in/JKBjbX ( Batch Starting 01 March )


Final Revision Questions
2020
– Elite Batch

1
y= 1+( x−vt )2 at t = 0, y = 1/ (1 + x)2
1
2
At t = 2 s, y = 1+[ x−v (2)]
Comparing with the given equation we get 2v = 1 and v = 0.5 m/s.

10. A train has just completed a U-curve in a track which is a semicircle. The engine is at forward
end of the semicircular part of the track while the last carriage is at the rear end of the semi-
circular track. The driver blows a whistle of frequency 200Hz. Velocity of sound is 340 m/s.
Then the apparent frequency as observed by a passenger in the middle of the train, when
the need of the train is 30 m/s is
(A) 219Hz (B) 288 Hz
(C) 200Hz (D) 181Hz
Solution: (C) Velocity component of the source in the direction
of motion of sound = 30 cos45 along BA.
Velocity component of observer in the direction Passenger
B r

BA = 30 cos45. r
 There is no relative motion between the source and 45
A
the observer, hence no change in real frequency is Driver
observed.

11. Two waves represented by y1 =10 sin(2000 t+2x) and y2=10 sin (2000  t +2x+/2) are
superposed at any point at a particular instant. The resultant amplitude is:
(A) 10 units (B) 20 units
(C) 14.1 units (D) zero
Solution: (C) The resultant amplitude A of two waves of amplitudes a 1 and a2 at a phase
2 2 1/2
difference  is ( (a1 +a2 +2 a 1 a2 cos φ)
Substituting
a1 = 10, a2 = 10 and  = 900, we get A = 14.1.

12. A transverse wave is described by the equation y = y 0 sin 2 (ft – x/a). The maximum
particle velocity is equal to four times the wave velocity if a is equal to
(A) y0 / 4 (B) y0/2
(C) y0 (D) 2y0
Solution: (B) The maximum particle velocity of a SHM of amplitude Y 0 and frequency f is 2fY0. The
wave velocity is f. For 2 fY0 to be equal to 4f,  has to be Y0 / 2
(Here  = a).

13. A standing wave having three antinodes and four nodes is formed between two atoms having
a distance of 2A between them. The wavelength of the standing wave is
(A) 1.33A (B) 6 A
(C) 1.4 A (D) 8 A
Join Elite Batch – Physics ( Batch -2 ) - Whatsapp -9264953797

Click Here - https://imjo.in/JKBjbX ( Batch Starting 01 March )


Final Revision Questions
2020
– Elite Batch

Solution: (A) A A A

3/2 = 2A
 = (4/3)A = 1.33 A N N N N

14. The speed of sound through oxygen at T K is (300 m/s). When the temperature is increased
to 3T, the molecule dissociates into oxygen atom, now the speed of sound will be
(A) 520 m/s (B) 801 m/s
(C) 600 m/s (D) 580 m/s
γ RT 7
Solution: (B) vT = √ M
=

5
RT / 32×10−3
= 300 m/s
5
v3T = √ 3
R 3 T /16×10−3

v 3T 5 R3T 32×10−3 ×5 50
vT √
=
3 16×10 −3
×
7 RT
v3T = 300  2.67 = 801 m/s
=
7 √ = 2.67

15. Standing wave is formed in string between two fixed ends. If separation between two
consecutive antinode is 2 mm and length of string is 9 cm. Then harmonics in which string is
vibrating
(A) 2nd harmonics (B) 9th harmonics
th
(C) 45 harmonics (D) 90th harmonics.
Solution: (C) If there exits pth harmonic
P 2 = 90
 P = 45

16. A source of sound emitting a note of frequency 200Hz moves towards an observer with a
velocity v equal to the velocity of sound. If the observer also moves away from the source
with the same velocity v, the apparent frequency heard by the observer is
(A) 50Hz (B) 100Hz
(C) 150Hz (D) 200Hz
Solution: No relative motion between source and observers. Hence (D).

17. A piano string 1.5m long is made of steel of density 7.7103 kg/m3 and Young's modulus
21011 N/m2. It is maintained at a tension which produces an elastic strain of 1% in the string.
The fundamental frequency of transverse vibrations of string is
(A) 85Hz (B) 170Hz
(C) 340Hz (D) 310Hz
F.ℓ Δℓ
⇒ F = YA .
Solution: Y = A . Δℓ ℓ
1
=21011 100 A

Join Elite Batch – Physics ( Batch -2 ) - Whatsapp -9264953797

Click Here - https://imjo.in/JKBjbX ( Batch Starting 01 March )


Final Revision Questions
2020
– Elite Batch

1 F 1 2×109 A
√ √
n= 2×1. 5 Ad
n = 170Hz
=
3 A×7 .7×103

18. [ −1
]
The stationary waves set up on a string have the equation y=( 2 mm ) sin ( 6.28m ) x cos ( ωt ) .
This stationary wave is created by two identical waves, of amplitude A, each moving in
opposite directions along the string. The value of A.
(A) A = 2 mm
(B) A = 1 mm
(C) The smallest length of the string is 60 cm
(D) The smallest length of the string is 2 m.
Solution : Comparing with the equation

Y = 2A sin
( nπxL )cos ωt
2A = 2 mm  A = 1 mm
nπx n
=6 . 28 x=2 πx m
L or L = 2
for n = 1, L = 50 cm
 (B)

19. An accurate and reliable audio oscillator is used to standardise a tuning fork. When the
oscillator reading is 514, two beats are heard per second. When the oscillator reading is
510, the beat frequency is 6 Hz. The frequency of the tuning fork is
(A) 506 (B) 510
(C) 516 (D) 158

Solution: When the oscillator reading is 514, two beats are heard. Hence the frequency f the
tuning fork is 514  2 = 516 or 512. When the oscillator reading is 510, the frequency of the
tuning fork is 510  6 = 516 or 504. The common value is 516. Hence the frequency is 516
Hz.
 (C)

20. A sound wave of wavelength  travels towards the right horizontally with a velocity v. It
strikes and reflects from a vertical plane surface, travelling at acceleration a starting from
rest. The number of positive crests striking in a time interval of 5 sec on the wall is

(A) 5
( v+ 5a2 ) / λ (B) 5
( v− 5a2 ) / λ
(C) 5v/  (D) (v – 5a) /5

1
Solution :  Distance covered = v.5 + 2 a.52 = 5[v+(5a/2)]

Join Elite Batch – Physics ( Batch -2 ) - Whatsapp -9264953797

Click Here - https://imjo.in/JKBjbX ( Batch Starting 01 March )


Final Revision Questions
2020
– Elite Batch

∵ Number of positive crests striking per second is same as the frequency.


 (A)

21. Two waves represented by y1 =10 sin (2000 t+2x) and y2=10 sin (2000  t +2x+/2) are
superposed at any point at a particular instant. The resultant amplitude is
(A) 10 units (B) 20 units
(C) 14.1 units (D) zero

Solution: The resultant amplitude A of two waves of amplitudes a 1 and a2 at a phase difference
2 2 1/2
 is ( (a1 +a2 +2 a 1 a2 cos φ) . Substituting a1 = 10, a2 = 10 and  = 900, we get A = 14.1.
 (C)

22. When pressure increases by 1/2 atm o here and temperature increases by 1 0C from (100C),
the velocity of sound may, (Take vs = 340 m/s)
(A) increases by 0.455 ms1 (B) decreases by 4.55 ms1
(C) increases by 0.455 ms1 (D) decreases by 4.55 ms1
Solution : (A) V  √ T and the increase of pressure does not change the velocity of sound. 
T'
V = V 0
√T 0  340
374

273  340.4554 m/s

23. Which of the following wave equations have maximum amplitude at the origin of coordinate
system at time t = 0 sec?
(A) y = ym sin k (x + vt) (B) y = ym cos k (x + vt)
(C) y = ym sin k(x  vt) (D) none of these.
Solution : (D) Because y = ym cos(0) = ym

24. A heavy uniform rope hangs vertically from the ceiling, with its lower end free. A disturbance
on the rope travelling upward from the lower end has a velocity v at a distance x from the
lower end.
1

(A) x (B) vx
1

(C) vα √ x (D) √x
Solution: Let m be the mass per unit length of the rope.
T = tension in the rope at a distance x from the lower end.
 T = mgx
V= √T /m=√ mgx/m=√ gx
 v  √x
 (C)

Join Elite Batch – Physics ( Batch -2 ) - Whatsapp -9264953797

Click Here - https://imjo.in/JKBjbX ( Batch Starting 01 March )


Final Revision Questions
2020
– Elite Batch

25. If x = a sin [t + (/3)] and x = a cos t, then what is the phase difference between two
waves?
(A) /3 (B) /6
(C) /2 (D) 
Solution: Now sin[t + (/3)] and cost = sin[(/2) + t]
 Phase difference = /2  /3 = /6
 (B)

26. The power of sound from a speaker is raised from 10 mW to 500 mW. What is the power in
creased in (decibel) dB as compared to initial original power is
(A) 1.6 dB (B) 50 dB
(C) 16.9 dB (D) 6.9 dB
P
Solution: P(dB) = 10 log
Po ( )
= 10 log 10
500
( )
= 10log1050
= 10(1.69) = 16.9 dB
 (C)

CMP : A small here of radius R is arranged to pulsate so that its radius varies in simple
harmonic motion between a minimum of R –  R and a maximum of R +  R with
frequency f. This produces sound waves in the surrounding air of density  and bulk
modulus B (The amplitude of oscillation of the here is the same as that of the air at the
surface of the here).

27. Find the intensity of sound waves at the surface of the here
ρB  2 f 2  ΔR  2 ρB  2 f 2  ΔR 
2 2

(A) (B)
4 ρB  f  ΔR 
2

(C)
2 2
(D)
8 ρB  2 f 2 R 2
Solution: (B) The density is (by definition) the time average value of P (x, t)v y (x, t). For any
value of x the average value of function cos2 (Kx –  t) over one period\
2 1
T  I  BKA 2
 is half 2
B
v2 
By using the relation   vk and 
1
I B2 A 2
So we get 2 (intensity of sinusoidal wave)
The amplitude of oscillation is R .
1
B  2f   R   22 f 2  B   R 
2 2 2
I
Hence, 2

28. The total acoustic power radiated by the here will be

Join Elite Batch – Physics ( Batch -2 ) - Whatsapp -9264953797

Click Here - https://imjo.in/JKBjbX ( Batch Starting 01 March )


Final Revision Questions
2020
– Elite Batch

ρB π 3 f 2 R 2  ΔR  2 ρB π 3 f 2 R 2  ΔR 
2 2

(A) (B)
 ΔR  ρB f 2 R 2  ΔR 
2 2 2 2 3 2
4 ρB π f R 8π
(C) (D)
P  I   Area   I 4 R  8 f B R 2  R 
2 3 2 2

Solution: (D)

29. At a distance d > > R from the centre of the here, find the amplitude.
R   Rf 
2

   ΔR  ρB  ΔR 
2 2
2.   π
(A)  d  (B)  d 
 Rf 
2
 Rf 
ρB  ΔR 
2
ρB  ΔR 
2 2
π
2
4.   π d
(C)  d  (D)  
Solution: (A)
Pmax Pmax  Pmax v R
A    v B   R
BK 2B 2fB d
B 
v  v 1
But  B
R
A    R
Hence d
30. At a distance d > > R from the centre of the here, find pressure amplitude
 Rf   Rf 
2
 B   R 2 B    R 
2 2

(A)  d (B)  d 
 Rf   Rf 
2
2 B    R  4 B    R 
2

(C)  d (D)  d 
2
Pmax
I
Solution: (C) 2 B
 Rf 
Pmax  2 B.   R
Hence,  d

For Test
1. When we hear a sound, we can identify its source from
(A) the frequency of the sound (B) the amplitude of the sound
(C) the wavelength of the sound (D) the overtones present in the sound

Solution:  (D)

Join Elite Batch – Physics ( Batch -2 ) - Whatsapp -9264953797

Click Here - https://imjo.in/JKBjbX ( Batch Starting 01 March )


Final Revision Questions
2020
– Elite Batch

2. A man is riding a motorcycle with velocity v, towards a stationary car sounding the horn at
the frequency of 165 Hz. Police jeep is following the motorcycle at 22 m/s and sounding a
siren with a frequency of 176 Hz. Find v so that the man on the motorcycle hears no beats.
(A) 33 m/s (B) 22 m/s
(C) 0 (D) 11 m/s
Ans. jeep  motorcycle  car
For no beats the person should hear and apparent frequency of 176 Hz. By stationary car
v + vs

 176 = 165
( ) vs
(vs = velocity of sound)
 v = 22 m/s
 (B)

3. In a resonance tube water is filled so that height of air column is 0.1 m when it resonates in
its fundamental mode. Now water is removed so that the height of air column becomes 0.35
m and it resonates with next higher frequency. Then end correction is
(A) 0.012 (B) 0.025
(C) 0.05 (D) 0.024
L2 −3 L1
Ans. End correction = 2 = 0.025
 (B)

4. The length of nth seconds pendulum on the surface of Earth is 1 m. Its length on the surface
of Moon, where g is 1/6th the value of g on the surface of Earth, is
(A) 1/6 m (B) 6m
(C) 1/36 m (D) 36 m.
Solution : (A) T = 2p ; T is constant.
l
;T iscons tan t.l  g.If gisreducedby aactor of 6, then l is reducedby a factor of 6.
g

5. A particle moves according to the equation x = a cos t. The distance covered by it in 2.5 s is
(A) 3a (B) 5a
(C) 2a (D) 9a.
2 5
  ,   or T  2s
Solution : (B) T clearly, 4 oscillations are completed in 2.5 second.
5
 Dis tance cov ered   4a  5a
4

6. The time period of a mass suspended from a ring is 5s. The ring is cut into four equal parts
and the same mass is now suspended from one of its parts. The period is now
(A) 5s (B) 2.5s
5
s
(C) 1.25s (D) 16 .

Join Elite Batch – Physics ( Batch -2 ) - Whatsapp -9264953797

Click Here - https://imjo.in/JKBjbX ( Batch Starting 01 March )


Final Revision Questions
2020
– Elite Batch

m m T 5
T  2 ,T '  2   s  2.5 s
Solution : (B) k 4k 2 2

7. Two pendulum oscillate with a constant phase difference of 90 o and same amplitude. The
maximum velocity of one is v. The maximum velocity of the other is v + x. The value of x is

(A) 0 (B) 2

(C) 3 (D) v tan 90o.
Solution : (A) When, one is at the mean position, the other is at the extreme position.
Since phase difference I constant. Therefore  is same for both pendulum. Also, a is same.
So,  is same.

8. The displacement of a particle in simple harmonic motion in one time period is


(A) A (B) 2A
(C) 4A (D) zero.
Solution : (D) At the end of one complete vibration, the particle returns to the initial position.

9. If the displacement equation of a particle be represented by y = A sin pt + B cos pt, the


particle executes
(A) a uniform circular motion (B) a uniform elliptical motion
(C) a SHM (D) a rectilinear motion.
Solution : (C) y = A sin pl + B cos pt
d
 (y) or   Ap
V= dt cos pt – Bp sin pt
d
 ( )
Acceleration dt
= –Ap2 sub pt–Bp2 cos pt
= –p2 (A sin pt + B cos pt) = –p2y
Clearly, motion is SHM.

10. The time period of a mass sounded by a ring is T. If the ring is cut into four equal parts and
the mass is again sounded by one of the pieces, the period of vibration is now
T
(A) T/2 (B) 3
T
(C) 2 (D) 2T.
1 T' 11 T
T ;T '  or  or T '  .
Solution : K 4K T 2 2

11. The need of sound in air is 350 ms –1. The fundamental frequency of an open pipe 50 cm long
will be
Join Elite Batch – Physics ( Batch -2 ) - Whatsapp -9264953797

Click Here - https://imjo.in/JKBjbX ( Batch Starting 01 March )


Final Revision Questions
2020
– Elite Batch

(A) 175 Hz (B) 350 Hz


(C) 700 Hz (D) 50 Hz.
 350  100
  Hz  350Hz
Solution : (C) 2l 2  50
Vl
 2
= 4l ( l << l)

12. A flat horizontal board moves us and down in SHM of amplitude a. Then the shortest
permissible time period of the vibration such that an object placed on the board may not lose
contact with the board is
g a
(A) 2 a (B) 2 g
1 g
(C) 2
ga (D) 2 a .
Solution : (B) m2 = mg
g

a
2 g a
 T  2
T a or g .

13. A progressive sound wave of frequency 500 Hz is travelling through air with a speed of 350
ms–1. A compression maximum appears at a place at a given instant. The minimum appears
at a place at a given instant. The minimum occurs at the same point, is
1
s
(A) 200 s (B) 250
1 1
s s
(C) 500 (D) 1000 .
1 T 1
s; t   s
Solution : (D) 500 2 1000

14. A uniform ring of force constant k is cut into two pieces, the lengths of which are in the ratio
1:2. The ratio of the force constants of the shorter and the longer pieces is
(A) 1:2 (B) 2:3
(C) 2:1 (D) 1 : 3.
F/a Fl Fl
Y  or Y   
Solution : (C) l / l  l  l  
kl
Y
or  or kl = constant

Join Elite Batch – Physics ( Batch -2 ) - Whatsapp -9264953797

Click Here - https://imjo.in/JKBjbX ( Batch Starting 01 March )


Final Revision Questions
2020
– Elite Batch

k1 l 2 k1 2
Y  
or k2 l1 or k 2 1.

15. A mass on the end of a ring undergoes simple harmonic motion with a frequency of 0.5 Hz. If
the attached mass is reduced to one quarter of its value, then the new frequency in Hz is
(A) 4.5 (B) 2.0
(C) 0.25 (D) 1.0.
1

Solution : (D) m
1 '
'  or  2 or  '  2
m 
4
or ’ = 2 × 0.5 Hz = 1 Hz

16. The motion of particle executing SHM is given by x = 0.01 sin  (t + 0.05), where x is in
meter and time is in second. The time period is
(A) 0.2 s (B) 0.1 s
(C) 0.02 s (D) 0.01 s.
Solution : (C) Comparing with x =  sin (t + o), we get
2
 100
 or T
2
s  0.02 s
or T = 100

17. A pendulum sounded from the ceiling of a train has a period T when the train is at rest. When
the train is accelerating with a uniform acceleration, the period of oscillation will
(A) increase (B) decrease
(C) remain unaffected (D) become infinite.
l l
2 ;T '2
g g2  a2
Solution : (B) Comparing with y =
Clearly, T’ < T

18. What fraction of the total energy is potential when the displacement is one-half of the
amplitude?
1
s
(A) 200 s (B) 250
1 1
s s
(C) 500 (D) 1000 .

Join Elite Batch – Physics ( Batch -2 ) - Whatsapp -9264953797

Click Here - https://imjo.in/JKBjbX ( Batch Starting 01 March )


Final Revision Questions
2020
– Elite Batch

1 a2
m2 (a 2  2 ) a 2

Ek 2 a2   2 4 3
  
E 1 a 2
a 2
4
m2a2
Solution : (C) 2 .

19. Consider a stretched string under tension and fixed at both ends. If the tension is doubled
and the cross-sectional area halved, then the frequency becomes
(A) twice (B) half
(C) four times (D) eight times.
1 T

Solution : (A) 2l m
l 
m  
But l
1 T T
  or  
2l  
2T '
' or  2 or  '  2
 
2

20. Two organ pipes produce 7 beats per second at 5 oC. When the temperature rises to 10 oC,
the number of beats is
(A) =7 (B) <8
(C) >7 (D) Data is inadequate.

 1 1 
1   2    
2  l1 l 2 
Solution : (C)
21. Two tuning forks when sounded simultaneously give one beat each 0.4 second. What is the
difference of the frequencies?
(A) 1 Hz (B) 1.5 Hz
(C) 2 Hz (D) 2.5 Hz.
1 10 5
  
Solution : (D) Beat frequency = 0.4 4 2
Difference of frequencies = 2.5 Hz

22. The ratio of intensity of wave and energy density gives


(A) momentum (B) total energy
(C) propagation constant (D) velocity.
energy
Solution : (D) Intensity = area × time
energy
Energy density = volume

Join Elite Batch – Physics ( Batch -2 ) - Whatsapp -9264953797

Click Here - https://imjo.in/JKBjbX ( Batch Starting 01 March )


Final Revision Questions
2020
– Elite Batch

Intensity energy
   velocity
Energy density area  time
area  dis tance
 velocity
= area × time

23. Two sound waves travel in the same direction in a medium. The amplitude of each wave is A
and the phase difference between the two waves is 120 o. The resultant amplitude will be.
(A) A (B) 2A
(C) 4A (D) 2A .
2 2 2 2 o
Solution : (A) A’ = A + A +2A cos 120
 1
    A 2 or A '  A
= 2A2 + 2A2  2 

24. The intensity ratio of two waves is 1 : 16. The ratio or their amplitudes is
(A) 1 : 16 (B) 1:4
(C) 4:1 (D) 2 : 1.
a1 I1 1 1
2
 2
 
Solution : (B) a I 16 4

25. If the speed of sound at 0oC is 0, then the velocity at 273oC will be
(A)  (B) 
(C)  (D) .
 273 273  273
  2 or  273  1.414  0
Solution : (C)  0 273

26. The equation of transverse wave in stretched string is


 t x
y  5 sin2   
 0.04 50 
Where distances are in cm and time in second. The wavelength of wave is
(A) 15 cm (B) 10 cm
(C) 25 cm (D) 50 cm.
2 2 2
k ,  or   50 cm.
Solution : (D) 50  50

CMP: A person observe two points on a string as a travelling wave passes them. The points
are at x1  0 and x2  1m. The transverse motions of the two points are found to be as
follows:
y1  0.2 sin 3t
y2  0.2 sin (3 t  /8)

Join Elite Batch – Physics ( Batch -2 ) - Whatsapp -9264953797

Click Here - https://imjo.in/JKBjbX ( Batch Starting 01 March )


Final Revision Questions
2020
– Elite Batch

27. What is the frequency in hertz?


(A) 1.5 Hz (B) 3 Hz
(C) 4.5 Hz (D) 1 Hz
Solution: (A)
  3
3
    1.5 Hz
2

28. What is the maximum wavelength?


(A) 32 m (B) 16 m
(C) 8m (D) 4m
Solution: (B)
For maximum wave length
2
Phase diff = max path difference
 2
 1
i.e., 8 max
 max  16m

29. With what maximum speed does the wave travel?


(A) 48 m/s (B) 24 m/s
(C) 16 m/s (D) 8 m/s
Solution: (B) Maximum wave speed
v max  max
 v max  1.5 16
= 24 m/s

30. Which way is the travelling? Show how you reach this conclusion.
(A) Positive x–axis (B) Negative x–axis
(C) Along y–axis (D) Data insufficient
Solution: (D) From the given data it is not possible to predict that wave is moving in +ve or –ve
direction.

OBJECTIVE
1: Calculate the root mean square speed of smoke particles of mass 5×10−17 kg in
Brownian motion in air at NTP. Boltzmann constant k =1. 38×10−23 JK−1

(A) 1.5 cm/s (B) 2.2 cm/s

Join Elite Batch – Physics ( Batch -2 ) - Whatsapp -9264953797

Click Here - https://imjo.in/JKBjbX ( Batch Starting 01 March )


Final Revision Questions
2020
– Elite Batch

(C) 2.3 cm/s (D) 4.4 cm/s


Ans. (a)
1 m
mV 2rms= RT
Solution: PV = 3 M

3 RT 3 ( NK ) T

V rms =
√ √ M
=

3kT
= √ μ
where  = mass of one molecule.

3×1. 38×10−23×273

−3
V rms =
√ 5×10−17

= 15×10 m/ s=1 .5 cm/s

2: During an experiment an ideal gas is found to obey an additional law VP2 = constant.
The gas is initially at temp T and volume V. What will be the temperature of the gas when
it expands to a volume 2V?
' '
(A) T =√4 T (B) T = √2 T
' '
(C) T = √5 T (D) T = √6 T
Ans. (b)
Solution: According to the given problems
VP2 = constant
From the gas law
PV = nRT
k

( )
√V
V =nRT

nR
√ V =( ) T
 K

Join Elite Batch – Physics ( Batch -2 ) - Whatsapp -9264953797

Click Here - https://imjo.in/JKBjbX ( Batch Starting 01 March )


Final Revision Questions
2020
– Elite Batch

V1 T1 V T
 √ ()
V2
=
T2
,
i.e, √ =
2V T'
'
 T = √2 T

Q.3-5 We have two vessels of equal volume, one filled with hydrogen and the other with
equal mass of Helium. The common temperature is 27oC.
3: What is the relative number of molecules in the two vessels ?
nH 1 nH 5
= =
(A) nHe 1 (B) nHe 1

nH 2 nH 3
= =
(C) nHe 1 (D) nHe 1

Ans. (C)

4: If pressure of Hydrogen is 2 atm, what is the pressure of Helium ?


(A) pHe = 2 atm. (B) pHe = 3 atm.

(C) pHe = 4 atm. (D) pHe = 1 atm.


Ans. (d)

5: If the temperature of Helium is kept at 27o C and that of hydrogen is changed, at what
temperature will its pressure become equal to that of helium ? The molecular weights
of hydrogen and helium are 2 and 4 respectively.
(A) 123oC (B) 140oC
(C) 160oC (D) 183oC
Ans. (a)

Solution 3-5:
3. The masses of hydrogen and helium gases in the vessels are equal. This means that the
product of the number of molecules and the mass of a molecule must be same for H2 and He
gases. Since molecular masses of H2 and He are in the ratio 1: 2, their number of

Join Elite Batch – Physics ( Batch -2 ) - Whatsapp -9264953797

Click Here - https://imjo.in/JKBjbX ( Batch Starting 01 March )


Final Revision Questions
2020
– Elite Batch

molecules nH and nHe in the vessels must be in the reverse ratio, that is,
nH 2
=
nHe 1

4. The equation of state for one mole of a gas is


pV = RT = NkT
Where N is Avogadro’s number (no. of molecules in one mole) and k is Boltzmann’s
constant. If a gas has n molecules, the equation of state will be
pV = nkT
For a given volume and a given temperature, we have
p  n.
Since H2 and He have same volume and same temperature (27 oC), we have
pH nH 2
= =
p He nHe 1

Here pH = 2 atm.
 pHe = 1 atm.

5. Again, we have
pV = nkT
H2 and He have equal volumes. For having equal pressure, we must have
nHTH = nHeTHe
T He nH
= =2
or TH n He

Here THe = 27 + 273 = 300 K


1
 TH = 2 THe = 150 K

= 150  273 = 123oC

6: A vessel contains a mixture of 7 gm of nitrogen and 11 gm of carbon dioxide at


temperature T = 290 K. If pressure of the mixture P = 1 atm, calculate its density (R = 8.31
J/mol k)
(A) 2.5 kg/m3 (B) 1.5 kg/m3

Join Elite Batch – Physics ( Batch -2 ) - Whatsapp -9264953797

Click Here - https://imjo.in/JKBjbX ( Batch Starting 01 March )


Final Revision Questions
2020
– Elite Batch

(C) 4.5 kg/m3 (D) 7.5 kg/m3


Ans. (b)
m
n=
Solution: As molecular weight of N2 and CO2 are 28 and 44, and M ,
7 1 11
nN= = and nC = =1 / 4
28 4 44
1 1 1
n=n N +nC = + =
So, 4 4 2

Now, according to gas law PV = nRT

V=
nRT
P =
( 12 ) 81..01×10
31×290
5
−2
=1 .19×10 m3

and m = 7+11 = 18 gm = 18  10-3 kg


m 18×10−3 kg
= 3
=1 .5 kg/m 3
so,  = V 1 .19×10 m
−2

Q.7-10.The pressure of a monoatomic gas increases linearly from 4×105 N/m2 to


8×105 N/m2 when its volume increases from 0.2 m3 to 0.5 m3. Calculate

7: work done by the gas


(A) 2.8×105 J (B) 1.8×106 J
(C) 1.8×105 J (D) 1.8×102 J
Ans. (C)

8: increase in internal energy

(A) ΔU =4 .8×105 J (B) ΔU =4 .8×10 4 J

(C) ΔU =6 . 8×10 5 J (D) ΔU =4 .8×106 J

Ans. (a)

9: amount of heat supplied

(A) 8 .6×105 J (B) 12. 6×105 J

Join Elite Batch – Physics ( Batch -2 ) - Whatsapp -9264953797

Click Here - https://imjo.in/JKBjbX ( Batch Starting 01 March )


Final Revision Questions
2020
– Elite Batch

(C) 6 . 6.×105 J (D) 10 .6×105 J

Ans. (C)

10: molar heat capacity of the gas [R = 8.31 J/mol k]


(A) 20.1 J/molK (B) 17.14 J/molK
(C) 18.14 J/molK (D) 20.14 J/molK
Ans. (b)

Solution 7- 10:
PF
7. Work done by the gas, B

P
ΔW =∫ PdV = area under P-v curve PI A

1
P1 ( V F −V I ) + P −P I ) ×( V F−V I )
= 2( F
VI VF
V
1
V −V P +P
= 2 ( F I )( F I )
1
( 0 .5−0 . )( 8+ 4 )×105 =1 .8×105 J
= 2

8. Change in internal energy of a gas is given by


nR ΔT PF V F−P I V I
ΔU =nC V ΔT = =
γ −1 γ −1
As the gas is monoatomic,  = 5/3
105 ( 8×0 . 5−4×0 .2 ) 3
ΔU = = ×105 ( 4−0 . 8 )
5 2
So 3
−1[ ]
 ΔU =4 .8×105 J

9. From 1st law of thermodynamics


ΔQ=ΔU +ΔW

Join Elite Batch – Physics ( Batch -2 ) - Whatsapp -9264953797

Click Here - https://imjo.in/JKBjbX ( Batch Starting 01 March )


Final Revision Questions
2020
– Elite Batch

= ( 4.8+1.8 )×105 =6.6.×105 J

10. Molar heat capacity is defined as


5
ΔQ ΔQ×R 6 . 6×10 ×8 .31
C= = = 5
nΔT P F V F −PI V I 10 ( 8×0 .05−4×0. 2 )
54 . 846
=17. 14
i.e, C = 3 .2 J/molK

Q.11-13.Two moles of Helium gas ( = 5/3) are initially at temperature 27oC and occupy a
volume of 20 litres. The gas is first expanded at constant pressure until the volume is
doubled. Then it undergoes an adiabatic change until the temperature returns
to its initial value.
11: What are the final volume.

(A) 113.13×10−3 m3 (B) 213.13×10−3m3

(C) 313.13×10−3m3 (D) 13.13×10−3m3


Ans. (A)

12: What are the final pressure of gas?

(A) 0.44×105 N/m2 (B) 0.84×105 N/m2

(C) 0.94×105 N/m2 (D) 0.34×105 N/m2


Ans. (a)

13: What is the work done by the gas? (Gas constant R = 8.3 T/mole K)

(A) 13450J (B) 14450J

(C) 16450J (D) 12450J


Ans. (d)

Join Elite Batch – Physics ( Batch -2 ) - Whatsapp -9264953797

Click Here - https://imjo.in/JKBjbX ( Batch Starting 01 March )


Final Revision Questions
2020
– Elite Batch

Solution:11. From ideal gas equation P


A B
PV = nRT PA
nRT 2  8.3  300 C
P= 
initial pressure V 20  10 3

 2.49  10 5 N / m 2 vA vB V

When volume of gas is doubled at constant pressure, its temperature is also doubled. This
process is shown on P-V curve by line AB. The gas then cools to temperature T
adiabatically. This is shown by curve BC. The whole process is represented by curve ABC.

5 2
At point B, pressure P B =P A =2.49×10 N/m . Volume V B =2V A =40×10 m
−3 3
, Temperature
T B =600K.

Now from adiabatic equation TV γ−1 = constant


T A V (γ−1) =T C V (γ−1) 
We have A C

V C γ−1 T 600

()
VB
= B = =2
TC 300
VC 1/ ( γ−1 )
=2 
 VB = 23/2
Final volume


V C =2 √2V B
−3 3
=2×1.414×40×10−3 =113.13×10 m
12. final pressure
nRTC 2×8.3×300
PC = = =0.44×105 N/m2
VC 113.13×10−3
13. The work done by gas in isobaric process AB
5
=2. 49×10 ×(40−20)×10
−3
=4980 J
The work done by gas during adiabatic process BC
2×8.3
nR =  [ 300−600 ]
W2 =  T −T 5
1−γ [ 2 1] ()
1−
3 =7470J .

Join Elite Batch – Physics ( Batch -2 ) - Whatsapp -9264953797

Click Here - https://imjo.in/JKBjbX ( Batch Starting 01 March )


Final Revision Questions
2020
– Elite Batch

 Net work done by gas W=W 1 +W 2


=4980+7470=12450J

14-15. When 1 gm of water changes from liquid to vapour phase at constant pressure
of 1 atmosphere, the volume increases from 1 cm 3 to 1671 c.c. The heat of vaporization
at this pressure is 540 cal/gm. Find
14: The work done (in J) in change of phase

(A) 170 .78 Joule (B) 200 .67 Joule

(C) 190. 78 Joule (D) 168.67 Joule


Ans. (d)

15: Increase in internal energy of water.


(A) 2099.33 J (B) 3099.33 J
(C) 4099.33 J (D) 5099.33 J
Ans. (a)

Solution:14. As the process is isobaric

ΔW =∫ PdV =P [ V F−V I ]

= 1.01×10 6 [ 1671−1 ] =1688.7×106 erg


= 168.67 Joule [1 erg = 10-7J]

15. From 1st law of thermodynamics


ΔQ=ΔU +ΔW
ΔQ=mL=1×540 cal
= 2268 J, [ 1 cal = 4.2J]

so, ΔU =ΔQ−ΔW

= ( 2268−168 . 67 ) J
= 2099.33 J

Join Elite Batch – Physics ( Batch -2 ) - Whatsapp -9264953797

Click Here - https://imjo.in/JKBjbX ( Batch Starting 01 March )


Final Revision Questions
2020
– Elite Batch

o
16: A glass flask of volume one litre at 0 C is filled level full of mercury at this
o
temperature. The flask and mercury are now heated to 100 C. How much mercury will spill
−4 o
out if coefficient of volume expansion of mercury is 1. 82×10 / C and linear expansion
of glass is 0 .1×10−4 / o C respectively?

(A) 14.2 c.c. (B) 15.2 c.c.


(C) 18.2 c.c. (D) 20.2 c.c.J
Ans. (b)
Solution: In case of thermal expansion of liquid, change in volume of liquid relative to
container is given by
ΔV =V ( γ L−γ S ) Δθ

Here V = 1 litre = 1000 c.c. γ S = 3 glass


−4 o
= 0 .3×10 / C

So, ΔV =1000 ( 1.82−0.3 )×10−4×( 100−0 )


= 15.2 c.c.

17: Two cylinders A and B fitted with pistons contain equal amounts of an ideal diatomic
gas at 300K. The piston A is free to move, while that of B is held fixed. The same amount
of heat is given to the gas in each cylinder. If the rise in temperature of the
gas in A is 30K, then the rise in temperature of the gas in B is.
(A) 30K (B) 18K
(C) 50K (D) 42K

Solution: For cylinder A. For cylinder B


dQ = nCPdT1 dQ = nCvdT2
 nCPdT1 = nCvdT2
From (I) and (II)
c v dT 2 = ( c v +R ) 30

(c v +R)30
dT 2 = 
 cv

Join Elite Batch – Physics ( Batch -2 ) - Whatsapp -9264953797

Click Here - https://imjo.in/JKBjbX ( Batch Starting 01 March )


Final Revision Questions
2020
– Elite Batch

5
cv = R
For diatomic gas 2

 dT2  42K .

o o
18: 80 gm of water at 30 C is poured on a large block of ice at 0 C . The mass of
ice that melts is
(A) 30 gm (B) 80 gm
(C) 150 gm (D) 1600 gm

o
Solution: Since the block of ice at 0 C is large, the whole of ice will not melt, hence final
o
temperature is 0 C .

 Q1 = heat given up by water in cooling up to 0O C

= ms Δθ=80×1×( 30−0 )
= 2400 cal
If m gm be the mass of ice melted, then
Q2 = ML = m×80
Q 1 =Q 2

m×80=2400 or m=30 gm
Here A is correct.

19: A gas at pressure Po is contained in a vessel. If the masses of all the molecules
are halved and their speeds doubled, the resulting pressure would be
(A) 4Po (B) 2Po
Po
(C) Po (D) 2

1 mn ( 2rms )
P= V
Solution: 3 V where m = mass of one gas molecules
n = total no. of gas molecules
i.e, P  m and P  Vrms
Here m is halved and Vrms is doubled
Join Elite Batch – Physics ( Batch -2 ) - Whatsapp -9264953797

Click Here - https://imjo.in/JKBjbX ( Batch Starting 01 March )


Final Revision Questions
2020
– Elite Batch

 pressure will be doubled


Hence, (B) is correct

20: The volume V versus temperature T graphs for a certain P2


amount of a perfect gas at two pressures P 1 and P2 are P1
shown in the figure. Here V

(A) P1 < P2 (C) P1 = P2


(B) P1 > P2 (D) can’t be O T

m
PV = RT
Solution: For a perfect gas, M
mR
v= RT
 PM
mR
So, the slope of the graph is PM
1
Slope  P
Hence P1 > P2
Hence, (C) is correct

21: At room temperature the rms speed of the molecules of a certain diatomic gas is
found to be 1930 m/s. The gas is

(A) H2 (B) F2

(C) O2 (D) Cl 2

3RT
Vrms 
Solution: M

3RT 3x8.31x300
M= = =2.0078x10−3 kg
V 2 (1930)2
 2.00 gm
 rms

It is molecular weight of hydrogen (H2 ) .

Join Elite Batch – Physics ( Batch -2 ) - Whatsapp -9264953797

Click Here - https://imjo.in/JKBjbX ( Batch Starting 01 March )


Final Revision Questions
2020
– Elite Batch

22: The latent heat of vaporization of water is 2240 J. If the work done in the process of
vaporization of 1 gm is 168 J, then increase in internal energy is
(A) 2408 J (B) 2240 J
(C) 2072 J (D) 1904 J
Solution: L = 2240 J, m = 1 gm
dW = 168 J
dQ = mL = dU + dW

or 1×2240=dU +168
dU = 2072 J
Hence, (C) is correct

23: For a gas, y = 1.286. What is the number of degrees of freedom of the moleculas
of this gas ?
(A) 3 (B) 5
(C) 6 (D) 7
Ans. (d)
2 2 2
1  1.286 or  0.286 or 7
Solution: (D) n n 0.286

24: Which of the following temperatures is the highest?


(A) 100 K (B) –13oF
(C) –20oC (D) –30oC
Solution: (B ) –13oF is (13+32)o below ice point on F scale.

25: An ideal gas ( = 1.5) is expanded adiabatically. How many times has the gas to
be expanded to reduce the root mean square velocity of molecules 2.0 times
(A) 4 times (B) 16 times
(C) 8 times (D) 2 times
Ans. (B)

Join Elite Batch – Physics ( Batch -2 ) - Whatsapp -9264953797

Click Here - https://imjo.in/JKBjbX ( Batch Starting 01 March )


Final Revision Questions
2020
– Elite Batch

3 RT
Solution:
V rms =
√ M


V rms ∝ √T
Vrms is to reduce two times i.e, temperature of the gas will have to reduce four times or
'
T 1
=
T 4
During adiabatic process
TV γ−1 =T ' V ¿
1
V' T 1

or,
V
= '
T () γ−1 =( 4 ) 1. 5−1
=42 =16

 V ' =16 V
Hence, (B) is correct

26: A thin copper wire of length L increases in length by 1% when heated from
0 C to 100o C . If a thin copper plate of area 2L×L is heated
o o
from 0 C
o
to 100 C ,
the percentage increase in its area will be
(A) 1% (B) 2%
(C) 3% (D) 4%
Ans. (b)

Solution: L = Lo [ 1+αΔθ ]
ΔL=Lo α Δθ

ΔL
=αΔθ
Lo

1
 αΔθ = percentage increase in length = 100
1

(
L=Lo 1+
100 )
2
1
Hence
2 L2 =2 L
o2 ( 1+
100 )
Join Elite Batch – Physics ( Batch -2 ) - Whatsapp -9264953797

Click Here - https://imjo.in/JKBjbX ( Batch Starting 01 March )


Final Revision Questions
2020
– Elite Batch

2
2 L2 −2 L2o ≈2 L2o ×
100
ΔA 2
=
2 100
=2 %
or 2 L o

Hence, (B) is correct

27: Gas at pressure Po is contained in a vessel. If the masses of all the molecules are
doubled and their speed is halved, the resulting pressure P will be equal to
(A) 2Po (B) Po/4
(C) Po (D) Po/2
Ans. (d)
1
Solution: Po = 3 ( mnv ) v 2
rms

1 '

P= 3 ( mvn ) v ¿
rms

v rms
v 'rms =
where m = 2m, 2

putting the value


' ¿ 2
P m v rms 2 m v rms 1
= =
P o mv 2rms m 4×v 2rms 2
 =
P = Po/2

28: The molar heat capacity in a process of a diatomic gas if it does a work of Q/4,
when Q amount of heat is supplied to it is
2 5
R R
(A) 5 (B) 2
10 6
R R
(C) 3 (D) 7
Ans. (C)

Join Elite Batch – Physics ( Batch -2 ) - Whatsapp -9264953797

Click Here - https://imjo.in/JKBjbX ( Batch Starting 01 March )


Final Revision Questions
2020
– Elite Batch

Solution: dU = CV dT =
( 52 R ) dT ⇒ dT = 2(5dUR )
Q 3Q
Q− =
st
From 1 law of thermodynamics dU = dQ – dW or dU = 4 4
dQ Q 5 QR 10
C= = = = R
dT dU 3Q 3
Now molar heat capacity
2( ) ( )
5R
2
4
Hence (C) is correct

29: For an ideal gas:


(A) the change in internal energy in a constant pressure process
from temperature T1 to T2 is equal to nCv (T2 - T1), where Cv is the molar
specific heat at constant volume and n the number of moles
of the gas.
(B) the change in internal energy of the gas and the work done by the
gas are equal in magnitude in an adiabatic process.
(C) the internal energy does not change in an isothermal process.
(D) no heat is added or removed in an adiabatic process.
(A) A, B (B) A, B, C
(C) A, B, C, D (D) A, C

Solution: (C) Change in internal energy depends only on change in temperature since
internal energy is a function of state only i.e. dU = nCv,dT.
In adiabatic process, dQ = 0,
Hence, dU + dW = 0  dU =  dW
i.e. magnitude of change in internal energy is equal to magnitude of work done.

30: Heat is supplied to a diatomic gas at constant pressure. The ratio of ΔQ : ΔU : ΔW is


(A) 5:3:2 (B) 5:2:3
(C) 7:5:2 (D) 7:2:5
Ans. (C)

Join Elite Batch – Physics ( Batch -2 ) - Whatsapp -9264953797

Click Here - https://imjo.in/JKBjbX ( Batch Starting 01 March )


Final Revision Questions
2020
– Elite Batch

7
Solution:
[ ] 7
ΔQ=nC P dT = nR ( ΔT )
2 ,
C P= R
2

5
[ ] 5
ΔU =nC V ΔT = nR ΔT
2 ,
CV= R
2

and ΔW =ΔQ− ΔU=nR ΔT

 ΔQ : ΔU : ΔW =7 :5 : 2
Hence, C is correct

31: Two mole of argon are mixed with one mole of hydrogen, then Cp/Cv for the mixture is
nearly
(A) 1.2 (B) 1.3
(C) 1.4 (D) 1.5
Ans.(C)
Solution: Average degree of freedom
2×3+1×5 11
=
fav = 2+3 5
1 5 16
mix = 1 + f av = 1 + 11 = 11 = 1.4
Hence, C is correct Answer.

32: An ideal gas is taken through the cycle A BC A as shown in figure. If the net
heat supplied to the gas in the cycle is 5J, the work done by the gas
in the process CA is,
(A) -5J (B) -10J
(C) -15J (D) -20J

Solution: (A)
 Qnet   Wnet   WCA   WAB

5   WCA  10  (2  1)
5  10   WCA
 WCA  5

Join Elite Batch – Physics ( Batch -2 ) - Whatsapp -9264953797

Click Here - https://imjo.in/JKBjbX ( Batch Starting 01 March )


Final Revision Questions
2020
– Elite Batch

33: When an ideal gas at pressure P, temperature T and volume V is isothermally


compressed to a V/n, its pressure becomes P i . If the gas is compressed adiabatically
to V/n, its pressure becomes Pa. The ratio Pi / Pa is
(A) 1 (B) n
(C) n (D) n 1-
Solution: (D) For isothermal process, PV = constant. Therefore
V
 PV
PiVi = PV or Pi n or Pi = nP ……(i)
For adiabatic process, PV = constant. Therefore

V 
Pa    PV 
 n
(P, V, T)
Pa (Va)  = PV or

 V   or Pa = nP …..(ii)
P  Pi ,   ,T 
 n 
From (i) and (ii) we get
 V  
 Pn ,   ,T ' 
 n 
Pi n
v    n (1 )
Pa n

34: When an ideal monatomic gas is heated at constant pressure, the fraction of heat
energy supplied which increases the internal energy of the gas is
(A) 2/5 (B) 3/5
(C) 3/7 (D) 3/4
nR T
U 
Solution: (B)  1

w
U   w  nRT 
 1
Q  U

 1
 Q  W  U 

Join Elite Batch – Physics ( Batch -2 ) - Whatsapp -9264953797

Click Here - https://imjo.in/JKBjbX ( Batch Starting 01 March )


Final Revision Questions
2020
– Elite Batch

U (   1)  Q  U
U    1  1  Q

U 1 3
 
Q  5 [for monoatomic gas  = 5/3 ]

35: A monatomic ideal gas, initially at temperature T 1 is enclosed in a cylinder fitted with a
frictionless piston. The gas is allowed to expand adiabatically to a temperature T 2 by
releasing the piston suddenly. If L 1 and L2 are the lengths of the gas column before
and after expansion respectively, then T1/T2 is given by
(A) (L1/L2)2/3 (B) L1/L2
(C) L2/L1 (D) (L2/L1) 2/3
1
Solution: (D) TV  constant
Initial position T1(L1A)–1 = constant
Final position T2 (L2A)–1 = constant
1
T L 
 1 1 1
T2  L2 
1
T1  L2 
 
T2  L1 
5 / 3 1
L 
 2 
 L1  [for monoatomic gas   = 5/3]
2/3
L 
 2
 L1 

36: An ideal mono atomic gas at 300K expands adiabatically to twice its volume. What is
the fine temperature
(A) 189K (B) 289K
(C) 30Kj (D) Non of these
 1
Solution: (A) TV  constant

Join Elite Batch – Physics ( Batch -2 ) - Whatsapp -9264953797

Click Here - https://imjo.in/JKBjbX ( Batch Starting 01 March )


Final Revision Questions
2020
– Elite Batch

1
TV
1 1  T2V2 1
300 V11  T2  2V1 
1

300  T2  21
300 300 300 300
T2  1
 5 / 31  2 / 3   189K
2 2 2 1.587

37: What will be P-V graph corresponding to the P-T graph (process AB) for an ideal gas
shown in figure

B C

A D

(A) Hyperbolic (B) Circle


(C) Straight line (D) Elliptical
Solution: (A)

38: Starting with the same initial conditions, an ideal gas expands from volume V1 to V2 in
three different ways. The work done by the gas is W1 if the process is purely isothermal, W2
if purely isobaric and W3 if purely adiabatic. Then
(A) W2 > W1 >W3 (B) W2 > W3 >W1
(C) W1 > W2 >W3 (D) W1 > W3 >W2
Solution: (A)

Isobaric

P
Isothermal
Adiabatic

Join Elite Batch – Physics ( Batch -2 ) - Whatsapp -9264953797

Click Here - https://imjo.in/JKBjbX ( Batch Starting 01 March )


Final Revision Questions
2020
– Elite Batch

39: One mole of argon is heated using PV5/2 = = const. By which amount of heat is
obtained by the process when the temperature change by T = = -26K.
(A) 100J (B) 200J
(C) 108J (D) 208J
Solution: (C) Here n = 1
R R
  For polytropic process 
C =   1 1 x
R R

5 / 3 1  3 
1 2 
C=  

3 
Q  nC T  1 R  2R   26   108J
2 

40: 3 moles of an ideal monoatmic gas performs a cycle as shown in the fig. The gas
temperatures T1 = = 400K, T2= = 800K, T3 = = 2400K, T4 = = 1200K. What will be the net
work done.

P
B C

A D

(A) 20J (B) 20000J


(C) 200J (D) 2000J
Solution: (B) WAB = CCD = 0, because process is isochoric.
WAD = nRT = 3R(TA – TB) = 3  8.31 (2400 – 800) = 39884
 Total work done WAD + WBC = 39888 – 19944 = 20  103J = 19944
 20  103J

41: How much heat is absorbed by the system in going through the process shown
in the fig. (consider that value is taken in SI system)

Join Elite Batch – Physics ( Batch -2 ) - Whatsapp -9264953797

Click Here - https://imjo.in/JKBjbX ( Batch Starting 01 March )


Final Revision Questions
2020
– Elite Batch

900
P

100
800 400
V

(A) 20.4104 J (B) 30.4104 J


(C) 21.4104 J (D) 25.12104 J
Solution: (D) Work done = Area of PV diagram

 P2  P1   V2  V1 
= 4

 800  400
= 4
= 800  100  3.14
= 8  104  3.14
= 25.12  104

42: 3000J of heat is given to a gas at constant pressure of 2105 N/m2. If its volume
increases by 10 litres during the process, what will be the change in the internal energy
of the gas
(A) 1000J (B) 100J
(C) 200J (D) 2000J

Solution: (A) Q  3000J


P = 2  105 = Vi
Vf = (Vi + 10  10–3)
W = Pdv
W = 2  105  10  10-3 = 2  103
Q  W  U
3000 = 2  103 + U

Join Elite Batch – Physics ( Batch -2 ) - Whatsapp -9264953797

Click Here - https://imjo.in/JKBjbX ( Batch Starting 01 March )


Final Revision Questions
2020
– Elite Batch

Q  1000

43: A gas at atmospheric pressure is contained in a cylinder of volume 80 litre. When it


is compressed adiabatically to 20 litre its pressure rises to 7 atm. What will be the ratio
of specific heats of the gas
(A) 1.33 (B) 1.4
(C) 1.67 (D) 1.5
Solution: (B) Pi = 1atm = 1  105 N/m2
Vi = 80  10 – 3m3
Vf = 20  10–3 m3
Pf = 7atm = 7  105 N/m2
PiVi = PfVf
1  105  (80  10–3)  =(7  105)(20  10–3) 

 80  103 
 3 
7
 20  10 

 4

7

 log4  log7
log7
  1.40
log4

44: A gas consisting of rigid diatomic molecules was initially under standard
conditions. Then gas was compressed adiabatically to one fifth of its initial volume. What
will be the mean kinetic energy of a rotating molecule in the final state?
(A) 1.44 J (B) 4.55J
(C) 787.98  10–23 (D) 757.310-23J
Solution: (C)
  1.4

TV 1  constant

T1V1 -1 = T2 V2  -1

Join Elite Batch – Physics ( Batch -2 ) - Whatsapp -9264953797

Click Here - https://imjo.in/JKBjbX ( Batch Starting 01 March )


Final Revision Questions
2020
– Elite Batch

7
1
 V1  5
 
(300)V1 7/5 – 1 = T2  5 
300  V12 / 5 300
T2  2/5
 2 / 5
 300  52 / 5  300  (1.903)
 1 5
V12 / 5   
5 = 571
Mean kinetic energy of rotating molecules = KT = 1.38  10 – 23  571
KT = 787.98  10–23

45: Immediately after the explosion of an atom bomb, the ball of fire produced has a
radius of 100m and a temperature 105K . What will be the approximate temperature when
the ball expands adiabatically to a radius of 1000m (suppose mono atomic gas is
there)
(A) 1000K (B) 100K
(C) 105  (10–3)2/3 (D) 200K
Solution: (C) r = 100 m
4
 (100)3
 Vi = 3

Ti = 105 K
4
 (1000)3
after explosion r = 1000 m,  Vf = 3
 1
TV
i i  Tf Vf 1
 1
V 
Tf  Ti  i 
 Vf 
5
1
 4 3 
3
 3  (100) 
4 
  (1000)3 
Tf = 105  3 

= 105  (10–3)2/3
= 1.05 K

Join Elite Batch – Physics ( Batch -2 ) - Whatsapp -9264953797

Click Here - https://imjo.in/JKBjbX ( Batch Starting 01 March )


Final Revision Questions
2020
– Elite Batch

46: Which of the following is false?


(A) Enthalpy is a path function.
(B) Work is a path function.
(C) Heat is a path function.
(D) Energy is a state function
Solution: (C)

47: A gas mixture consists of 32 gram of oxygen and 36 gram of Ar a temperature T.


Neglecting all vibration modes, the total internal energy of the system is
(A) 4RT (B) 8RT
(C) 9RT (D) 11RT
F 5 3
RT  2  RT  4  RT  11RT
Solution: (D) Energy = n2 2 2

48: A mono atomic gas is supplied heat Q very slowly keeping the pressure constant. The
work done by the gas is
(A) 2/5 Q (B) 3/5 Q
(C) Q /5 (D) 2/3 Q
U 1 Q
 or U 
Solution: (D) For monatomic gas Q 3 3

From the first law of thermodynamics


Q = U + W
2
 W  Q
3
49: Which of the following parameters does not characterise the thermodynamic state
of matter
(A) Work (B) Pressure
(C) Temperature (D) Volume
Solution: (A) Work

50: Which of the following is correct

Join Elite Batch – Physics ( Batch -2 ) - Whatsapp -9264953797

Click Here - https://imjo.in/JKBjbX ( Batch Starting 01 March )


Final Revision Questions
2020
– Elite Batch

(A) For an isothermal change PV = = constant


(B) For a isothermal process, the change in internal energy must be
equal to the work done

P2  V2 
 
P1  V1 
(C) For an adiabatic change , where  is the ratio of the two
specific heats
(D) In an adiabatic process external work done must be equal to the
heat entering the system
Solution: (A)

51: An ideal gas goes through cyclic process ABC and following (P vs T) curve is
obtained. This process can be represented by

A C

B B

P P

A C A C

V V

(A) (B)

B
B
P
P
A C
A C

V V

Join Elite Batch – Physics ( Batch -2 ) - Whatsapp -9264953797

Click Here - https://imjo.in/JKBjbX ( Batch Starting 01 March )


Final Revision Questions
2020
– Elite Batch

(C) (D)
1
Solution:(B) Process A to B isothermal. Then P  V . Process B  C is isobaric and C  A
adiabatic. Slope of adiabatic > slope of isothermal.

52: A container contain 0.1 mol of H2 and 0.1 mol of O2 , If the gases are in thermal
equilibrium then
(A) Only the average kinetic energy of the molecule of H2 and O2 is
same.
(B) Average speed of the molecule of H2 and O2 is same.
(C) Only the specific heat at constant pressure of two gases is
same.
(D) (d) The specific heat at constant pressure and the kinetic energy
are same for both the gases.

Solution: (D) The specific heat at constant pressure (C p) is the


amount of heat required to raise the temperature of one gram through 1°C when the
pressure of the gas is kept constant. Again, the mean kinetic energy per molecule (3/2)kT
depends only upon temperature. Clearly both the specific heats at constant pressure and
mean kinetic energy are depending on the temperature which is again same for the two
gases.

53: Two systems are in thermal equilibrium. The quantity which is common for them is
(A) Heat (B) Momentum
(C) Temperature (D) Specific heat
Solution: (C)

54: Mean molecular weight is defined as


(A) the number of free particles per positron mass
(B) the number of free particles per electron mass
(C) the number of free particles per neutral mass
(D) the number of free particles per photon mass
Solution: (D)

Join Elite Batch – Physics ( Batch -2 ) - Whatsapp -9264953797

Click Here - https://imjo.in/JKBjbX ( Batch Starting 01 March )


Final Revision Questions
2020
– Elite Batch

55: Which one of the following statements is true about a gas undergoing an
adiabatic change
(A) The temperature of the gas remains constant
(B) The pressure of the gas remains constant
(C) The volume of the gas remains constant
(D) The gas is completely insulated from the surroundings
Solution: (D)

56: If an ideal gas is allowed to expand adiabatically, the work done is equal to
(A) The loss in heat
(B) The loss in internal energy
(C) The gain in internal energy
(D) The gain in enthalpy
Solution: (B)

57: For the Boyle’s law to hold, the necessary condition is


(A) Isothermal (B) Adiabatic
(C) Isobaric (D) Isochoric
Solution: (A)

58: Specific heat of a gas undergoing adiabatic changes is


(A) zero (B) infinite
(C) positive (D) negative
Solution: (B)

59: The internal energy of the system remains constant when it undergoes
(A) a cyclic process (B) an adiabatic process
(C) an isothermal process (D) an isobaric process
Solution: (C)
Join Elite Batch – Physics ( Batch -2 ) - Whatsapp -9264953797

Click Here - https://imjo.in/JKBjbX ( Batch Starting 01 March )


Final Revision Questions
2020
– Elite Batch

60: The first law of thermodynamics incorporates the concepts of


(A) conservation of energy
(B) conservation of heat
(C) conservation of work
(D) equivalence of heat and work
Solution: (D)

OBJECTIVE

1: A long string with a charge of  per unit length passes through an imaginary cube of edge a.
The maximum flux of the electric field through the cube will be
λa √2 λa
(A) εo (B)
εo
2
6 λa √3 λa
(C)
εo (D)
εo

Solution : The maximum length of the string which can fit into the cube is √3 a, equal to its

body diagonal. The total charge inside the cube is √ 3 aλ , and hence the total flux
√3 aλ
through the cube is εo .
 (D)

2: Potential in the x-y plane is given as V = 5(x 2 + xy) volts. The electric field at the point (1, 2) will
be
(A) 3J V/m (A) 5 J V/m
(C) 5J V/m (D) 3J V/m

∂V
Solution : Ex =  ∂x = (10 x + 5y) = 10 + 10 = 0
∂V
Ey =  ∂x = 5x = 5

 E=− 5 j V /m .
⃗ ^
 (B)

Join Elite Batch – Physics ( Batch -2 ) - Whatsapp -9264953797

Click Here - https://imjo.in/JKBjbX ( Batch Starting 01 March )


Final Revision Questions
2020
– Elite Batch

3: In the circuit shown, the equivalent capacitance C


between the points A and B is
C C
(A) C/5 (B) C/3 A B
(C) C/2 (D) C C

E
Solution: Rearranging the circuit, the points E and D C C

are at the same potentital (by symmetry).  B


A A
6C
Then the capacity between E and D can be C C
D
removed.
1 1 1
= +
C C C
'

 C = C/2
C C
2 and 2 are in parallel.
C C
+
Hence Ceq = 2 2 =C
 (D)

^ ^ ^
4: Electric field in a region is given by ⃗E=(2 i +3 j−4 k ) V/m. The potential difference between
points (0, 0, 0) and (1,2,3) will be
(A) 2 V (B) 5 V
(C) 4V (D) 6 V

Solution: P.d. across the points =  ⃗E . Δ⃗r


^ ^i +2 ^j+3 k^ )
(2 ^i +3 ^j−4 k).(
V2 – V 1 =  = 2  6 + 12 = 4 volts.
 (C)

5: What is the mechanical work done in pulling the slab out of the capacitor after disconnecting it
from the battery
1 2
E C (ε r −1 )
(A) 2 (B) E2C
(C) E C (r  1)
2
(D) none of these

Solution: Work done = change in potential energy = U2 – U1


2
v1 = (1/2) EC

Join Elite Batch – Physics ( Batch -2 ) - Whatsapp -9264953797

Click Here - https://imjo.in/JKBjbX ( Batch Starting 01 March )


Final Revision Questions
2020
– Elite Batch

2
1 (EC ) 1 E2 C 2
= ε
v2 = 2 C' 2 C r = (1/2) E2 Cr
 Work done = (1/2) E2C (r – 1).
 (A).

6: Two point charges each of charge +q are fixed at Y


(+a, 0) and (-a, 0). Another positive point charge q
q q
placed at the origin is free to move along x- axis. The q
charge q at origin in equilibrium will have X
(-a, 0) (+a, 0)
(A) maximum force and minimum potential energy.
(B) minimum force & maximum potential energy.
(C) maximum force & maximum potential energy.
(D) minimum force & minimum potential energy.

Solution: The net force on q at origin is


2 2
1 q ^ 1 q
⃗F = ⃗F + F⃗ . i+ . (− ^i )=0^
1 2 = 4 πε 0 r 2 4 πε 0 r 2
The P.E. of the charge q in between the extreme charges at a distance x from the
origin along +ve x axis is
2 2
1 q 1 q
. + . 2
U = 4 πε 0 x 4 πε 0 ( a +x)
1 1 1
= 4 πε 0
. q2 [
+
a−x a+x.
]
2
dU q 1 1
=
[ −
dx 4 πε 0 (a−x ) ( a+x )2
2 ]
For U to be minimum
2
dU dU
=0 , >0 ,
dx dx 2
 (a-x)2 = (a + x)2
 a + x =  (a – x)
 x = 0, because other solution is irrelevent.
Thus the charged particle at the origin will have minimum force and minimum P.E.
 (D).

7: In a parallel plate capacitor of capacitance C, a metal sheet is inserted between the plates,
parallel to them. The thickness of the sheet is half of the separation between the plates. The
capacitance now becomes
(A) 4C (B) 2C
(C) C/2 (D) C/4

Join Elite Batch – Physics ( Batch -2 ) - Whatsapp -9264953797

Click Here - https://imjo.in/JKBjbX ( Batch Starting 01 March )


Final Revision Questions
2020
– Elite Batch

Solution : Before the metal sheet is inserted, C = ε o A /d


After the sheet is inserted, the system is equivalent to two capacitors in series, each of
'
εo A
C= =4 C
capacitance d/4
The equivalent capacity is now 2C.
 (B)

8: The value of q if it floats in air is q


mgε r σ m
(A) σ (B) mgε r r
2mgε r
(C) σ (D) none of these

Solution : The magnitude of electric field due to the charged qE = q(/r)


σ
conducting plate is E = ε r
q

As the charged particle is floating in air (neglecting the


buoyant force due to air we obtain) mg
mg = qE
mg
 q= E
mgε r
 q = σ
 (A)

9: The capacitance of an filled parallel plate capacitor is 20 F. The separation between the plates
is doubled and the space between the plates is then filled with wax giving the capacitance a new
value of 401012 farads. The dielectric constant of wax is
(A) 12.0 (B) 10.0
(C) 8.0 (D) 4.2

εo A Kε o A
Solution : 1010 =12 d . 401012 = 2d
 K=8
 (C)

10 : The ratio of the time periods of small oscillation of q k q


the insulated spring and mass system before and + +
after charging the masses is m m

Join Elite Batch – Physics ( Batch -2 ) - Whatsapp -9264953797

Click Here - https://imjo.in/JKBjbX ( Batch Starting 01 March )


Final Revision Questions
2020
– Elite Batch

(A)  1 (B) > 1


(C)  1 (D) = 1
1 q2
.
4 πε 0 ( ℓ0 +x 0 )2
Solution : In equilibrium of the charged small bodies = kx0 where x0 is the
elongration in the spring in equilibrium.
Let a further small elongation of x is made in the spring.
Then net restoring force on any of the charged particle is given by,
1 q2
F=
[ k ( x 0 + x )−
4 πε 0 ( ℓ 0 + x 0 + x )2 ]
= kx. Since x < < x0 from (1)
2k
 a= m x
m×m
As F = a where  = m+ m  a =  2 x,

2k m
Hence  = √ m  T = 2  √ 2k
m
In absence of charge is T0 = 2 √2k .
T
=1
Therefore T0

 (D)

11: The charge flowing across the cell on closing the key k is
equal to C C
(A) CV (B) CV/2
1 2
(C) 2CV (D) zero

Solution: When the key is kept open, the charge drawn from the source is
C
Q = CeqV = 2 V
When the key is closed the capacitor 2 gets short circuited
And Ceq = C
 Q = CV
Join Elite Batch – Physics ( Batch -2 ) - Whatsapp -9264953797

Click Here - https://imjo.in/JKBjbX ( Batch Starting 01 March )


Final Revision Questions
2020
– Elite Batch

C
charge flown through cell Q Q = 2 V
 (B) is correct choice.

12: The conducting spherical shells shown in the


figure are connected by a conductor. The b
capacitance of the system is
ab
a
(A) 40 b−a (B) 40 a
2
a
(C) 40 b (D) 40 b−a

Solution: Hence, the capacitance of the system is the capacitance due to outer sphere of radius
b.
 C = 4ob
(C) is correct choice.

13: A point charge q moves from point P to S along y

the path PQRS in a uniform electric field ⃗ E 


E
pointing parallel to the positive direction of the P(a, b, 0)

x-axis. The coordinate of the point P, Q, R and S


are (a, b, 0), (2a, 0, 0), (a, b, 0) and Q(2a, 0, 0)
x
(0, 0, 0) respectively. The work done by the field S(0, 0, 0)
100V
in the above process is given by the expression R(a, b, 0)

(A) qaE (B)  qaE


2 2 2 2
(C) q √ ( a +b ) E (D) 3qE √ ( a +b )

Solution: The work done is independent of the path followed and is equal to ( q ⃗E ) . ⃗r ,
where ⃗r = displacement from P to S

Here, ⃗r = a ^i −b ^j , while ⃗E = E ^i
^ ^
 work = (qE ^i ). ( a i + b j ) =  qaE
Hence, B is correct choice.

Join Elite Batch – Physics ( Batch -2 ) - Whatsapp -9264953797

Click Here - https://imjo.in/JKBjbX ( Batch Starting 01 March )


Final Revision Questions
2020
– Elite Batch

14: Charge Q is given to the upper plate of an isolated parallel plate Q


capacitor of capacitance C. The potential difference between the
plates
Q
Q C
(A) C (B) 2
Q /2
(C) C (D) zero

Solution: In general, for charge Q1 and Q2on upper and lower plate respectively the charge
distributions on outer and inner part of the plates are shown in figure.
Here Q1 =Q, Q2 = 0
 Charge on inner side of Q1
Q1+Q2
Q Q 2

plate are 2 and  2 


Q1 Q2 (Q1Q2)
2
respectively. Q2
2
Q1+Q2
Q /2
2
Hence V = C
Hence (C) is correct choice.
15: A particle A of mass m and charge Q moves directly towards a fixed particle B. Which has
charge Q. The speed of A is v when it is far away from B. The minimum separation between the
particle is proportional to
(A) v (B) v2
1
(C) v (D) v2

Solution: From Conservation of energy


(KE+EPE)minimum separation = (KE+EPE)far away
2
1 Q 1
= mv 2 +0
 0+ 4 πε o r min 2
1
2
 rmin  v
Hence, (D) is correct choice.

16: A dipole of dipole moment ⃗p is kept along an electric field ⃗ E such that ⃗E and ⃗p are in
the same direction. Find the work done in rotating the dipole by an angle .
(A) W = 2Ep (B) W = –2Ep
(C) W = Ep (D) W = –Ep
Ans. (A)

Join Elite Batch – Physics ( Batch -2 ) - Whatsapp -9264953797

Click Here - https://imjo.in/JKBjbX ( Batch Starting 01 March )


Final Revision Questions
2020
– Elite Batch

Solution : W = U = U2 – U1
Now U2 = (Ep cos ) = Ep
U1 = (Ep cos 0) = Ep
 W = 2Ep.

17: A simple pendulum of mass m and length  carries a charge


q. Find its time period when it is suspended in a uniform
electric field region as shown in figure. E

ℓ ℓ
(A) T = 4  √√ g2 +( Eq/m)2

(B) T = 2  √√ g2 +( Eq/m)2

(C) T = 2 2
Ans. (B)
√√ g +( Eq/m)2
2
(D) T = 2 
ℓ2
√ g2 +( Eq/ m)2

Solution: Time period of the pendulum



= 2 √
Tension in the string in equiblirium position
geff

Here, geff = mass of bob


√(mg)2+( Eq)2
= m

= √ g 2+( Eq/m )2

T=2 √ √ g +( Eq/m)2 .
2

18: A particle of mass 100 gm and charge 2 C is released from a distance of


50 cm from a fixed charge of 5 C. Find the speed of the particle when its distance from the fixed
charge becomes 3 m. Neglect any other force.
(A) –1.73 m/s (B) 2.73 m/s
(C) –2.73 m/s (D) 1.73 m/s.
Ans. (D)

Solution: Loss of potential energy = gain in kinetic energy


U1 – U2 = K.

Join Elite Batch – Physics ( Batch -2 ) - Whatsapp -9264953797

Click Here - https://imjo.in/JKBjbX ( Batch Starting 01 March )


Final Revision Questions
2020
– Elite Batch

1 1
[ ] KQq

r1 r2
= ½ mv2 – 0
2 kQq r 2 −r 1
v=
√ [ ]
m r1 r2

2×9×10 9 ×5×10−6 ×2×10−6 ×2 .5


= √ 0 .1×3×0 . 5 = 1.73 m/s.

19: If a point charge q is placed at the centre of a cube what is the flux linked
with the cube?
1 3
φ= q φ= q
(A) ∈o (B)
∈o
6 1
φ= q φ= q
(C)
∈o (D)
6 ∈o

Ans. (A)
1
Solution : From gauss’s law, flux linked with a closed body is
( )
∈o
times the charge
1
φ= q
enclosed. The cube encloses a charge q so flux linked with cube, ∈o
20: If a point charge q is placed at the centre of a cube what is the flux linked
with each face of the cube?
1 3
φ= q φ= q
(A) ∈o (B)
∈o
6 1
φ= q φ= q
(C)
∈o (D)
6 ∈o
Ans. (B)

Now as cube is a symmetrical body with a faces and the point charge is at its centre,
so electric flux linked with each face will be
1 q
φ F = φ=
6 6 ∈o

21: Supposing that the earth has a surface charge density of 1 electron/m 2; calculate
earth's potential

Join Elite Batch – Physics ( Batch -2 ) - Whatsapp -9264953797

Click Here - https://imjo.in/JKBjbX ( Batch Starting 01 March )


Final Revision Questions
2020
– Elite Batch

(A)  0.115Volt (B)  0.110 Volt


(C)  0.105Volt (D)  0.112 Volt
Ans. (A)

Solution: If R be the radius and  the surface charge density of earth, then the total charge q on
its surface is given by
Q = 4R2

(i) The potential V at a point on earth's surface is same as if the entire charge q were
concentrated as its centre. Thus
2
1 q 1 4 πR σ Rσ
=
V= 4 πε 0 R = 4 πε 0 R εo
Substituting the given value:
6 . 4×10 6 ×(−1. 6×10−19 )
V= 8 . 9×10−12
=  0.115 N-m/C = 0.115 J/C =  0.115Volt.
22: Supposing that the earth has a surface charge density of 1 electron/m 2; calculate
electric field just outside earth's surface.
The electronic charge is  1.61019C and earth's radius is 6.4106m.
(0 = 8.91012C2/N-m2)
8
(A)  1.810 N/C (B) + 1.8108 N/C
9
(C)  1.810 N/C (D) + 1.8109 N/C
Ans. (A)

Solution: (ii) Again, the electric field E just outside the earth's surface is same is if the entire
charge q were concentrated at this cetnre. Thus
2
1 q 1 4 πR σ σ
2
=
E= 4 πε 0 R 2 = 4 πε 0 R εo
Substituting the given value:
(−1. 6×10−19 ) C / m2
−12 2 2
E = 8. 9×10 C / N −m
=  1.8108 N/C
The minus sign indicates that E is radially inward.

23 : A circular wire of radius R carries a total charge Q distributed uniformly over its circumference.
A small length of the wire subtending angle  at the centre is cut off. Find the electric field at the
centre due to the remaining portion .

Join Elite Batch – Physics ( Batch -2 ) - Whatsapp -9264953797

Click Here - https://imjo.in/JKBjbX ( Batch Starting 01 March )


Final Revision Questions
2020
– Elite Batch

Q Q θ
(A)
=
4 π ε 0 R2
2
sin ( θ )
(B)
= 2
4 π ε0 R ()
2
sin
2
Q θ Q θ
(C)
= 2
4 π ε0 R 2
sin
4 () (D)
= 2
4π ε R
sin ( )
0
82

Ans. (B)

Solution : Electric field due to an arc at its centre is

kλ θ
R ()
2 sin
,2 Where k =
1
4 pe 0 ,

 = angle subtended by the wire at the centre,
 = Linear density of charge.
Let E be the electric field due to remaining portion.
Since intensity at the centre due to the circular wire is
zero.
Applying principle of superposition.
kλ θ
R
2 sin n+
2 ()
^ E⃗ =0

1 Q θ
E|=
|⃗ .
4 πε 0 R 2 πR
.2 sin
2 ()
Q θ
¿ 2
4 π ε0 R
2
sin
2 ()
24:An electric dipole, made up of a positive and a negative charge, each of 1 C and placed at a distance 2 cm
apart, is placed in an electric field 10 5N/C. Compute the maximum torque which the field can
exert on the dipole, and the work that must be done to turn the dipole from a position  = 0 to 
= 180
(A) 4106N-m or Joule (B) 4109N-m or Joule
(C) 410+6N-m or Joule (D) 4103N-m or Joule
Ans. (D)

Solution : The torque exerted by an electric field E on a dipole of moment p is given by


 = pE sin,

Where  is the angle which the dipole is making with the field.
 is a maximum, when  = 90. That is
 max = pE

Join Elite Batch – Physics ( Batch -2 ) - Whatsapp -9264953797

Click Here - https://imjo.in/JKBjbX ( Batch Starting 01 March )


Final Revision Questions
2020
– Elite Batch

Here p = q(2) = 1106  0.02C/m and E = 105N/C


 max = 1106  0.02  105 = 2103N-m

The work done in rotating the dipole from an angle o to  is given by


θ

W=
∫θ pE sin θ dθ
o = pE (coso cos)
Here o = 0 and  = 180
 W = pE (cos0 cos180) = 2pE = 4103N-m or Joule

25: A parallel-plate air capacitor has a plate area of 100cm 3 and separation 5mm. A potential
difference of 300V is established between its plates by a battery. After disconnecting a battery,
the space between the plates is filled by ebonite (K = 2.6). Find out new potential difference
between plates of the capacitor.
(A) 115V (B) 120V
(C) 110V (D) 125V
Ans. (A)

26: A parallel-plate air capacitor has a plate area of 100cm 3 and separation 5mm. A potential
difference of 300V is established between its plates by a battery. After disconnecting a battery,
the space between the plates is filled by ebonite (K = 2.6). Find out initial and final
capacitances of the capacitor.
(A) 2.771011F, 4.61011F (B) 1.771011F, 4.61011F
(C) 1.771011F, 6.61011F (D) 1.771013F, 4.61011F
Ans. (B)

27: A parallel-plate air capacitor has a plate area of 100cm 3 and separation 5mm. A potential
difference of 300V is established between its plates by a battery. After disconnecting a battery,
the space between the plates is filled by ebonite (K = 2.6). Find out initial and final surface-
density of charge on the plates.

(A) 5.3110+7C/m2 (B) 6.31107C/m2


7
(C) 5.3110 C/m2 (D) 9.31107C/m2
Ans. (C)

Solution: Capacity of the parallel plate air capacitor


εo A 8 .86×10−12×100×10−4
=
= d 5×10−3 = 1.771011F
Final capacity of the capacitor with dielectric between the plates is
C = KC = 2.61.771011, C = 4.61011F
Initial charge on the capacitor 1.771011  300 = 5.31109C
Since, the battery has been disconnected, the charge remains the same, therefore the
new potential difference is

Join Elite Batch – Physics ( Batch -2 ) - Whatsapp -9264953797

Click Here - https://imjo.in/JKBjbX ( Batch Starting 01 March )


Final Revision Questions
2020
– Elite Batch

q 5 .31×10−9
'
=
V = C 4 . 6×10−11
= 115V
The surface density of charge remains the same in both the cases, i.e.,
q 5. 31×10−9
 = A = 100×10 = 5.31107C/m2
−4

28: A spherical condenser has 10cm and 12 cm as the radii of q2


q1
inner and outer spheres. The space between the two is R
filled with a dielectric of dielectric constant 3. Find the
capacity when r

the outer sphere is earthed.

(A) 41010F (B) 11010F


(C) 21010F (D) 61010F
Ans. (C)
ab
Solution: C = 4k0
( )b−a =
3
9×10 9
×
0 .1×0 .12
0 .12−0 .1 = 21010F
29: A spherical condenser has 10cm and 12 cm as the radii of q2
q1
inner and outer spheres. The space between the two is R
filled with a dielectric of dielectric constant 3. Find the
capacity when r

the inner sphere is earthed.

15 32
×10−10 F ×10+10 F
(A) 32 (B) 15
32 32
×10−9 F ×10−10 F
(C) 15 (D) 15
Ans. (D)

By choice of reference potential at infinity and the potential of earthed conductor zero,
the given system can be visualised as combination of two spherical capacitors,
both being at same potential difference. Connection wise these may be considered
to be in parallel connection.
ab
 C = 4ob + 4k0
( )
b−a =
32
15
×10−10 F

30: A charge Q is distributed over two concentric hollow spheres of radii r and R (R >r) such that
the surface densities are equal. Find the potential at the common centre.

Join Elite Batch – Physics ( Batch -2 ) - Whatsapp -9264953797

Click Here - https://imjo.in/JKBjbX ( Batch Starting 01 March )


Final Revision Questions
2020
– Elite Batch

1 Q ( R+ r ) 1 Q ( R−r )
(A) 4 πε 0 R 2 +r 2 (B) 4 πε 0 R 2 +r 2
2
1 Q ( R+r ) 1 Q ( R+r )
(C) 4 πε 0 R2 −r 2 (D) 4 πε 0 R2 +r 2
Ans. (A)
Solution: q1 + q2 = Q . . . (i)
q1 q2
2
= 2
 = 4 πr 4 πR . . . (ii)
from (i) and(ii)
Qr 2 QR 2
q1 = ( r 2 + R2 ) q2 = ( r 2 + R2 )
1 q 1 q2
Vcentre = V1 + V2 = 4 πε 0 ( r
+
R )
1 Q ( R+ r )
= 4 πε 0 R 2 +r 2

31: An electric dipole of dipole moment P is placed in a uniform electric field E is stable equilibrium
position. Its moment of inertia about the centroidal axis is I. If it is displaced slightly from its
mean position find the period of small oscillation.
PE I
(A)
T =2 π
√ I (B)
T =2π
√ PE
I I
(C)
Ans. (B)
T =4 π
√ PE (D)
T =4 π 2
√ PE


Solution: When displaced at an angle θ , E  +q
from its mean position the mean q +q
   
position the magnitude of restoring P q 
torque is

τ =−PEθ

For small angular displacement sin  


τ =−PEθ
The angular acceleration is,
τθ pE
α= =−
I I ( )
θ=−cos 2 θ

Join Elite Batch – Physics ( Batch -2 ) - Whatsapp -9264953797

Click Here - https://imjo.in/JKBjbX ( Batch Starting 01 March )


Final Revision Questions
2020
– Elite Batch

PE
ω2 =
Where I
I

T =2π
√ PE

32: Figure shows two conducting thin concentric shells of q

R2
radii r and 3r. The outer shell carries charge q. Inner
shell is neutral. Find the charge that will flow from inner r

shell to earth after the switch S is closed.


s

3r

q q
− +
(A) 3 (B) 2
q q
− +
(C) 3 (D) 3
Ans. (D)

Solution: Let q be the charge on inner shell when it is earthed.


Vinner = 0
1 q' q

[ ]
+ =0
4 π ∈o r 3 r

 q' =−q /3
q
+
i.e. 3 charge will flow froms inner shell to earth.
33: A capacitor stores 10C charge when connected across a battery. When the gap between the
plates is filled with a dielectric, a charge of 20 C flows through the battery. Find the dielectric
constant of the dielectric.
(A) k = 2 (B) k = 4
(C) k = 3 (D) k = 1
Ans. (C)

Solution: In absence of dielectric


Q = CV = 10C . . . (1)

Join Elite Batch – Physics ( Batch -2 ) - Whatsapp -9264953797

Click Here - https://imjo.in/JKBjbX ( Batch Starting 01 March )


Final Revision Questions
2020
– Elite Batch

With dielectric
Q = kCV = 30C . . . (2)
From (i) and (2)
k=3

34: Two infinitely large sheets S1 and S2 having surface charge densities 1 and 2 (1 > 2)
respectively are placed at a distance d. Find the work done by the electric field when a charge
particle ‘ Q’ is displaced by a distance ‘a’ (a < d) at an angle 45 0 with the normal of the sheets. It
is assume that the charge does not affect the surface charge densities of the two plates.
q( s 2 - s 1) q( s 1 - s 2 )
WPQ = d WPQ = d
(A) 2 2 (B) -2 2
q( s 1 - s 2 ) q( s 2 - s 1)
WPQ = d WPQ = d
(C) 2 2 (D) -2 2

Ans. (C)
Ans. Electric field at any point between the plate
æs s ö
=ç 1 + 2 ÷
è2 Î o 2 Î o ø
P
W PQ =−q ∫ ⃗E . ⃗
dr
Q
P
æs s2 ö
= - qòç 1 - ÷ dr cos 45o
Q è 2 Î o 2 Î o ø
0
æs s2 ö
= - qòç 1 - ÷ cos 45o dr
d è 2 Î o 2 Î o ø
1 1 o
- q( s 1 - s 2 ) r
=
2Îo 2 d
q( s 1 - s 2 )
WPQ = d
2 2

35.: A positive charge (+q) is located at the centre of a circle as shown in figure. W 1
is the work done in taking a unit positive charge from A to B and W 2 is the work
done in taking the same charge from A to C. Then.
(A) W 1 > W2 (B) W 1 < W2
(C) W 1 = W2 (D) W1 = W2 = 0

Solution: Point A, B and C are at the same distance from charge +q; hence electrical potential is
the same at these points, i.e. There is no potential difference between A, B and C.
Hence W1 = W2 = 0.

Join Elite Batch – Physics ( Batch -2 ) - Whatsapp -9264953797

Click Here - https://imjo.in/JKBjbX ( Batch Starting 01 March )


Final Revision Questions
2020
– Elite Batch

36. A point Charge q is placed inside the cavity of a metallic shell.


Which one of the diagram correctly represents the electric lines of force.

(A) (B)

(C) (D)

Solution: (C) Because electric field inside the conductor is zero and electric field lines are
perpendicular to Gaussian surface.

37.: A soap bubble has radius R, charge Q, surface tension T. Find the excess
pressure in it.
32π 2 R 2 ε0T - q 2 64π 2 R 3ε0T - q 2
(A) 32π 2 R 4 ε0 (A) 32π 2 R 4 ε0
128π 2 R 3ε0T - q 2
(C) 32π 2 R 4 ε0 (D) none of these
Solution: (C)
4T q2
Pexces  
R 32 2R 4 0

38.. A charge of 2C is brought from B to C along the path as shown by arrow in the
figure. The work done is
(A) 0.75 J (B) 0.6J B 2 F
(C) 0.06J (D) 0.075J

Solution: (D)
Join Elite Batch – Physics ( Batch -2 ) - Whatsapp -9264953797
80 cm

Click Here - https://imjo.in/JKBjbX ( Batch Starting 01 March )

C
60 cm
Final Revision Questions
2020
– Elite Batch

q1q2  1 1
w   
4 0  rf ri 
 1 1 
 10  2 1012  9  109   
 0.6 0.8 
= 0.075 J

  

 20 i + 15 j  N/C
39. The electric field intensity at a point is   . Considering potential
at origin to be zero, the potential at P (2, 2) is
  
   

  40 i + 60 j V   10 i + 15 j  V
(A)   (B)  
(C) –100V (D) 20V

V =    E x dx   E y dy    E x  x  E y  y   100V
x y

Solution: (C) 
 0 0 

40. The equivalent capacitance between A and B is (each of the capacitors obtained is of
capacitance equal to C)
B’

A’

1 3
C C
(A) 2 (B) 5
5 2
C C
(C) 3 (D) 5
Ans. (C)
1

2 1 2
B
3
5 4 4 3 3 2

4 A D E B
5

D
A=E B

Join Elite Batch – Physics ( Batch -2 ) - Whatsapp -9264953797

Click Here - https://imjo.in/JKBjbX ( Batch Starting 01 March )


Final Revision Questions
2020
– Elite Batch

2C  C 5C
 CAB  C 
3C 3

41. 28. Page 426 Four large metal plates are located a small distance apart from one another as
shown in the Fig. 24. The extreme plates are connected by means of a conductor, while a
potential difference V is applied to the internal plates. Find the electric fields between the
neighbouring plates
V V V V
E23  , E34  E12  , E23  E34 
(A) d 2d (B) 2d 2
(C) zero (D) None of these

Ans. (A) We may take the plates to be connected as shown in the Fig. 56. Let E 23 be the
electric field between plates 2 and 3. Then from the fact that electric field = rate of
V
E22 
change of potential, d.
1 2 4 4

– + + –– +

+ –

– + + –– +

+ –

– + + –– +

A B
Applying `loop rule' to the loop
`A – 1 – 2 – 3 – 4 – B – A'
V
V' – V – + V'  V' = 2
where V' is the numerical value of the potential difference
between plates 1 and 2 or 3 and 4
V
 E12 or E34 
2d

E
0
 1 (density of charge on the plate 1)
V
 0 E12  0  4
2d (numerically)
The surface density of charge of the right surface of plate

2 is
 '  0 ,E23  0 Vd
 2 (surface density of charge on plate 2)

Join Elite Batch – Physics ( Batch -2 ) - Whatsapp -9264953797

Click Here - https://imjo.in/JKBjbX ( Batch Starting 01 March )


Final Revision Questions
2020
– Elite Batch

0 V  0 V 3  0 V
  
2d d 2 d
Thus
V V
E23  E12 or E34 
(i) d 2d
V 3 V
1  4  0 2  0   0
(ii) 2d 2 d.

42. A simple pendulum consists of a small sphere of mass m suspended by a


thread of length l. The sphere carries a positive charge q. The pendulum is
placed in a uniform electric field of strength E directed vertically upwards. With
what period will the pendulum oscillate if the electrostatic force acting on the
sphere is less that the gravitational force? Assume the oscillations to be small
1/2 1/2
l   ml 
T = 2π   T = 2π  
(A) g (B)  qE 
1/2 1/2
   
 l   l 
T = 2π   T = 2π  
  g - qE     g + qE  
  m     m  
(C) (D)

Solution: Let x be the small displacement given to the pendulum such that the angle  is small.
The forces acting at A are
(i) tension T along the thread
(ii) weight mg acting vertically
downwards. qE
(iii) electrical force qE vertically upwards. θ
l T
The resultant force vertically down wards is (mg –
qE). Therefore A
qE
g'  g  B θ
Net acceleration m
Time period  mg  qE  sinθ mg  mg  qE  cosθ
1/ 2
 
l  l 
T  2  2 
g' qE 
g  
 m 

43. A ring has charge Q and radius R. If a charge q is placed at its centre then the
increase in tension in the ring is

Join Elite Batch – Physics ( Batch -2 ) - Whatsapp -9264953797

Click Here - https://imjo.in/JKBjbX ( Batch Starting 01 March )


Final Revision Questions
2020
– Elite Batch

Qq
(A) 4πε0 R 2 (B) zero
Qq Qq
(C) 4π 2 ε0 R 2 (D) 8π 2 ε0 R 2

T cos T cos
Solution: Consider a small element AB,  is very small. Then
AB = R(2)  
A B
Q Q T T
dQ =  2R   
Change on AB is 2 R 
q
dQ  q Qqθ
2T sinθ = =
4π 0 R 2
4π 2 0 R 2 T sin  T cos
Qq Qq
2Tθ = or T= 2
4π 0 R
2 2
8π 0 R 2

44. A charge Q is uniformly distributed over a large plastic plate. The electric field
at point P, close to the centre of the plate is 10Vm -1 . If the plastic plate is
replaced by a copper plate of the same dimensions and carrying same charge Q
then the electric field at the point P will be
(A) 5Vm-1 (B) zero
-1
(C) 10Vm (D) 20Vm-1

Solution: (C) because E =  0

45. A hemisphere of radius r is placed in a uniform electric field of strength E. The


electric flux through the hemisphere is

(A 2Er2 (B) –Er2


(C) -2Er2 (D) zero
x

A B

Join Elite Batch – Physics ( Batch -2 ) - Whatsapp -9264953797

Click Here - https://imjo.in/JKBjbX ( Batch Starting 01 March )


Final Revision Questions
2020
– Elite Batch

B X A
   E.dS   E.dS   E.dS   E.dS
Solution: A B X

 E r 2  E r 2  E r 2  E r 2
x

A dS B

E
short cut AXB is symmetrical surface
 Electric flux due to this part is zero. However, electric flux due to AB part is –
ER2.

46. 1. What is the potential difference at the centre C


Kq
(A) Zero (B) a 2

2 Kq
(C) a (D) none of these
Solution: (A) We know that potential is a scalar quantity
1 q
V = 40 r
Where r is the distance between charge and point where potential has to be found.
 total potential at center C
1 1
= 4 0 (q1 + q2 + q3 + q4). r

1
= 40r (+ q – q + q – q)
=0

47. MeV is

(A) 1.6 1019 J (B) 1.6 10 22 J


13
(C) 1.6 10 J (D) 1.6 10
13
J
Solution: (C) 1 MeV = 1.6  10–16  106 Joules
= 1.6  10–13 Joules

48. What is the angle between electric dipole moment and the electric field strength due to
it on the axial line

Join Elite Batch – Physics ( Batch -2 ) - Whatsapp -9264953797

Click Here - https://imjo.in/JKBjbX ( Batch Starting 01 March )


Final Revision Questions
2020
– Elite Batch

(A) 0 (B) 90


(C) 180 (D) none of these
Solution: (D) Direction of dipole movement will be opposite to the electric field

49. The total electric flux, leaving spherical surface of radius 1 cm, and surrounding an
electric dipole is
q
(A)  0 (B) zero
q
2q 8 r 2
(C) 0 (D) 0

+q
Solution: Electric flux passes as due to q charge of dipole  4pe 0
-q
Electric flux passes due to – q charge of dipole  4pe 0
q q
= -
\ Net flux passes due to both the charges 4pe 0 4pe 0  0

50. An isolated sphere of radius R contains a uniform volume


distribution of positive charge. Which of the curve on the graph Electric field magnitude

below correctly illustrates the dependence of the magnitude of A


B
the electric field of the sphere as a function of the distance r A,B,C,D
C
from its centre D
r
(A) A (B) B O R
(C) C (D) D
 KQTOTAL
Solution: (C) E  3  0 r for inside point and E  r2 for outside point and it
will be represented by C.

51. The variation of potential with distance R from a fixed point is as shown in figure
4
R =5m is  V  m 1
(A) the electric field at 5
æ4 ö
R =5m is ç ÷V - m +1
(B) the electric field at è5 ø ,
(C) the potential at R = 5m is + 2.5 V- m– 1 is continuous and a
decreasing function of distance.
(D) the segment AB corresponds to an equipotent surface.

Join Elite Batch – Physics ( Batch -2 ) - Whatsapp -9264953797

Click Here - https://imjo.in/JKBjbX ( Batch Starting 01 March )


Final Revision Questions
2020
– Elite Batch

Solution: (D) (The segment AB corresponding to an equipotential surface)

52. An electric dipole is placed inside a conducting shell. Mark the correct statement(s)
(A) the flux of the electric field through the shell is zero
(B) the electric field is zero at every point on the shell
(C) the electric field is not zero anywhere any where on the shell
(D) the electric field is zero on a circle on the shell.
Solution: (A) Inside the surface the total charge is zero. So flux must be zero.

53. A network of six identical capacitors, each of value C is made as shown in the
figure. Equivalent capacitance between points A and B is
(A) C/4 (B) 3C/4
(C) 4C/3 (D) 3C
A

B
 
Solution: (C) The network is equivalent to –
Therefore equivalent capacitance = [2C series C] // [C series 2C]
 2C ×C  4C
= 2 =
 2C + C  3

54. The charge flowing across the cell on closing the


key k is equal to C C

(A) CV (B) CV/2 1 2

(C) 2CV (D) zero


V
Solution: When the key is kept open, the charge drawn from the source is
C
Q = CeqV = 2 V
When the key is closed the capacitor 2 gets short circuited
And Ceq = C
 Q = CV
C
charge flown through cell Q Q = 2 V
 (B) is correct choice.

Join Elite Batch – Physics ( Batch -2 ) - Whatsapp -9264953797

Click Here - https://imjo.in/JKBjbX ( Batch Starting 01 March )


Final Revision Questions
2020
– Elite Batch

55. The capacitance of the system of parallel A1


plate capacitor shown in the figure is
2 ε 0 A1 A2
d
(A) ( A 1+ A 2) d (B)
2 ε 0 A1 A 2 A2
( A 2 −A 1 ) d
ε 0 A1 ε 0 A2
(C) d (D) d
Solution : Since the electric field between the parallel charge + A1
plates is uniform and independent of the distance,
neglecting the edge effect, the effective area of the
plate of area A2 is A1. Thus the capacitance between d 0
the plates is E
ε 0 A1
- A2
C= d
 (C)

56. The three capacitors in figure, store a total energy in J of


6μF 6μF
(A) 12 (B) 36
(C) 48 (D) 80
3μF
Solution: (C) Total capacitance  6 F
4V
Applied voltage  4v
1 1
CV 2   6  10 6  (4)2  48F
Stored energy U  2 2
57. In the circuit shown in figure C = 6 F. The charge stored in capacitor of capacity C is
C 2C 10V

(A) zero (B) 90 C


(C) 40 C (D) 60 C

Solution: (C) Both the capacitors are in series. Therefore charge stored on them
will be same.
(C)(2C) 2 2
 C   6F  4F
Net capacity  C  2C 3 3
Potential difference  10V
 q  CV  40C

Join Elite Batch – Physics ( Batch -2 ) - Whatsapp -9264953797

Click Here - https://imjo.in/JKBjbX ( Batch Starting 01 March )


Final Revision Questions
2020
– Elite Batch

58. Figure shows two parallel plates, R and S joined to a battery of voltage V and with
charges +Q and –Q.
1. the energy stored is QV.
2. the electric field strength between the plates increases uniformly from S to R.
3. the electric potential between the plates decreases uniformly from R to S. Which of
the statement is/are correct

(A) 3 only (B) 1 and 2 only


(C) 2 and 3 only (D) 1 only

Solution: (A) The electric potential between the plates decreases uniformly
from R to S.

59. In the circuit shown in figure potential difference between A and B


E = 190V

C 3C

A
 B
is C 3C

(A) 30 V (B) 60 V
(C) 10 V (D) 90 V

Solution: (C) Potential difference is divided among two capacitance C 1 and C2 in the inverse
ratio of their capacities when it is final in series. Therefore
E = 190V

C 3C


A
B
C 3C
Voltage across P and Q is
C
=190 
C  CapacitanceacrossPandQ
C
=190 
3C
P Q
C +

C 3C

equivalent capacitance between P and Q


C
=190   40 volt
15C
C
4
Join Elite Batch – Physics ( Batch -2 ) - Whatsapp -9264953797

Click Here - https://imjo.in/JKBjbX ( Batch Starting 01 March )


Final Revision Questions
2020
– Elite Batch

Potential difference between P and Q (i.e. 40 volt)


Will divide between two capacitor C and 3C which is in series. Therefore
voltage across 3C capacitance.
C
volt  40   10Volt
C  3C
Hence (C) is correct choice

60. A parallel plate capacitor has two layers of dielectrics as shown in figure. This
capacitor is connected across a battery, then the ratio of potential difference across
the dielectric layers is

k 1= 2 k2 = 6
 

d 2d
(A) 4/3 (B) 1/2
(C) 1/3 (D) 3/2

Solution: (D) Capacitance (for k  2)


0kA
C1 
d
0 A
C
C1  2C Where d
Capacitance (for k  6)
6 0 A
C2 
2d
C2  3C
Therefore ratio of potential difference across the dielectric layer is
 3/2

----------------------------------

Join Elite Batch – Physics ( Batch -2 ) - Whatsapp -9264953797

Click Here - https://imjo.in/JKBjbX ( Batch Starting 01 March )

You might also like